Вы находитесь на странице: 1из 97

Mathematics Standard X Solved Sample Paper 1 www.rava.org.

in

CLASS X (2019-20)
MATHEMATICS STANDARD(041)
SAMPLE PAPER-1

Time : 3 Hours Maximum Marks : 80


General Instructions :
(i) All questions are compulsory.
(ii) The questions paper consists of 40 questions divided into four sections A, B, C and D.
(iii) Section A comprises of 20 questions of 1 mark each. Section B comprises of 6 questions of 2 marks each. Section
C comprises of 8 questions of 3 marks each. Section D comprises of 6 questions of 4 marks each.
(iv) There is no overall choice. However, an internal choices have been provided in two questions of 1 mark each, two
questions of 2 marks each, three questions of 3 marks each, and three questions of 4 marks each. You have to
attempt only one of the alternatives in all such questions.
(v) Use of calculators is not permitted.

Section A 4.
k =2
If the nth term of an A.P. is given by an = 5n − 3 ,
Q.1-Q.10 are multiple choice questions. Select the then the sum of first 10 terms if [1]
most appropriate answer from the given options. (a) 225 (b) 245
(c) 255 (d) 270
1. If p1 and p2 are two odd prime numbers such that
p1 > p2 , then p 12 − p 22 is [1] Ans : (b) 245
(a) an even number (b) an odd number
Putting, n = 1, 10
(c) an odd prime number (d) a prime number
we get, a =2
Ans : (a) an even number
l = 47
p 12 − p 22 is an even number. S10 = 10 (2 + 47) = 5 # 49 = 245
2
Let us take p1 = 5
5. It is given that TABC + TPQR with BC = 1 .
and p2 = 3 ar (TPRQ) QR 3
Then is equal to [1]
Then, p 12 − p 22 = 25 − 9 = 16 ar (TBCA)
(a) 9 (b) 3
16 is an even number.
(c) 1 (d) 1
2. The points (7, 2) and (− 1, 0) lie on a line [1] 3 9
(a) 7y = 3x − 7 (b) 4y = x + 1 Ans : (a) 9
(c) y = 7x + 7 (d) x = 4y + 1 Since, TABC + TPQR
Ans : (b) 4y = x + 1 ar (TPRQ) 2
= AR 2
ar (TBCA) AC
The point satisfy the line, 4y = x + 1.
QR2
:BC = 1 D = 9
QR 3
3. If 1 is a root of the equation x2 + kx − 5 = 0 , then the = =9
2 4 BC 2 1
value of k is [1] 6. Ratio in which the line 3x + 4y = 7 divides the line
(a) 2 (b) − 2 segment joining the points (1, 2) and (− 2, 1) is [1]
(c) 1 (d) 1 (a) 3 : 5 (b) 4 : 6
4 2
(c) 4 : 9 (d) None of these
Ans : (a) 2
Ans : (c) 4 : 9
Since, 1 is a root of the quadratic equation
2 3 (1) + 4 (2) − 7
x2 + kx − 5 = 0 =− 4 = 4
4 3 (− 2) + 4 (1) − 7 −9 9
7. (cos 4 A − sin 4 A) is equal to [1]
b 1 l + kb 1 l − 5 = 0
2
Then,
2 2 4
(a) 1 − 2 cos2 A (b) 2 sin2 A − 1
1+k −5 =0 (c) sin2 A − cos2 A (d) 2 cos2 A − 1
4 2 4
Ans : (d) 2 cos2 A − 1
1 + 2k − 5 = 0
4 (cos 4 A − sin 4 A) = (cos2 A) 2 − (sin2 A) 2
2k − 4 = 0 = (cos2 A − sin2 A) (cos2 A + sin2 A)
2k = 4 = (cos2 A − sin2 A) (1)

To Get 20 Solved Paper Free PDF by whatsapp add +91 89056 29969 in your class Group Page 1
Mathematics Standard X Solved Sample Paper 1 www.cbse.online

= cos2 A − (1 − cos2 A) = 2 cos2 A − 1 14. The length of the diagonal of a cube that can be
inscribed in a sphere of radius 7.5 cm is .......... [1]
8. Two chords AB and CD of a circle intersect at E such
that AE = 2.4 cm , BE = 3.2 cm and CE = 1.6 cm . Ans : 15 cm
The length of DE is [1]
15. A dice is thrown once, the probability of getting a
(a) 1.6 cm (b) 3.2 cm
prime number is .......... [1]
(c) 4.8 cm (d) 6.4 cm
Ans : 1/2
Ans : (c) 4.8 cm
(Q.16-Q.20) Answer the following
16. Find the positive root of 3x2 + 6 = 9 . [1]
Ans :

We have 3x2 + 6 = 9
Taking square at both side, we get,
3x2 + 6 = 81
3x2 = 81 − 6 = 75
x2 = 75 = 25
3
Thus x =! 5
Hence 5 is positive root.
Apply the rule, AE # EB = CE # ED 17. The diameter of a wheel is 1.26 m. What the distance
2.4 # 3.2 = 1.6 # ED covered in 500 revolutions. [1]
ED = 4.8 cm Ans :

9. To divide a line segment AB in the ratio 3 : 4, we Distance covered in 1 revolution is equal to


draw a ray AX , so that +BAX is an acute angle and circumference of wheel and that is
then mark the points on ray AX at equal distances 2πr = 2πd = πd .
such that the minimum number of these points is [1] 2
Distance covered in 500 revolutions
(a) 3 (b) 4
(c) 7 (d) 10 = 500 # π # d
Ans : (c) 7 = 500 # π # 1.26

Minimum number of these points = 3 + 4 = 7 = 500 # 22 # 1.26


7
10. If the radius of the sphere is increased by 100%, the = 1980 m. = 1.98 km
volume of the corresponding sphere is increased by [1]
18. A rectangular sheet paper 40 cm # 22 cm is rolled
(a) 200% (b) 500%
to form a hollow cylinder of height 40 cm. Find the
(c) 700% (d) 800% radius of the cylinder. [1]
Ans : (c) 700% Ans :
When the radius is increased by 100%, the Given,
corresponding volume becomes 800% and thus Height, h = 40 cm, circumference = 22 cm
increase is 700%.
2πr = 22
(Q.11-Q.15) Fill in the blanks.
r = 22 # 7 = 7 = 3.5 cm
11. H.C.F. of 6, 72 and 120 is .......... [1] 2 # 22 2
Ans : 6 or
A cylinder, a cone and a hemisphere have same base
12. If α and β are the zeroes of the quadratic
and same height. Find the ratio of their volumes.
polynomialax2 + bx + c, then α + β = − b/.......... and
αβ = c/........... [1] Ans :
Ans : a, a Volume of cylinder | Volume of cone | Volume of
hemisphere
or = πr 2 h | 1 πr 2 h | 2 πr 3
3 3
Degree of remainder is always .......... than degree of
divisor. = πr 2 h | 1 πr 2 h | 2 πr 2 # h (h = r )
3 3
Ans : Smaller/less
= 1 | 1 | 2 or 3 | 1 | 2
13. Length of arc of a sector angle 45c of circle of radius 3 3
14cm is .......... [1] 19. If the median of a series exceeds the mean by 3, find
7 by what number the mode exceeds its mean? [1]
Ans : πcm
2 Ans :

Download 20 Solved Sample Papers pdfs from www.cbse.online or www.rava.org.in Page 2


Mathematics Standard X Solved Sample Paper 1 www.rava.org.in

Given, Median = Mean + 3 Thus AB = BC = AC


PQ QR PR
Mode = 3 Median – 2 Mean
= 3 (Mean + 3) – 2 Mean z =8=4 3
3 6 y
& Mode = Mean + 9
z = 8 and 8 = 4 3
Hence Mode exceeds Mean by 9. 3 6 6 y
20. 20 tickets, on which numbers 1 to 20 are written,
are mixed thoroughly and then a ticket is drawn at z = 8 # 3 and y = 4 3 # 6
6 8
random out of them. Find the probability that the
number on the drawn ticket is a multiple of 3 or 7. [1] z = 4 and y = 3 3
Ans : Thus y+z = 3 3 +4
23. If the point P ^x, y h is equidistant from the points
Total number of cases = 20
Q ^a + b, b − a h and R ^a − b, a + b h , then prove that
n ^s h = 20 bx = ay . [2]
A = favourable cases Ans :
= "3, 6, 7, 9, 12, 14, 15, 18,
We have PQ = PR
` n ^Ah = 8
Required probability = P ^Ah 8x − ^a + b hB + 8y − ^b − a hB
2 2
`
n ^Ah = 8x − ^a − b hB + 8y − ^b + a hB
2 2

= = 8 =2
n ^S h 20 5

Section B
21. Solve the following pair of linear equations by cross
multiplication method: [2]
x + 2y = 2
x − 3y = 7
Ans :

We have x + 2y − 2 = 0
x − 3y − 7 = 0
Using the formula
x y 1
= =
b1 c2 − b2 c1 c1 a2 − c2 a1 a1 b2 − a2 b1
8x − ^a + b hB + 8y − ^b − a hB = 8x − ^a − b hB + 8y − ^a + b hB
2 2 2 2

x y 1
we have = = − 2x (a + b) − 2y (b − a) = − 2x (a − b) − 2y (a + b)
− 14 − 6 −2 + 7 −3 − 2
2x (a + b) + 2y (b − a) = 2x (a − b) + 2y (a + b)
x = y = −1
− 20 5 5 2x (a + b − a + b) + 2y (b − a − a − b) = 0
x = −1 & x = 4 2x (2b) + 2y (− 2a) = 0
− 20 5 xb − ay = 0
= −1 & y =− 1
y bx = ay Hence Proved
5 5
22. In the given figure, TABC ~TPQR. Find the value of or
y + z. [2] Show that the points A ^0, 1h, B ^2, 3h and C ^3, 4h are
collinear.
Ans :
If the area of the triangle formed by the points is zero,
then points are collinear.
We have A ^0, 1h, B ^2, 3h and C ^3, 4h
Δ = 1 0 ^3 − 4h + 2 ^4 − 1h + 3 ^1 − 3h
2

= 1 0 + ^2 h^3 h + ^3 h^− 2h
2

= 1 6−6 = 0
Ans : 2
24. As a part of a campaign, a huge balloon with message
In the given figure TABC ~TPQR of “AWARENESS OF CANCER” was displayed from

To Get 20 Solved Paper Free PDF by whatsapp add +91 89056 29969 in your class Group Page 3
Mathematics Standard X Solved Sample Paper 1 www.cbse.online

the terrace of a tall building. It was held by string of OA2 = AC2 + OC2
length 8 m each, which inclined at an angle of 60c at [by Pythagoras theorem]
the point, where it was tied as shown in the figure.[2]
OA = 4 + 32 = 5 m
2

Which is the radius of the circle.


25. Find the mean of the data using an empirical formula
when it is given that mode is 50.5 and median in
45.5. [2]
Ans :

Given, Mode = 50.5


Median = 45.5
3 × Median = Mode + 2 Mean
& 3 # 45.5 = 50.5 + 2 Mean
& Mean = 136.5 − 50.5
2
Hence, Mean = 43

or
A bag contains 6 red and 5 blue balls. Find the
i. What is the length of AB ?
probability that the ball drawn is not red.
ii. If the perpendicular distance from the centre of
the circle to the chord AB is 3 cm, then find the Ans :
radius of the circle. No. of possible outcomes = 6 + 5 = 11
Ans :
No. of favourable outcome = 5
(i) Here,PA = PB = 8 m p (not red) = 11 − 6 = 5
From the figure it is clear that PA and PB are
tangents to the circle. ` = 5
11
Now, draw OP which bisects +APB and perpendicular 26. The Class XII students of a senior secondary school
to the chord AB . in Kishangarh have been allotted a rectangular plot of
land for this gardening activity as shown in figure [2]

Thus, we have
+APC = +BPC = 30c
and +ACP = +BCP = 90c Sapling of Neem tree are planted on the boundary at a
In TACP , distance of 1 m from each other. There is a triangular
+APC + +ACP + +PAC = 180c grassy lawn in the plot as shown in above figure.
After substituting the values, we get The students are to sow seeds of flowering plants on
the remaining area of the plot.
30c + 90c + +PAC = 180c
Then, taking A a origin, find the area of the triangle
+PAC = 180c − 120c = 60c in this case.
Similarly, +PBC = 60c Ans :
Thus, TAPB is an equilateral triangle.
When A is taken as origin, AD and AB as coordinate
AB = AP = BP = 8 m axes. i.e. X and Y -axes, respectively. Hence
(ii) Here, OC = 3 m coordinates of P, Q and R are respectively, (4,6), (3,2)
As, we know that, if a perpendicular drawn from the and (6,5).
centre of the circle to the chord, then it bisects the In this case, AD and AB taken as coordinate axes.
chord. Then, area of TPQR
AC = BC = AB = 8 = 4
2 2 = 1 4 ^2 − 5h + 3 ^5 − 6h + ^6 − 2h
2
In right angled TACO [a area of triangle]

Download 20 Solved Sample Papers pdfs from www.cbse.online or www.rava.org.in Page 4


Mathematics Standard X Solved Sample Paper 1 www.rava.org.in

= 1 x1 ^y2 − y3h + x2 ^y3 − y1h + x3 ^y1 − y2h x =


y
= 1
2 − 80 − 48 − 4 + 100 − 60 − 4
= 1 4 ^− 3h + 3 ^− 1h + 6 # 4 x =
y
= 1
2 − 128 96 64
= 1 − 12 − 3 + 24 = 9 sq. units. x = 1 & x =2
2 2 − 128 − 64

Section C and
y
96
= 1 & y = −3
− 64 2
Hence, x = 2 and y = − 3
27. Quadratic polynomial 2x2 − 3x + 1 has zeroes as α 2
and β . Now form a quadratic polynomial whose zeroes 29. Find the 20th term of an A.P. whose 3rd term is 7 and
are 3α and 3β . [3] the seventh term exceeds three times the 3rd term by
Ans : 2. Also find its nth term ^an h . [3]
Ans :
We have f ^x h = 2x2 − 3x + 1
If α and β are the zeroes of 2x2 − 3x + 1, then Let the first term be a , common difference be d and
n th term be an .
Sum of zeroes α + β = −b = 3
a 2 We have a3 = a + 2d = 7 (1)
Product of zeroes αβ = c = 1 a7 = 3a3 + 2
a 2
New quadratic polynomial whose zeroes are 3α and a + 6d = 3 # 7 + 2 = 23 (2)
3β is, Solving (1) and (2) we have
p (x) = x2 − ^3α + 3βh x + 3α×3β 4d = 16 & d = 4
= x2 − 3 ^α + βh x + 9αβ a + 8 = 7 & a =− 1
a20 = a + 19d = − 1 + 19 # 4 = 75
= x2 − 3 b 3 l x + 9 b 1 l a1 = a + ^n − 1h d = − 1 + 4n − 4
2 2
= 4n − 5.
= x2 − 9 x + 9
2 2 Hence n term is 4n − 5
th

= 1 ^2x2 − 9x + 9h or
2
2
or In an A.P. the sum of first n terms is 3n + 13n . Find
2 2
If α and β are the zeroes of a quadratic polynomial the 25th term.
such that α + β = 24 and α − β = 8 . Find the Ans :
quadratic polynomial having α and β as its zeroes.
We have Sn = 3n + 13n
2
Ans : 2
We have α + β = 24 ...(1) a n = Sn − Sn − 1
α−β = 8 ...(2) a25 = S25 − S24
Adding equations (1) and (2) we have 3 ^25h2 + 13 ^25h 3 ^24h2 + 13 ^24h
= −
2α = 32 & α = 16 2 2
Subtracting (1) from (2) we have = 1 $3 ^252 − 242h + 13 ^25 − 24h.
2
2β = 24 & β = 12
Hence, the quadratic polynomial = 1 ^3 # 49 + 13h = 80
2
p (x) = x2 − ^α + βh x + αβ
30. In a trapezium ABCD , diagonals AC and BD
= x2 − ^16 + 8h x + ^16h^8h intersect at O and AB = 3DC, then find ratio of areas
= x2 − 24x + 128 of triangles COD and AOB. [3]
Ans :
28. Solve using cross multiplication method: [3]
5x + 4y − 4 = 0 As per given condition we have drawn the figure
below.
x − 12y − 20 = 0
Ans :

We have 5x + 4y − 4 = 0 ...(1)
x − 12y − 20 = 0 ...(2)
By cross-multiplication method,
x y 1
= =
b2 c1 − b1 c2 c1 a2 − c2 a1 b1 b2 − a2 b1

To Get 20 Solved Paper Free PDF by whatsapp add +91 89056 29969 in your class Group Page 5
Mathematics Standard X Solved Sample Paper 1 www.cbse.online

because of AA similarity we have Ans :


TAOB ~TCOD As per question we draw figure shown below.
ar ^TCOD h 2 2 2
= CD 2 = CD 2 = CD 2 = 1
ar ^TAOB h AB ^3CD h 9CD 9
ratio = 1: 9
31. A local Outdoors Club has just hiked to the south rim
of a large canyon, when they spot a climber attempting
to scale the taller northern face. Knowing the distance
between the sheer walls of the northern and southern
faces of the canyon is approximately 175m, they
attempt to compute the distance remaining for the
climbers to reach the top of the northern rim. Using
a homemade transit, they sight an angle of depression
of 60c to the bottom of the north face, and angles of Since length of tangents from an external point to a
elevation of 30c and 45c to the climbers and top of the circle are equal,
northern rim respectively.
(a) How high is the southern rim of the canyon? At A, AX = AY (1)
(b) How high is the northern rim? At B BX = BZ (2)
(c) How much farther until the climber reaches the At C CY = CZ (3)
top? [3]
Perimeter of TABC ,
p = AB + AC + BC
= (AX − BX) + (AY − CY) + BZ + ZC)
= AX + AY − BX + BZ + ZC − CY
= AX + AY = 2AX
Thus AX = 1 p Hence Proved
2
or
In TABD, AB = AC. If the interior circle of TABC
Ans : touches the sides AB, BC and CA at D, E and F
respectively. Prove that E bisects BC.
Let’s first find the distances hs, h1 and h2 in the
Ans :
diagram below, then answer the questions.
As per question we draw figure shown below.

tan 60c = hs ; hs = 175 tan 60c = 175 3 m


175

tan 30c = h1 ; h1 = 175 tan 30c = 175 m Since length of tangents from an external point to a
175 3 circle are equal,
tan 45c = h 2
; h = 175 tan 45c = 175 m At A, AF = AD (1)
175 2
At B BE = BD (2)
(a) hs = 175 3 m is the height of the south rim.
(b) hs + h2 = 175 3 + 175 = 175 ^1 + 3 h m is the At C CE = CF (3)
height of the north rim. Now we have AB = AC
(c) h2 − h1 = 175 − 175 = 175 d1 − 1 n m is how far AD + DB = AF + FC
3 3
BD = FC ( AD = AF )
the climbers have to go to the top.
BE = EC (BD = BE, CE = CF)
32. ABC is a triangle. A circle touches sides AB and AC
produced and side BC at BC at X , X,Y and Z Thus E bisects BC.
respectively. Show that AX = 12 perimeter of TABC . [3] 33. Construct a TABC in which AB = 4 cm, BC = 5
cm and AC = 6 cm. Then construct another triangle

Download 20 Solved Sample Papers pdfs from www.cbse.online or www.rava.org.in Page 6


Mathematics Standard X Solved Sample Paper 1 www.rava.org.in

whose sides are 2 times the corresponding sides of The angle of depression of the foot of the tower is 20c.
3 (b) Distance between the tower and the building,
TABC . [3]
Ans : BC = DE = FG
In right TBCG ,
Steps of construction :
1. Draw a line segment BC = 5 cm. tan 20c = CG & 0.36 = 1.6 + 9.2
BC BC
2. With B as centre and radius = AB = 4 cm, draw
an arc. BC = 10.8 = 30 m
3. With C as centre and radius = AC = 6 cm, draw 0.36
another arc, intersecting the arc drawn in step 2 Hence, distance between the tower and the building
at the point A. = 30 m
4. Join AB and AC to obtain TABC . (c) In right TACB
5. Below BC , make an acute angle +CBX . AC = BC tan 50c
6. Along BX mark off three points B1, B2, B3 such
= 30 # 1.19 = 35.7 m
that BB1 = B1 B2 = B2 B3 .
Hence, height of the tower
7. Join B3 C .
8. From B2 , draw B2 C' | | B3 C . = AC + CE + EG
9. From C' , draw C'A' | | CA, meeting BA at the = 35.7 + 1.6 + 9.2 = 46.5 m
point A' .
Then A'BC' is the required triangle.
Section D
35. For any positive integer n , prove that n3 − n is
divisible by 6. [4]
Ans :

We have n3 − n = n (n2 − 1)
= ^n − 1h n ^n + 1h
= ^n − 1h n ^n + 1h
Thus n3 − n is product of three consecutive positive
integers.
Since, any positive integers a is of the form 3q, 3q + 1
or 3q + 2 for some integer q .
34. Hari, standing on the top of a building, sees the top of
a tower at an angle of elevation of 50c and the foot of Let a, a + 1, a + 2 be any three consecutive integers.
the tower at an angle of depression of 20c. Hari is 1.6 Case I : a = 3q
metre tall and the height of the building on which he If a = 3q then,
is standing is 9.2 mitres. [3]
(a) Draw a rough sketch according to the given a ^a + 1h^a + 2h = 3q ^3q + 1h^3q + 2h
information. Product of two consecutive integers ^3q + 1h and
(b) How far is the tower from the building? ^3q + 2h is an even integer, say 2r .
(c) Calculate the height of the tower. Thus a ^a + 1h^a + 2h = 3q ^2r h
[sin 20c = 0.34 , cos 20c = 0.94 , tan 20c = 0.36 = 6qr , which is divisible by 6.
sin 50c = 0.77 , cos 50c = 0.64 , tan 50c = 1.19 ]
Case II : a = 3q + 1
Ans :
If a = 3q + 1 then
(a) Rough sketch
a ^a + 1h^a + 2h = ^3q + 1h^3q + 2h^3q + 3h
= ^2r h^3 h^q + 1h
= 6r ^q + 1h
which is divisible by 6.
Case III : a = 3q + 2
If a = 3q + 2 then
a ^a + 1h^a + 2h = ^3q + 2h^3q + 3h^3q + 4h
= 3 ^3q + 2h^q + 1h^3q + 4h
Here ^3q + 2h and = 3 ^3q + 2h^q + 1h^3q + 4h
= multiple of 6 every q
= 6r (say)
Hari is standing at D . His height BD is 1.6 m. which is divisible by 6. Hence, the product of three
Height of the building, DF = 9.2 m consecutive integers is divisible by 6 and n3 − n is also
The angle of elevation of the top of the tower AG is divisible by 3.
50c.
To Get 20 Solved Paper Free PDF by whatsapp add +91 89056 29969 in your class Group Page 7
Mathematics Standard X Solved Sample Paper 1 www.cbse.online

or x =4
Prove that 3 is an irrational number. Hence, show Hence, x = 15, 4
that 7 + 2 3 is also an irrational number. 37. The base BC of an equilateral triangle ABC lies on
Ans : y-axis. The co-ordinates of point C are ^0, 3h . The
Assume that 3 be a rational number then we have origin is the mid-point of the base. Find the co-
ordinates of the point A and B. Also find the co-
3 =a, (a ,b are co-primes and b ! 0 ) ordinates of another point D such that BACD is a
b
a =b 3 rhombus. [4]
Ans :
Squaring both the sides, we have
As per question, diagram of rhombus is shown below.
a2 = 3b2
Thus 3 is a factor of a2 and in result 3 is also a factor
of a .
Let a = 3c where c is some integer, then we have
a2 = 9c2
Substituting a2 = 9b2 we have
3b2 = 9c2
b2 = 3c2
Thus 3 is a factor of b2 and in result 3 is also a factor
of b .
Thus 3 is a common factor of a and b . But this
contradicts the fact that a and b are co-primes. Thus,
our assumption that 3 is rational number is wrong.
Hence 3 is irrational. Co-ordinates of point B are ^0, 3h
Let us assume that 7 + 2 3 be rational equal to a , Thus BC = 6 unit
then we have Let the co-ordinates of point A be ^x, 0h

7+2 3 =
p
q ! 0 and p and q are co-primes Now AB = x2 + 9
q
Since AB = BC , thus
p p − 7q
2 3 = −7 = x2 + 9 = 36
q q
p − 7q x2 = 27 & x = ! 3 3
or 3 =
2q Co-ordinates of point A is _3 3 , 0i
Here p − 7q and 2q both are integers, hence 3 Since ABCD is a rhombus
should be a rational number. But this contradicts
AB = AC = CD = DB
the fact that 3 is an irrational number. Hence our
assumption is not correct and 7 + 2 3 is irrational. Thus co-ordinate of point D is _− 3 3 , 0i
2
36. Solve for x : b 2x l + b 2x l − 24 = 0, x ! 5 [4] or
x−5 x−5
Ans : Prove that the area of a triangle with vertices
2 ^t, t − 2h, ^t + 2, t + 2h and ^t + 3h is independent of t.
b x − 5 l + 5 b x − 5 l − 24 = 0
We have 2x 2x
Ans :

Let 2x = y then we have Area of the triangle


x−5
Δ = 1 | t ^t + 2 − t h + ^t + 2h^t − t + 2h +
y2 + 5y − 24 = 0 2
^y + 8h^y − 3h = 0 + ^t + 3h^t − 2 − t − 2h |
y = 3, − 8 = 1 62t + 2t + 4 − 4t − 12@
2
Taking y = 3 we have
2x = 3 = 4 sq. units. which is independent of t .
x−5 38. From the top of tower, 100 m high, a man observes
2x = 3x − 15
two cars on the opposite sides of the tower with the
x = 15 angles of depression 30c & 45c respectively. Find the
Taking y = − 8 we have distance between the cars. (Use 3 = 1.73 ) [4]
2x = − 8 Ans :
x−5 Let DC be tower of height 100 m. A and B be two car
2x = − 8x + 40 on the opposite side of tower. As per given in question
10x = 40 we have drawn figure below.
Download 20 Solved Sample Papers pdfs from www.cbse.online or www.rava.org.in Page 8
Mathematics Standard X Solved Sample Paper 1 www.rava.org.in

Here +CBD = +ECB = 45c due to alternate angles.


In right TABC we have
CD = tan 45c
BD
7 =1
x
x =7
In right TAEC we have
CE = tan 60c
In right TADC , AE
tan 30c = CD h−7 = 3
AD x
1 = 100 h−7 = x 3
3 x
h−7 = 7 3
x = 100 3 ...(1)
h = 7 3 +7
In right TBDC ,
= 7 ^ 3 + 1h
tan 45c = CD = 7 ^1.732 + 1h
DB
Hence, height of tower = 19.124 m
1 = 100
y
39. In the given figure, O is the centre of the circle.
& y = 100 m Determine +APC , if DA and DC are tangents and
Distance between two cars +ADC = 50c. [4]

AB = AD + DB = ^100 3 + 100h
= ^100 # 1.73 + 100h m
= ^173 + 100h m
= 273 m
Hence, distance between two cars is 273 m.

or
From the top of a 7 m high building, the angle of
elevation of the top of a tower is 60c and the angle
of depression of its foot is 45c. Find the height of the
tower. (Use 3 = 1.732 )
Ans : Ans :
Let AB be the building of height 7 m and CD be the We redraw the given figure by joining A and C to O
tower of height h . Angle of depressions of top and as shown below.
bottom are given 30c and 60c respectively. As per
given in question we have drawn figure below.

Since DA and DC are tangents from point D to the


circle with centre O , and radius is always perpendicular
to tangent, thus
+DAO = +DCO = 90c
and
+ADC + +DAO + +DCO + +AOC = 360c
50c + 90c + 90c + +AOC = 360c

To Get 20 Solved Paper Free PDF by whatsapp add +91 89056 29969 in your class Group Page 9
Mathematics Standard X Solved Sample Paper 1 www.cbse.online

230c + +AOC = 360c


+AOC = 360c − 230c = 130c
Now Reflex +AOC = 360c − 130c = 230c
+APC = 1 reflex +AOC
2
(angle subtended at centre...)
+APC = 1 # 230c = 115c
2
40. The following distribution gives the weights of 60
students of a class. Find the mean and mode weights
of the students. [4]
Weight 40- 44- 48- 52- 56- 60- 64- 68-
(in kg) 44 48 52 56 60 64 68 72
Number 4 6 10 14 10 8 6 2
of
students
Ans :

C.I. xi fi ui = xi − a fi ui
h
40-44 42 4 -3 -12
44-48 46 6 -2 -12
48-52 50 10 -1 -10
52-56 54 14 0 0
56-60 58 10 1 10
60-64 62 8 2 16
64-68 66 6 3 18
68-72 70 2 4 8
/ fi = 60 / fi ui = 18
Let a = Assumed mean = 54

Mean, x = a +
/ fi ui h
/ fi #
Mean = 54 + 18 # 4 = 55.2
60
Maximum frequency = 14
& Modal class = 52 − 56 , l = 52 , f1 = 14 ,
f0 = 10 , f2 = 10 , h = 4
Mode = 52 + 14 − 10 # 4 = 54
28 − 10 − 10
Hence, Mean = 55.2 and Mode = 54
WWW.CBSE.ONLINE

Download unsolved version of this paper from


www.cbse.online

This sample paper has been released by website www.cbse.online for the benefits of the students. This paper has been prepared by
subject expert with the consultation of many other expert and paper is fully based on the exam pattern for 2019-2020. Please note
that website www.cbse.online is not affiliated to Central board of Secondary Education, Delhi in any manner. The aim of website is
to provide free study material to the students.
Download 20 Solved Sample Papers pdfs from www.cbse.online or www.rava.org.in Page 10
Mathematics Standard X Solved Sample Paper 2 www.rava.org.in

CLASS X (2019-20)
MATHEMATICS STANDARD(041)
SAMPLE PAPER-2

Time : 3 Hours Maximum Marks : 80


General Instructions :
(i) All questions are compulsory.
(ii) The questions paper consists of 40 questions divided into 4 sections A, B, C and D.
(iii) Section A comprises of 20 questions of 1 mark each. Section B comprises of 6 questions of 2 marks each. Section
C comprises of 8 questions of 3 marks each. Section D comprises of 6 questions of 4 marks each.
(iv) There is no overall choice. However, an internal choices have been provided in two questions of 1 mark each, two
questions of 2 marks each, three questions of 3 marks each, and three questions of 4 marks each. You have to
attempt only one of the alternatives in all such questions.
(v) Use of calculators is not permitted.

4. An AP starts with a positive fraction and every


Section A alternate term is an integer. If the sum of the first 11
terms is 33, then the fourth term is [1]
Q.1-Q.10 are multiple choice questions. Select the (a) 2 (b) 3
most appropriate answer from the given options. (c) 5 (d) 6
1. The number 313 − 310 is divisible by [1] Ans : (a) 2
(a) 2 and 3 (b) 3 and 10
(c) 2, 3 and 10 (d) 2, 3 and 13 Given, S11 = 33

26 @
Ans : (d) 2, 3 and 13 11 2a + 10d = 33 & a + 5d = 3

313 − 310 = 310 (33 − 1) = 310 (26) i.e., a6 = 3 & a 4 = 2


[Since, Alternate terms are integers and the given sum
= 2 # 13 # 310 is possible]
Hence, 3 − 3
13 10
is divisible by 2, 3 and 13.
5. The areas of two similar triangles ABC and PQR are
2. A can do a piece of work in 24 days. If B is 60% more in the ratio 9 : 16. If BC = 4.5 cm , then the length of
efficient than A, then the number of days required by QR is [1]
B to do the twice as large as the earlier work is [1] (a) 4 cm (b) 4.5 cm
(a) 24 (b) 36
(c) 3 cm (d) 6 cm
(c) 15 (d) 30
Ans : (d) 6 cm
Ans : (d) 30
Since, TABC + TPQR
Work ratio of A : B = 100 : 160 or 5 : 8
ar (TABC ) 2

Time ratio = 8 : 5 or 24 : 15 = BC 2
ar (TPQR) QR
If A takes 24 days, B takes 15 days, Hence, B takes 2
30 days to do double the work. 9 = (4.5)
16 QR2
3. Value ^s h of k for which the quadratic equation
2x2 − kx + k = 0 has equal roots is/are [1] 16 # (4.5) 2
QR2 =
(a) 0 (b) 4 9
(c) 8 (d) 0, 8 QR = 6 cm
Ans : (d) 0, 8 6. If the points A (4, 3) and B (x, 5) are on the circle with
centre O (2, 3), then the value of x is [1]
Given equation is, 2x2 − kx + k = 0 (a) 0 (b) 1
On comparing with ax2 + bx + c = 0 , (c) 2 (d) 3
we get a = 2 , b = − k and c = k Ans : (c) 2
For equal roots, the discriminant must be zero.
Since, A and B lie on the circle having centre O .
D = b2 − 4ac = 0
OA = OB
^− k h2 = − 4 ^2h k = 0
(4 − 2) + (3 − 3) 2 =
2
(x − 2) 2 + (5 − 3) 2
k2 − 8k = 0
2 = (x − 2) 2 + 4
k ^k − 8h = 0
4 = (x − 2) 2 + 4
k = 0, 8
Hence, the required values of k are 0 and 8. (x − 2) 2 = 0
x =2

To Get 20 Solved Paper Free PDF by whatsapp add +91 89056 29969 in your class Group Page 1
Mathematics Standard X Solved Sample Paper 2 www.cbse.online

7. If sec 5A = cosec ^A + 30ch , where 5A is an acute is .......... [1]


angle, then the value of A is [1] Ans : 49 : 25
(a) 15c (b) 5c
(c) 20c (d) 10c 15. If P (E ) = 0.05 , the probability of ‘not E ’ is .......... [1]
Ans : .95
Ans : (d) 10c

We have, sec 5A = cosec ^A + 30ch (Q.16-Q.20) Answer the following


sec 5A = sec 690c − ^A + 30ch@ 16. If one root of the quadratic equation 6x2 − x − k = 0 is
2 , then find the value of k .
6sec ^90c − θh = cosec θ@ 3
[1]

sec 5A = sec ^60c − Ah Ans :


5A = 60c − A
We have 6x2 − x − k = 0
6A = 60c Substituting x = 2 , we get
3
A = 10c
2
8. If a regular hexagon is inscribed in a circle of radius r 6b 2 l − 2 − k = 0
3 3
, then its perimeter is [1]
(a) 3r (b) 6r 6# 4 −2−k = 0
9 3
(c) 9r (d) 12r
k = 6 # 4 − 2 = 24 − 6 = 2
Ans : (b) 6r 9 3 9
Thus k = 2 .
Side of the regular hexagon inscribed in a circle of
17. Two coins of diameter 2 cm and 4 cm respectively are
radius r is also r , the perimeter is 6r .
kept one over the other as shown in the figure, find
9. The sides of a triangle (in cm) are given below. In which the area of the shaded ring shaped region in square
case, the construction of triangle is not possible. [1] cm. [1]
(a) 8, 7, 3 (b) 8, 6, 4
(c) 8, 4, 4 (d) 7, 6, 5
Ans : (c) 8, 4, 4
We know that, in a triangle sum of two sides of
triangle is greater than the third side. Here, the sides
of triangle given in option (c) does not satisfy this
condition. So, with these sides the construction of a
triangle is not possible.
Ans :
10. Ratio of lateral surface areas of two cylinders with
equal height is [1] Area of circle = πr2
(a) 1 : 2 (b) H : h Area of the shaded region = π (2) 2 − π (1) 2
(c) R : r (d) None of these = 4π − π = 3π sq cm
Ans : (c) R : r 18. What is the ratio of the total surface area of the solid
hemisphere to the square of its radius. [1]
2πRh : 2πrh = R : r
Ans :
(Q.11-Q.15) Fill in the blanks.
= 3π2r = 3π
2
Total surface area of hemisphere
11. Numbers having non-terminating, non-repeating 1
Square of its radius r
decimal expansion are known as .......... [1]
Ans : Irrational numbers Total surface area of hemisphere|Square of radius
= 3π | 1
12. A quadratic polynomial can have at most 2 zeroes and
a cubic polynomial can have at most .......... zeroes. [1] or
Ans : 3 If the area of three adjacent faces of a cuboid are X, Y,
and Z respectively, then find the volume of cuboid.
or Ans :
If α, β, γ are the zeroes of the cubic polynomial Let the length, breadth and height of the cuboid is l, b
ax3 + bx2 + cx + d = 0 , then α + β + γ = − b and h respectively.
..........
Ans : a
X = l#b
13. If radius of a circle is 14 cm the area of the circle is Y = b#h
.......... [1]
Z = l#h
Ans : 616 cm 2

XYZ = l2 # b2 # h2
14. If the heights of two cylinders are equal and their radii
Volume of cuboid = l # b # h
are in the ratio of 7 : 5, then the ratio of their volumes

Download 20 Solved Sample Papers pdfs from www.cbse.online or www.rava.org.in Page 2


Mathematics Standard X Solved Sample Paper 2 www.rava.org.in

l2 b2 h2 = XYZ Prove that TABC is right angled at A. [2]


or, lbh = XYZ Ans :
19. Find median of the data, using an empirical relation when As per given condition we have drawn the figure
it is given that Mode = 12.4 and Mean = 10.5. [1] below.
Ans :

Median = 1 Mode + 2 Mean


3 3

= 1 ^12.4h + 2 ^10.5h
3 3

= 12.4 + 21
3 3
= 12 . 4 + 21 = 33.4
3 3
= 33 . 4 = 11.13 We have AD2 = BD # CD
3
AD = BD
20. A game of chance consists of spinning an arrow which CD AD
comes to rest pointing at one of the numbers 1, 2, 3, 4,
Since +D = 90º , by SAS we have
5, 6, 7, 8 and these are equally likely outcomes. Find
the probability that the arrow will point at any factor TADC ~TBDA
of 8 ? [1] and +BAD = +ACD;
Ans : Since corresponding angles of similar triangles are
equal
Given, Total number of points = 8
+DAC = +DBA
Total number of possible outcomes = 8
+BAD + +ACD + +DAC + +DBA = 180º
= ^1 # 8h, ^2 # 4h, ^8 # 1h, ^4 # 2h
2+BAD + 2+DAC = 180º
No. of favourable outcomes = 4
+BAD + +DAC = 90º
` P (Factor of 8) = No. of favourable outcomes +A = 90º
Total no. of possible outcomes
Thus TABC is right angled at A.
=4 =1
8 2 23. Prove that the point ^3, 0h , ^6, 4h and ^− 1, 3h are the
vertices of a right angled isosceles triangle. [2]
Section B Ans :
21. In the figure given below, ABCD is a rectangle. Find We have A ^3, 0h , B ^6, 4h and C ^− 1, 3h
the values of x and y . [2] Now AB2 = ^3 − 6h2 + ^0 − 4h2
Ans : = 9 + 16 = 25
BC = ^6 + 1h2 + ^4 − 3h2
2

= 49 + 1 = 50
CA2 = ^− 1 − 3h2 + ^3 − 0h2
= 16 + 9 = 25
AB2 = CA2 or, AB = CA
Hence triangle is isosceles.

From given figure, we have


x + y = 22 ...(1)
and x − y = 16 ...(2)
After adding equation (1) and (2), we have
2x = 38
x = 19
Substituting the value of x in equation (1), we get
19 + y = 22
Also, 25 + 25 = 50
y = 22 − 19 = 3
or, AB + CA2 = BC2
2

Hence, x = 19 and y = 3. Since Pythagoras theorem is verified, therefore triangle


22. In TABC, AD = BC, such that AD2 = BD # CD. is a right angled triangle.

To Get 20 Solved Paper Free PDF by whatsapp add +91 89056 29969 in your class Group Page 3
Mathematics Standard X Solved Sample Paper 2 www.cbse.online

or
Find the relation between x and y , if the point & / fx = 5000
A ^x, y h, B ^− 5, 7h and C ^− 4, 5h are collinear. correct, / fx' = 5000 − 100 + 110 = 5010
Ans :
` Correct Mean = 5010 = 50.1
If the area of the triangle formed by the points is zero, 100
then points are collinear. Median will remain same i.e. median = 52 .

26 1^ 2 3h 2^ 3 1h 3^ 1 2h@
1 x y −y +x y −y +x y −y = 0 or
There are 30 cards of the same size in a bag in which
6x ^7 − 5h − 5 ^5 − y h − 4 ^y − 7h@ = 0 the numbers 1 to 30 are written. One card is taken
2x − 25 + 5y − 4y + 28 = 0 out of the bag at random. Find the probability that
2x + y + 3 = 0 the number on the selected card is not divisible by 3.
Ans :
24. One tends to become lazy. Also, starting at your mobile
screen for long hours can affect you eyesight and give Here, Total cards = 30
you headaches. Those who are addicted to playing
Number divisible by 3 = 3, 6, 9, 12, 15, 18, 21,
PUBG can get easily stressed out or face anxiety
issues in public due to lack of social interaction. 24, 27, 30
To raise social awareness about ill effects of playing Total number of favourable outcomes
PUBG, a school decided to start “BAN PUBG:
= 30 − 10 = 20
campaign, students are asked to prepare campaign board
in the shape of rectangle (as shown in the figure). [2] ` required probability = 20 = 2
30 3
26. Pawan is fly fishing in a stream as shown in the figure.
The tip of her fishing rod is 1.8 m above the surface of
the water and the fly at the end of the string rests on
the water 3.6 m away and 2.4 m from a point directly
under the tip of the rod. [2]

(i) Find the area of the board.


(ii) It cost of 1 cm2 of board is `8, then find the cost
of board.
Ans :
(i) From the figure, we have
AB = ^8 − 2h2 + ^2 − 2h2
= 62 + ^0 h2 = 6 cm Assuming that her string (from the tip of her rod to

^8 − 8h2 + ^6 − 2h
2 the fly) is taut, how much string does she have out?
BC =
Ans :
= ^0h2 + 42 = 4 cm
Let the tip of her fishing rod be A. So, its distance
Area of board = Area of rectangle ABCD from surface of water B is AB = 1.8 m
= AB # BC
= 6 # 4 = 24 cm2
(ii) Total cost of board = Area of board × Rate
= 24 # 8 = ` 192
25. The mean and median of 100 observation are 50 and
52 respectively. The value of the largest observation
is 100. It was later found that it is 110. Find the true
mean and median. [2]
Ans : Again, let C be the point at 2.4 m away from B .
Then, length of the string that she has out.
As we know that,
AC = ^AB h2 + ^BC h2
Mean =
/ fx
/f [using Pythagoras theorem]

& 50 =
/ fx = ^1.8h2 + ^2.4h2
100
Download 20 Solved Sample Papers pdfs from www.cbse.online or www.rava.org.in Page 4
Mathematics Standard X Solved Sample Paper 2 www.rava.org.in

= 3.24 + 5.76 x + 20 ^100h = 3000


= 9 = 3 cm x = 1000
Thus x = 1000 and y = 100
Section C Fixed charge and cost of food per day are Rs. 1,000
and Rs. 100.
27. If α and β are the zeroes of the polynomial 6y2 − 7y + 2, 29. Divide 56 in four parts in A.P. such that the ratio
find a quadratic polynomial whose zeroes are α1 and 1β . of the product of their extremes (1st and 4rd ) to the
[3] product of means (2nd and 3rd ) is 5: 6. [3]
Ans : Ans :
We have p ^y h = 6y2 − 7y + 2 Let the four numbers be a − 3d, a − d, a + d, a + 3d
Now a − 3d + a − d + a + d + a + 3d = 56
Sum of zeroes α + β = − b− 7 l = 7
6 6 4a = 56 & a = 14
Hence numbers are 14 − 3d, 14 − d, 14 + d, 14 + 3d
Product of zeroes αβ = 2 = 1
6 3
Now, according to question,
Sum of zeroes of new polynomial g (y) ^14 − 3d h^14 + 3d h
=5
1 +1 = α+β = 7 6 = 7 ^14 − d h^14 + d h 6
α β αβ 2 6 2 196 − 9d2 = 5
and product of zeroes of new polynomial g (y), 196 − d2 6
1 1 = 1 = 1 =3 6 ^196 − 9d2h = 5 ^196 − d2h
α#β αβ 1 3
6 # 196 − 54d2 = 5 # 196 − 5d2
The required polynomial is
^6 − 5h # 196 = 49d
2

g (x) = y2 − 7 y + 3 = 1 62y2 − 7y + 6@ d2 = 196 = 4


2 2 49
or
d =! 2
If α, β and γ are zeroes of the polynomial
Thus numbers are a − 3d = 14 − 3 # 2 = 8
6x3 + 3x2 − 5x + 1, then find the value of α−1 + β−1 + γ−1 .
Ans : a − d = 14 − 2 = 12
a + d = 14 + 2 = 16
We have p (x) = 6x3 + 3x2 − 5x + 1
Since α, β and γ are zeroes polynomial p (x), we have a + 3d = 14 + 3 # 2 = 20
Thus required AP is 8, 12, 16, 20.
α + β + γ =−b =− 3 =− 1
c 6 2
or
αβ + βγ + γα = c = − 5 If the sum of the first n terms of an A.P. is 12 63n2 + 7n@,
a 6
then find its nth term. Hence write its 20th term.
and αβγ = − d = − 1 Ans :
a 6
1 + 1 + 1 = αβ + βγ + γα Let the first term be a , common difference be d , n th
Now term be an and sum of n term be Sn .
α β γ αβγ
−5 6 −5 6 =5 Sum of n term Sn = 1 63n2 + 7n@
= = 2
−1 6 6 # −1
Hence α−1 + β−1 + γ−1 = 5 . Sum of 1 term S1 = 1 83 # ^1 h2 + 7 ^1 hB
2
28. A part of monthly hostel charge is fixed and the
= 1 63 + 7@
remaining depends on the number of days one has 2
taken food in the mess. When Swati takes food for
20 days, she has to pay Rs. 3,000 as hostel charges = 1 # 10 = 5
2
whereas Mansi who takes food for 25 days Rs. 3,500
as hostel charges. Find the fixed charges and the cost Sum of 2 term S2 = 1 83 ^2 h2 + 7 # 2B
2
of food per day. [3]
Ans : = 1 612 + 14@
2
Let fixed charge be x and per day food cost be y
= 1 # 26
x + 20y = 3000 ...(1) 2
x + 25y = 3500 ...(2) = 13
Subtracting (1) from (2), we have Now a1 = S1 = 5
5y = 500 & y = 100 a2 = S2 − S1 = 13 − 5 = 8
Substituting this value of y in equation (1), we get d = a2 − a1 = 8 − 5 = 3

To Get 20 Solved Paper Free PDF by whatsapp add +91 89056 29969 in your class Group Page 5
Mathematics Standard X Solved Sample Paper 2 www.cbse.online

Now, A.P. is 5, 8, 11, ....


nth term, an = a + ^n − 1h d
= 5 + ^n − 1h 3
= 5 + ^20 − 1h^3 h
= 5 + 57
= 62
Hence, a2 = 62
30. TABC is right angled at C. If p is the length of the
perpendicular from C to AB and a, b, c are the lengths
of the sides opposite +A, +B and +C respectively,
then prove that 12 = 12 + 12 . [3]
p a b
Ans :
As per given condition we have drawn the figure
below.
h0
tan 30c = ; h = 300 tan 30c
300 0

= 300 m = 100 3
3
tan 45c = h1 ; h1 = 300 tan 45c = 300 m
300

tan 60c = h2 ; h2 = 300 tan 60c = 300 3


300
(a) h0 = 100 3 m is the height of the observation
post.
In TACB and TCDB (b) h0 + h2 = 100 3 + 300 3 = 400 3 m is the
+ABC = +CDB = 90º height of the tree.
+B = +B (common) (c) h0 + h1 = 100 3 + 300 = 100 ^ 3 + 3h m ft. is
Because of AA similarity, we have the height of the baboons.

TABC ~TCDB 32. In the figure, PQ is a tangent to a circle with center


O . If +OAB = 30º , find +ABP and +AOB. [3]
Now b =c
p a
1 = c
p ab
1 = c2
p2 a2 b2
1 = a2 + b2
p2 a2 b2
1 = 1 +1 Hence Proved
p2 a2 b2
31. From her elevated observation post 300 m away, a
naturalist spots a troop of baboons high up in a tree. Ans :
Using the small transit attached to her telescope, she Here OB is radius and QT is tangent at B , OB = PQ
finds the angle of depression to the bottom of this tree +OBP = 90º
is 30c, while the angle of elevation to the top of the tree Since the tangent is perpendicular to the end point
is 60c. The angle of elevation to the troop of baboons is of radius,
45c. Use this information to find (a) the height of the Here OA and OB are radius of circle and equal. Since
observation post, (b) the height of the baboons’ tree, angles opposite to equal sides are equal,
and (c) the height of the baboons above ground. [3]
+OAB = +OBA = 30º
Ans :
Now +AOB = 180º − ^30º + 30ºh
Let’s first find the distances h0, h1 and h2 in the
diagram below, then answer the question. = 120º
+ABP = +OBP − +OBA
= 90º − 30º = 60º

or

Download 20 Solved Sample Papers pdfs from www.cbse.online or www.rava.org.in Page 6


Mathematics Standard X Solved Sample Paper 2 www.rava.org.in

A circle is inscribed in a TABC, with sides AC, AB


and BC as 8 cm, 10 cm and 12 cm respectively. Find
the length of AD, BE and CF.
Ans :
As per question we draw figure shown below.

34. A boy, 1.4 metre tall standing at the edge of a river


We have AC = 8 cm bank sees the top of a tree on the edge of the other
bank at an elevation of 55c. Standing back by 3 metre,
AB = 10 cm
he sees it at elevation of 45c. [3]
and BC = 12 cm (a) Draw a rough figure showing these facts.
Let AF be x . Since length of tangents from an (b) How wide is the river and how tall is the tree?
external point to a circle are equal, [ sin 55c = 0.8192 . cos 55c = 0.5736 , tan 55c = 1.4281]
Ans :
At A, AF = AD = x (1)
(a) The rough sketch is as follows:
At B BE = BD = AB − AD = 10 − x (2)
At C CE = CF = AC − AF = 8 − x (3)
Now BC = BE + EC
12 = 10 − x + 8 − x
2x = 18 − 12 = 6
or x =3
Now AD = 3 cm,
BE = 10 − 3 = 7 cm
and CF = 8 − 3 = 5
33. Construct a triangle similar to a given equilateral
TPQR with side 5cm such that each of its side is 6
7
of the corresponding sides of TPQR . [3] (b) Here, BF represents the tree, and CD represents
the river.
Ans :
DG is the initial position of the boy and AE is the
Steps of construction : new position.
1. Draw a line segment QR = 5 cm.
Here, AE = DG = BC = 1.4 m
2. With Q as centre and radius PQ = 5 cm, draw an
arc. If DC = x m, then EC = ^x + 3h m
3. With R as center and radius = PR = 5 cm,draw In right TECF ,
another arc meeting the arc drawn in step 2 at the
point P . tan 45c = CF & 1 = CF
EC x+3
4. Join PQ and PR to obtain TPQR . CF = ^x + 3h
5. Below QR , construct an acute +RQX . In right TDCF ,
6. Along QX , mark off seven points Q1, Q2, ....Q7 tan 55c = CF & 1.4281 = x + 3
DC x
such that QQ1 = Q1 Q2 = Q2 Q3 = ..... = Q6 Q7 .
7. Join Q7 R . 1.4281x = x + 3
8. Draw Q6 R' | | Q7 R . 0.4281x = 3
x = 3 =7
9. From, Rl draw RlPl | | RP. 0.4281
Hence, PlQRl is the required triangle. Width of the river = CD = 7 m
Height of the tree = BF + BC + CF
= ^1.4 + 7 + 3h = 11.4 m

To Get 20 Solved Paper Free PDF by whatsapp add +91 89056 29969 in your class Group Page 7
Mathematics Standard X Solved Sample Paper 2 www.cbse.online

Section D 36. Find x in terms of a, b and c :


a + b = 2c , x ! a, b, c
[4]

35. Find HCF of 81 and 237 and express it as a x−a x−b x−c
Ans :
linear combination of 81 and 237 i.e. HCF
^81, 237h = 81x + 237y for some x and y . [4]
We have a + b = 2c
Ans : x−a x−b x−c
By using Euclid’s Division Lemma, we have a ^x − b h^x − c h + b ^x − a h^x − c h = 2c ^x − a h^x − b h
237 = 81 # 2 + 75 ...(1) ax2 − abx − acx + abc + bx2 − bax − bcx + abc
81 = 75 # 1 + 6 ...(2) = 2cx2 − 2cxb − 2cxa + 2abc
75 = 6 # 12 + 3 ...(3) ax2 + bx2 − 2cx2 − abx − acx − bax − bcx + 2cbx + 2acx
6 = 3#2+0 ...(4) =0
Hence,HCF ^81, 237h = 3. x ^a + b − 2c h − 2abx + acx + bcx = 0
2

In order to write 3 in the form of 81x + 237y , x ^a + b − 2c h + x ^− 2ab + ac + bc h = 0


2

3 = 75 − 6×12 Thus x = − b ac + bc − 2ab l


a + b − 2c
= 75 − ^81 − 75×1h ×12 Replace 6 from (2)
37. If P ^− 5, − 3h, Q ^− 4, − 6h, R ^2, − 3h and S ^1, 2h are the
= 75 − 81×12 + 75×12
vertices of a quadrilateral PQRS, find its area. [4]
= 75 + 75×12 − 81×12 Ans :
= 75 ^1 + 12h − 81×12 We have P ^− 5, − 3h , Q ^− 4, − 6h , R ^2, − 3h and S ^1, 2h
= 75×13 − 81×12 Area of quadrilateral
= 13 ^237 − 81×2h − 81×12 Replace 75 from (1) = 1 6^x1 y2 − x2 y1h + ^x2 y3 − x3 y2h + ^x3 y 4 − x 4 y3h
2
= 13×237 − 81×2×13 − 81×12 + ^x 4 y1 − x1 y 4h@
= 237×13 − 81 ^26 + 12h
Area = 1 6− 5 ^− 6h − ^− 4h^− 3h + ^− 4h^− 3h − 2^− 6h
= 237×13 − 81×38 2
= 81× ^− 38h + 237× ^13h + ^2 h^2 h − 1 # ^− 3h + 1 # ^− 3h − ^− 5h^2 h@
= 81x + 237y = 630 − 12 + 12 + 12 + 4 + 3 − 3 + 10@
1
Hence x = − 38 and y = 13 . These values of x and y 2
are not unique. = 1 630 + 12 + 4 + 10@ = 1 656@ = 28 sq. units
2 2
or or
Show that there is no positive integer n , for which If P ^9a − 2, − b h divides the line segment joining
n − 1 + n − 1 is rational. A ^3a + 1, − 3h and B ^8x, 5h in the ratio 3: 1. Find the
Ans : values of a and b.
Ans :
Let us assume that there is a positive integer n for
which n − 1 + n − 1 is rational and equal to qp , Using section formula we have
where p and q are positive integers and ^q ! 0h . 3 ^8a h + 1 + ^3a + 1h
9a − 2 = ...(1)
p 3+1
n−1+ n−1 = ...(1)
q
3 ^5 h + 1 ^− 3h
−b = ...(2)
or,
q
= 1 3+1
p n−1+ n+1
Form (2) − b = 15 − 3 = 3 & b = − 3
= n−1− n+1 4
^ n − 1 + n + 1 h^ n − 1 − n + 1 h
From (1), 9a − 2 = 24a + 3a + 1
= n−1− n+1
4
(n − 1) − (n + 1) 4 ^9a − 2h = 27a + 1
or
q
= n−1− n+1 36a − 8 = 27a + 1
p −2
9a = 9
2q
n+1− n−1 = ...(2) a =1
p
Adding (1) and (2), we get 38. The angle of elevation of the top Q of a vertical tower
p 2q p2 + 2q2 PQ from a point X on the ground is 60c. From a point
2 n+1 = + = ...(3)
q p pq Y 40 m vertically above X , the angle of elevation of
Subtracting (2) from (1) we have the top Q of tower is 45c. Find the height of the PQ
p2 − 2q2 and the distance PX . (Use 3 = 1.73 ) [4]
2 n−1 = ...(4) Ans :
pq
Let PX be x and PQ be h . As per given in question

Download 20 Solved Sample Papers pdfs from www.cbse.online or www.rava.org.in Page 8


Mathematics Standard X Solved Sample Paper 2 www.rava.org.in

we have drawn figure below. Here M is the centre of the line joining their feet.
Let BM = MD = z
In right TABM , we have,
x = tan 30c
z
x = z# 1
3
In right TCDM , we have
y
= tan 60c
z
y = z# 3
From (1) and (2), we get
1
x = z# 3
y z# 3
Now QT = ^h − 40h m x =1
In right TPQX , we have, y 3

tan 60c = h Thus x :y = 1:3


x
39. Prove that opposite sides of a quadrilateral
3 =h circumscribing a circle subtend supplementary angles
x
at the centre of the circle. [4]
h = 3x ...(1) Ans :
In right TQTY we have A circle centre O is inscribed in a quadrilateral ABCD
tan 45c = h − 40
as shown in figure given below.
x

1 = h − 40
x
x = h − 40 ...(2)
Solving (1) and (2), we get
x = 3 x − 40
3 x − x = 40
^ 3 − 1h x = 40
^ h
x = 40 = 20 3 + 1 m
Since OE and OF are radius of circle
3 −1
OE = OF (radii of circle)
Thus x = 3 # 20 ^ 3 + 1h
Tangent drawn at any point of a circle is perpendicular
= 20 ^3 + 3h m to the radius through the point contact.
= 20 ^3 + 1.73h = 20 # 4.73 Thus +OEA = +OFA = 90c
Hence, height of tower is 94.6 m. Now in TAEO and TAFO
or OE = OF
The tops of two towers of height x and y , standing +OEA = +OFA = 90c
on level ground, subtend angles of 30c and 60c OA = OA (Common side)
respectively at the centre of the line joining their feet,
then find x : y . Thus TAEO , TAFO (SAS congruency)
Ans : +7 = +8

Let AB be the tower of height x and CD be the Similarly, +1 = +2


tower of height y . Angle of depressions of both tower +3 = +4
at centre point M are given 30c and 60c respectively. +5 = +6
As per given in question we have drawn figure below.
Since angle around a point is 360c,
+1 + +2 + +3 + +4 + +5 + +6 + +7 + +8 = 360c
2+1 + 2+8 + 2+4 + 2+5 = 360c
+1 + +8 + +4 + +5 = 180c
^+1 + +8h + ^+4 + +5h = 180c
+AOB + +COD = 180c
Hence Proved.
40. On the sports day of a school, 300 students participated.

To Get 20 Solved Paper Free PDF by whatsapp add +91 89056 29969 in your class Group Page 9
Mathematics Standard X Solved Sample Paper 2 www.cbse.online

Their ages are given in the following distribution :


Age (in 5-7 7-9 9-11 11- 13- 15- 17-
years) 13 15 17 19
Number 67 33 41 95 36 13 15
of
students
Find the mean and mode of the data. [4]
Ans :

Here, Modal class = 11 − 13


l = 11, f1 = 95 , f0 = 41, f2 = 36 , h = 2
f1 − f0
Mode = l + h
2f1 − f0 − f2 #

= 11 + 95 − 41 2
190 − 41 − 36 #

= 11 + 54 # 2 ]
113
Mode = 11 + 0.95 = 11.95

Age xi fi fi xi
5-7 6 67 402
7-9 8 33 264
9-11 10 41 410
11-13 12 95 1140
13-15 14 36 504
15-17 16 13 208
17-19 18 15 270
/ fi = 300 / fi xi = 3, 198

Mean = x =
/ fi xi
/ fi
3, 198
=
300
= 10.66
WWW.CBSE.ONLINE

Download unsolved version of this paper from


www.cbse.online

This sample paper has been released by website www.cbse.online for the benefits of the students. This paper has been prepared by
subject expert with the consultation of many other expert and paper is fully based on the exam pattern for 2019-2020. Please note
that website www.cbse.online is not affiliated to Central board of Secondary Education, Delhi in any manner. The aim of website is
to provide free study material to the students.
Download 20 Solved Sample Papers pdfs from www.cbse.online or www.rava.org.in Page 10
Mathematics Standard X Solved Sample Paper 3 www.rava.org.in

CLASS X (2019-20)
MATHEMATICS STANDARD(041)
SAMPLE PAPER-3

Time : 3 Hours Maximum Marks : 80


General Instructions :
(i) All questions are compulsory.
(ii) The questions paper consists of 40 questions divided into 4 sections A, B, C and D.
(iii) Section A comprises of 20 questions of 1 mark each. Section B comprises of 6 questions of 2 marks each. Section
C comprises of 8 questions of 3 marks each. Section D comprises of 6 questions of 4 marks each.
(iv) There is no overall choice. However, an internal choices have been provided in two questions of 1 mark each, two
questions of 2 marks each, three questions of 3 marks each, and three questions of 4 marks each. You have to
attempt only one of the alternatives in all such questions.
(v) Use of calculators is not permitted.

Ans : (d) 2 # k #− 2
Section A We have, x2 + kx + 1 = 0
Q.1-Q.10 are multiple choice questions. Select the On comparing with ax2 + bx + c = 0 ,
most appropriate answer from the given options. we get a = 1, b = k and c = 1
1. (i) The L.C.M. of x and 18 is 36. For linear factors, D $0
(ii) The H.C.F. of x and 18 is 2.
What is the number x ? [1] b − 4ac $ 0
2

(a) 1 (b) 2 k −4#1#1 $ 0


2

(c) 3 (d) 4 ^k2 − 22h $ 0


Ans : (d) 4 ^k − 2h^k + 2h $ 0
L.C.M. # H.C.F. = First number # second number k $ 2 and k #− 2
Hence, required number = 36 # 2 = 4 4. An AP starts with a positive fraction and every
18
alternate term is an integer. If the sum of the first 11
2. In a number of two digits, unit’s digit is twice the tens
terms is 33, then the fourth term is [1]
digit. If 36 be added to the number, the digits are
(a) 2 (b) 3
reversed. The number is [1]
(a) 36 (b) 63 (c) 5 (d) 6
(c) 48 (d) 84 Ans : (a) 2
Ans : (c) 48 Given, S11 = 33
Let unit’s digit : x
26 @
11 2a + 10d = 33 & a + 5d = 3
tens digit : y
i.e., a6 = 3 & a 4 = 2
Then, x = 2y
[Since, Alternate terms are integers and the given sum
Number = 10y + x is possible]
According to the question.
5. Which of the following statement is false? [1]
10y + x + 36 = 10x + y (a) All isosceles triangles are similar.
9x − 9y = 36 (b) All quadrilateral triangles are similar.
or x−y = 4 ....(1) (c) All circles are similar.
Solve, x = 2y (d) None of the above
2y − y = 4 Ans : (a) All isosceles triangles are similar.
y =4 An isosceles triangle is a triangle with two side of
Now, from equation, equal length hence statement given in option (a) is
false.
x−4 = 4 & x = 8
6. C is the mid-point of PQ , if P is (4, x), C is (y, − 1)
Number = 10 # 4 + 8 = 40 + 8 = 48
and Q is (− 2, 4), then x and y respectively are [1]
x−y = 4 (a) − 6 and 1 (b) − 6 and 2
3. The linear factors of the quadratic equation (c) 6 and − 1 (d) 6 and − 2
x2 + kx + 1 = 0 are [1] Ans : (a) − 6 and 1
(a) k $ 2 (b) k # 2
(c) k $− 2 (d) 2 # k #− 2 Since, C (y, − 1) is the mid-point of P (4, x) and

To Get 20 Solved Paper Free PDF by whatsapp add +91 89056 29969 in your class Group Page 1
Mathematics Standard X Solved Sample Paper 3 www.cbse.online

Q (− 2, 4). The highest power of a variable in a polynomial is


4−2 = y called its ..........
We have, ...(1)
2 Ans : Degree
and 4 + x =− 1 ...(2)
2 13. Area of a circle is .......... [1]
From equation (1) and (2), we get Ans : πr2
y =1 14. The volume and surface area of a sphere are numerically
and x =− 6 equal, then the radius of sphere is .......... units. [1]
7. If tan 2A = cot ^A − 18ch , where 2A is an acute angle, Ans : 3
then the value of A is [1] 15. Someone is asked to make a number from 1 to 100.
(a) 12c (b) 18c The probability that it is a prime is .......... [1]
(c) 36c (d) 48c 1
Ans :
Ans : (c) 36c 4
(Q.16-Q.20) Answer the following
Given, tan 2A = cot ^A − 18ch
16. Find the value (s) of k if the quadratic equation
cot ^90c − 2Ah = cot ^A − 18ch
3x2 − k 3 x + 4 = 0 has real roots. [1]
90c − 2A = A − 18c Ans :
[since, ^90c − 2Ah and ^A − 18ch If discriminant of quadratic equation is equal to zero,
both are acute angles] or more than zero, then roots are real.
90c + 18c = A + 2A We have 3x2 − k 3 x + 4 = 0
3A = 108c Compare with ax2 + bx + c = 0
A = 108c = 36c D = b2 − 4ac
3
8. An equation of the circle with centre at (0, 0) and For real roots b2 − 4ac $ 0
radius r is [1] ^− k 3 h − 4 # 3 # 4 $ 0
2

(a) x2 + y2 = r2 (b) x2 − y2 = r2
3k2 − 48 $ 0
(c) x − y = r (d) x2 + r2 = y2
k2 − 16 $ 0
Ans : (a) x2 + y2 = r2
^k − 4h^k + 4h $ 0
Here, h = k = 0 . Therefore, the equation of the circle Thus k #− 4 and k $ 4
is x2 + y2 = r2 . 17. A chord of a circle of radius 10 cm subtends a right
9. The ratio of the sides of the triangle to be constructed angle at the centre. Find area of minor segment.
with the corresponding sides of the given triangle is ^Use π = 3.14h [1]
known as [1] Ans :
(a) scale factors (b) length factor
Radius of circle r = 10 cm, central angle = 90c
(c) side factor (d) K -factor Area of minor segment
Ans : (a) scale factors
= 1 # 102 # : 3.14 # 90 − sin 90cD
The ratio of the sides of the triangle to be constructed 2 180
with the corresponding sides of the given triangle is
= 1 # 100 # 61.57 − 1@ = 28.5 cm2
known as scale factor. 2
10. Ratio of volumes of two cylinders with equal height 18. Volume and surface area of a solid hemisphere are
is [1] numerically equal. What is the diameter of hemisphere
(a) H : h (b) R : r ? [1]
(c) R2 : r2 (d) None of these Ans :
Ans : (c) R2 : r2 Let radius of sphere be r .
πR 2 h : π r 2 h = R 2 : r 2 Given, volume of sphere = S.A. of hemisphere
2 π r 3 = 3π r 2
(Q.11-Q.15) Fill in the blanks. 3
11. If p is a prime number and it divides a2 then it also
divides .........., where a is a positive integer. [1] r = 9 units
2
Ans : a
Diameter d = 9 # 2 = 9 units
2
12. .......... equation is valid for all values of its variables.
[1] or
Ans : Identity Find the number of solid sphere of diameter 6 cm can
be made by melting a solid metallic cylinder of height
or 45 cm and diameter 4 cm.

Download 20 Solved Sample Papers pdfs from www.cbse.online or www.rava.org.in Page 2


Mathematics Standard X Solved Sample Paper 3 www.rava.org.in

Ans : 100 = AB2


196 ^7 h2
Let the number of sphere = n
100 = AB2
Radius of sphere = 3 cm, 196 49
radius of cylinder = 2 cm
AB2 = 49 # 100
Volume of spheres = Volume of cylinder 196

n # 4 πr3 = πr12 h AB2 = 25


3
AB = 5 cm
n # 4 # 22 # ^3 h3 = 22 # ^2 h2 # 45 23. If A ^5, 2h , B ^2, − 2h and C ^− 2, t h are the vertices of
3 7 7
a right angled triangle with +B = 90º , then find the
36n = 180 value of t. [2]
n = 180 = 5 Ans :
36
As per question, triangle is shown below.
Number of solid sphere = 5 .
19. What is abscissa of the point of intersection of the
‘‘Less than type’’ and of the ‘‘More than type’’
cumulative frequency curve of a grouped data ? [1]
Ans :
The abscissa of the point of intersection of the
‘‘Less than type’’ and ‘‘More than type’’ cumulative
frequency curve of a grouped data is median.
20. A dice is thrown once. Find the probability of getting
a prime number. [1]
Ans :

Total outcomes = 6
Prime numbers = 2, 3, 5 = 3
Now AB2 = ^2 − 5h2 + ^− 2 − 2h2 = 9 + 16 = 25
P (prime no.) = 3 = 1 BC2 = ^− 2 − 2h2 + ^t + 2h2 = 16 + ^t + 2h2
6 2
AC2 = ^5 + 2h2 + ^2 − t h2 = 49 + ^2 + t2h
Section B Since TABC is a right angled triangle
21. Solve the following system of linear equations by AC2 = AB2 + BC2
substitution method: [2]
49 + ^2 − t h2 = 25 + 16 + ^t + 2h2
2x − y = 2
49 + 4 − 4t + t2 = 41 + t2 + 4t + 4
x + 3y = 15
53 − 4t = 45 + 4t
Ans :
8t = 8
We have 2x − y = 2 ...(1)
t =1
x + 3y = 15 ...(2)
or
From equation (1), we get y = 2x − 2 ...(3)
For what values of k are the points ^8, 1h, ^3, − 2k h
Substituting the value of y in equation (2),
and ^k, − 5h collinear?
x + 6x − 6 = 15 Ans :
or, 7x = 21
Since points ^8, 1h, ^3, − 2k h and ^k, − 5h are collinear,
x =3 area of triangle formed must be zero.
Substituting this value of x in (3), we get
28 ^ h ^ h ^ hB
1 8 − 2k + 5 + 3 − 5, − 1 + k 1 + 2k = 0
From equation (1), we have
y = 2#3−2 = 4 2k2 − 15k + 22 = 0

x = 3 and y = 4 k = 2, 11
2
22. Let TABC ~TDEF. if ar ^TABC h = 100 cm2,
24. A book seller has 420 science stream books and 130
ar ^DEF h = 196 cm2 and DE = 7, then find AB. [2]
Arts stream books. He wants to stack them in such a
Ans : way that each stack has the same number and they
We have TABC ~TDEF , thus take up the least area of the surface. [2]
ar ^TABC h 2
= AB2
are ^TDEF h DE
To Get 20 Solved Paper Free PDF by whatsapp add +91 89056 29969 in your class Group Page 3
Mathematics Standard X Solved Sample Paper 3 www.cbse.online

or
A bag contains cards bearing numbers from 11 to 30.
A card is taken out from the bag at random. Find
the probability that the selected card has multiple of
5 on it.
Ans :

Here, Number of cards = 20


Multiples of 5 from 11 to 30 are 15, 20, 25, 30
Number of favourable outcomes = 4
Required probability = 4 = 1
20 5
26. Rajesh starts walking from his house to office. Instead
of going to the office directly, he goes to a mall first,
(i) What is the maximum number of books that can from there to his wife’s office and then reaches the
be placed in each stack for this purpose? office. What is the extra distance travelled by Rajesh
(ii) Which mathematical concept is used to solve the in reaching his office? Assume that all distance
problems? covered are in straight lines, if the house is situated at
(2,4), mall at (5,8), wife’s office at (13,14) and office
Ans :
at (13,26) and coordinates are in kilometre. [2]
(i) Given number of science books = 420 and number Ans :
of Arts books = 130
From questions,
420 = 2 # 2 # 3 # 5 # 7

Extra distance travelled by Vicky


130 = 2 # 5 # 13
= ^AB + BC + CD h − AD
^5 − 2h2 + ^8 − 4h2 + ^13 − 5h + ^14 − 8h2
2
=
+ ^13 − 13h2 + ]26 − 14g2 − ^13 − 2h2 + ^26 − 4h2
= 9 + 16 + 64 + 36 + 0 + 14 − 121 + 484

Maximum number of books that can be placed in each = 5 + 10 + 12 − 24.6 = 2.4 km


stack for the given purpose
= HCF ^420, 130h Section C
= 2 # 5 = 10
1 1
27. Find the zeroes of the quadratic polynomial x2 − 2 2 x
(ii) Prime factorisation method.
and verify the relationship between the zeroes and the
25. Write the relationship connecting three measures of coefficients. [3]
central tendencies. Hence find the median of the give Ans :
data if mode is 24.5 and mean is 29.75. [2]
Ans : We have x2 − 2 2 x = 0
x ^x − 2 2 h = 0
Given, Modal = 24.5
Thus zeroes are 0 and 2 2 .
and Mean = 29.75
The relationship connecting measures of central Sum of zeroes 2 2 = − Coefficient of x2
tendencies is : Coefficient of x
3 Median = Mode + 2 Mean and product of zeroes 0 = Cons tan term
Coefficient of x2
3 Median = 24.5 + 2 # 29.75
Hence verified
= 24.5 + 59.50
3 Median = 84.0 or
What should be added to x3 + 5x2 + 7x + 3 so that it
` Median = 84 = 28
3 is completely divisible by x2 + 2x .

Download 20 Solved Sample Papers pdfs from www.cbse.online or www.rava.org.in Page 4


Mathematics Standard X Solved Sample Paper 3 www.rava.org.in

Ans : Sn = n 82a + ^n − 1h dB
2
x+3
g
x 2 + 2x x 3 + 5 x 2 + 7 x + 3 Now S 4 = 2 62a + 3d @ = 24
x + 2x
3 2 2a + 3d = 12 ...(2)
Multiplying (1) by 2 and subtracting (2) from it we
3x + 7x + 3
2
get
3x2 + 6x
^2a + 22d h − ^2a + 3d h = − 26 − 12
x+3
19d = − 38

28. Solve for x and y : [3] d =− 2


x + 2y = − 1 Substituting the value of d in (1) we get
2 3 a + 11 # − 2 = − 13
y
x− = 3 a = − 13 + 22
3
Ans : a =9

x + 2y = − 1 Now, Sn = 82a + ^n − 1h dB
n
We have 2
2 3
or 3x + 4y = − 6 ...(1) S10 = 10 ^2 # 9 + 9 # − 2h
2
and x −y =3 = 5 # ^18 − 18h = 0
1 3
Hence, S10 = 0
or 3x + y = 9 ...(2)
30. ABC is a triangle, PQ is the line segment intersecting
Subtracting equation (2) from equation (1), we have AB in P and AC in Q such that PQ | | BC and divides
5y = − 15 & y = − 1 TABC into two parts, equal in area, find BP: AB, [3]
Substituting y = − 3 in eq (1), we get Ans :
3x + 4 ^− 3h = − 6 As per given condition we have drawn the figure
3x − 12 = − 6 below.
3x = 12 − 6
Thus x =2
Hence x = − 2 and y = − 3.
29. For what value of n, are the nth terms of two A.Ps 63,
65, 67, ... and 3, 10, 17, .... equal? [3]
Ans :
Let a, d and A, D be the 1st term and common
difference of the 2 APs respectively.
n is same
For 1st AP, a = 63 , d = 2
For 2nd AP, A = 3, D = 7 Here, Since PQ | | BC and PQ divides TABC into
Since n th term is same, two equal parts, thus TAPQ~TABC
an = An ar ^TAPQ h 2
Now = AP 2
ar ^TABC h AB
a + ^n − 1h d = A + ^n − 1h D
1 = AP2
63 + ^n − 1h 2 = 3 + ^n − 1h 7 2 AB2
63 + 2n − 2 = 3 + 7n − 7 1 = AP
61 + 2n = 7n − 4 2 AB
65 = 5n & n = 13 1 = AB − BP
AB ^AB = AP + BP h
When n is 13, the n terms are equal i.e., a13 = A13
th 2
1 = 1 − BP
or 2 AB
In an A.P., if the 12 term is − 13 and the sum of
th
BP = 1 − 1 = 2 −1
its first four terms is 24, find the sum of its first ten AB 2 2
terms. BP : AB = ^ 2 − 1h : 2
Ans :
31. The tallest free-standing tower in the world is the
Let the first term be a , common difference be d , n th CN Tower in Toronto, Canada. The tower includes
term be an and sum of n term be Sn . a rotating restaurant high above the ground. From
a12 = a + 11d = − 13 ...(1) a distance of 500 m the angle of elevation to the

To Get 20 Solved Paper Free PDF by whatsapp add +91 89056 29969 in your class Group Page 5
Mathematics Standard X Solved Sample Paper 3 www.cbse.online

pinnacle of the tower is 60c. The angle of elevation = 2 # 42 = 32


to the restaurant from the same vantage point is 45c. Hence AB = 32 = 4 2 cm
How tall is the CN Tower? How far below the pinnacle
of the tower is the restaurant located? [3] or
Two tangents TP and TQ are drawn to a circle with
centre O from an external point T . Prove that
+PTO = +OPQ
Ans :
As per question we draw figure shown below.

Ans :
Let ht be the height of the tower and hr be the height
of the restaurant.
tan 60c = ht ; ht = 500 tan 60c
500
Let +TPQ be θ . the tangent is perpendicular to the
ht = 500 3 = 866.025 m end point of radius,
tan 45c = hr ; hr = 500 tan 45c +TPO = 90º
500
Now +TPQ = +TPO − θ = ^90º − θh
= 500 m Since, TP = TQ and opposite angels of equal sides
Difference, 866.025 − 500 = 366.025 m are always equal, we have
32. In the given figure, PA and PB are tangents to a +TQP = ^90º − θh
circle from an external point P such that PA = 4cm Now in ΔTPQ we have
and +BAC = 135º . Find the length of chord AB . [3]
+TPQ + +TQP + +PTQ = 180º
90º − θ + 90º − θ + +PTQ = 180º
+PTQ = 180º − 180º + 2θ = 2θ
Hence +PTQ = 2+OPQ .
33. Construct an isosceles triangle whose base is 7.5 cm
and altitude 3.5 cm then another triangle whose sides
are 74 times the corresponding sides of the isosceles
triangle. [3]
Ans :
Steps of construction :
1. Draw a line BC = 7.5 cm.
Ans : 2. Draw a perpendicular bisector of BC which
intersects the line BC at O .
Since length of tangents from an external point to a
3. Cut the line OA = 3.5 cm.
circle are equal,
PA = PB = 4 cm
Here +PAB and +BAC are supplementary angles,
+PAB = 180c − 135c = 45c
Angle +ABP and = +PAB = 45c opposite angles of
equal sides, thus
+ABP = +PAB = 45c
In triangle TAPB , we have
+APB = 180c − +ABP − +BAP
= 180c − 45c − 45c = 90c
Thus TAPB is a isosceles right angled triangle
Now AB2 = AP2 + BP2 = 2AP2

Download 20 Solved Sample Papers pdfs from www.cbse.online or www.rava.org.in Page 6


Mathematics Standard X Solved Sample Paper 3 www.rava.org.in

4. Join A to B and A to C . Case 1 : a = 4q


5. Draw a ray BX making an acute angle with BC . a2 = ^4q h2 = 16q2 = 4 ^4q2h
6. Locate 7 points at equal distance among B1, B2, ......B7
on line segment BX . = 4m where m = 4q2
7. Join B7 C . Draw a parallel line through B 4 to Case 2 : a = 4q + 1
B7 C intersecting line segment BC at C' . a2 = ^4q + 1h2 = 16q2 + 8q + 1
8. Through C' draw a line parallel to AC intersecting
line segment AB at A' . = 4 ^4q2 + 2q h + 1
9. Hence, TA'BC' is a required triangle. = 4m + 1 where m = 4q2 + 2q
34. A boy, standing on the top of a tower 20 meter height, Case 3 : a = 4q + 2
saw the top of a building at an elevation of 50c and a2 = ^4q + 2h2 = 16q2 + 16q + 4
its base at a depression of 30c [3]
(a) Draw a rough figure according to the given data. = 4 ^4q2 + 4q + 1h
(b) Find the distance between the tower and the = 4m where m = 4q2 + 4q + 1
building. Case 4 : a2 = ^4q + 3h2 = 16q2 + 24q + 9
(c) Find the distance from the top of the tower to the
base of the building. = 16q2 + 24q + 8 + 1
[use sin 50c = 0.77 , cos 50c = 0.64 , tan 50c = 1.2 , = 4 ^4q2 + 6q + 2h + 1
3 = 1.7 ] = 4m + 1 where m = 4q2 + 6q + 2
From cases 1, 2, 3 and 4 we conclude that the square
Ans :
of any +ve integer is of the form 4m or 4m + 1.
(a)
or
Express the HCF/LCM of 48 and 18 as a linear
combination.
Ans :
Using Euclid’s Division Lemma, we have
48 = 18×2 + 12 (1)
18 = 12×1 + 6 (2)
12 = 6×2 + 0
Thus HCF ^18, 48h = 6
Now 6 = 18 − 12×1 From (2)
6 = 18 − (48 − 18×2) From (1)
6 = 18 − 48×1 + 18×2
6 = 18 # (2 + 1) − 48 # 1 = 18×3 − 48×1
6 = 18×3 + 48× ^− 1h
(b) In TEAB tan 60c = AB
AE Thus 6 = 18x + 48y , where x = 3, y =− 1
Difference between tower and building, Here x and y are not unique.
AB = AE tan 60c 6 = 18×3 + 48× ^− 1h
= 20 # 3m = 18 # 3 + 48 # ^− 1h + 18 # 48 − 18 # 48
(c) In TEAB ^EB h2 = ^AE h2 + ^AB h2 = 18 (3 + 48) + 48 ^− 1 − 18h
^EB h = ^20h + ^20 3 h = 18×51 + 48× ^− 19h
2 2 2

^EB h2 = 400 + 1200 6 = 18x + 48y , where x = 51, y = − 19


EB = 1600 = 40 m 36. The denominator of a fraction is two more than its
numerator. If the sum of the fraction and its reciprocal

Section D is 15 , find the fraction. [4]


34

Ans :
35. Show that the square of any positive integer is of the Let numerator be x , then denominator will be x + 2 .
forms 4m or 4m + 1, where m is any integer. [4]
and fraction = x
Ans : x+2
Let a be any positive integer, then by Euclid’s division Now x + x + 2 = 34
algorithm a can be written as x+2 x 15
a = bq + r 15 ^x2 + x2 + 4x + 4h = 34 ^x2 + 2x h
Take b = 4 , then 0 # r < 4 because 0 # r < b, 30x2 + 60x + 60 = 34x2 + 68x
Thus a = 4q, 4q + 1, 4q + 2, 4q + 3 4x2 + 8x − 60 = 0

To Get 20 Solved Paper Free PDF by whatsapp add +91 89056 29969 in your class Group Page 7
Mathematics Standard X Solved Sample Paper 3 www.cbse.online

x2 + 2x − 15 = 0 reflection in the lake is 60c. Find the height of the


x + 5x − 3x − 15 = 0
2 cloud. [4]
Ans :
x ^x + 5h − 3 ^x + 5h = 0
As per given in question we have drawn figure below.
^x + 5h^x − 3h = 0
We reject the x = − 5 . Thus x = 3 and fraction = 3
5
37. Find the values of k so that the area of the triangle
with vertices ^k + 1, 1h, ^4, − 3h and ^7, − k h is 6 sq.
units. [4]
Ans :
We have ^k + 1, 1h, ^4, − 3h and ^7, − k h
Area of triangle
Δ = 1 6x1 ^y2 − y3h + x2 ^y3 − y1h + x3 ^y1 − y2h@
2

6 = 1 8^k + 1h^− 3 + k h + 4 ^− k − 1h + 7 ^1 + 3hB


2
12 = 6k2 − 2k − 3 − 4k − 4 + 28@
12 = k2 − 6k + 21
k 2 − 6k + 9 = 0 Here, A is cloud and Al is refection of cloud.
k − 3k − 3k + 9 = 0
2 In right TAOP we have
k ^k − 3h − 3 ^k − 3h = 0 tan 30c = H − 120
OP
^k − 3h^k − 3h = 0 1 = H − 120
k = 3, 3 3 OP
or OP = ^H − 120h 3 ...(1)
The base QR of an equilateral triangle PQR lies on In right TOPA' we have
x-axis. The co-ordinates of point Q are ^− 4, 0h and
the origin is the mid-point of the base. find the co- tan 60c = H + 120
OP
ordinates of the point P and R.
Ans : OP = H + 120 ...(2)
3
As per question, line diagram is shown below. From (1) and (2), we get
H + 120 = 3 H − 120
^ h
3
Thus height of cloud is 240 m.
or
The angle of depression of two ships from an aeroplane
flying at the height of 7500 m are 30c and 45c. if both
the ships are in the same that one ship is exactly
behind the other, find the distance between the ships.
Ans :
Let A, C and D be the position of aeroplane and two
ship respectively. Aeroplane is flying at 7500 m height
from point B . As per given in question we have drawn
figure below.
Co-ordinates of point R is ^4, 0h
Thus QR = 8 units
Let the co-ordinates of point P be ^0, y h
Since PQ = QR
^− 4 − 0h2 + ^0 − y h2 = 64
16 + y2 = 64
y =! 4 3
Coordinates of P are `0, 4 3 j or `0, − 4 3 j
38. The angle of elevation of a cloud from a point 120 m
above a lake is 30c and the angle of depression of its

Download 20 Solved Sample Papers pdfs from www.cbse.online or www.rava.org.in Page 8


Mathematics Standard X Solved Sample Paper 3 www.rava.org.in

In right TABC , we have TP = 8


AB = tan 45c 10 6
BC
TP = 80 = 40
7500 = y 6 3
y Hence length of TP is 403 cm.
y = 7500 ...(1) 40. Monthly expenditures on milk in 100 families of a
housing society are given in the following frequency
In right TABD , we have
distribution : [4]
AB = tan 30c
BD Monthly 0 - 175- 350- 525- 7 0 0 - 8 7 5 - 1050-
expendi- 175 350 525 700 875 1050 1125
7500 = 1 ture (in
x+y 3 Rs.)
x + y = 7500 3 ...(2) Number 10 14 15 21 28 7 5
Substituting the value of y from (1) in (2) we have of
families
x + 7500 = 7500 3 Find the mode and median for the distribution.
x = 7500 3 − 7500 Ans :
= 7500 ^ 3 − 1h
C.I. f c.f.
= 7500 ^1.73 − 1h
0-175 10 10
= 7500 # 0.73
157-350 14 24
= 5475 m
350-525 15 39
Hence, the distance between two ships is 5475 m.
525-700 21 60
39. In figure, PQ , is a chord of length 16 cm, of a circle of
radius 10 cm. the tangents at P and Q intersect at a 700-875 28 88
point T . Find the length of TP . [4] 875-1050 7 95
1050-1225 5 100
Median = N th term
2

= 100 = 50 th term
2
` Median class = 525 − 700
N − c.f.
Median = l + 2 #h
f

= 525 + 50 − 39 # 175
21

= 525 + 11 # 175
21
= 525 + 91.6
Ans :
= 616.6
Here PQ is chord of circle and OM will be and Modal class = 700 − 875
perpendicular on it and it bisect PQ . Thus TOMP is
f −f
a right angled triangle. Mode = l + c 1 0 m
2f1 − f0 − f2
We have OP = 10 cm (Radius) l = 700 , f0 = 21, f1 = 28
PM = 8 cm (PQ = 16 cm) f2 = 7 , h = 175
Now in TOMP ,OM = 10 − 8
2 2
28 − 21
= 700 + b
2 # 28 − 21 − 7 l #
175
= 100 − 64 = 36
= 6 cm = 700 + 7 # 175
28
Now +TPM + +MPO = 90c
= 700 + 43.75
Also, +TPM + +PTM = 90c
= 743.75
+MPO = +PTM
WWW.CBSE.ONLINE
+TMP = +OMP = 90c
TTMP + TPMO ^AAh Download unsolved version of this paper from
TP = MP www.cbse.online
or,
PO MO

To Get 20 Solved Paper Free PDF by whatsapp add +91 89056 29969 in your class Group Page 9
Mathematics Standard X Solved Sample Paper 4 www.rava.org.in

CLASS X (2019-20)
MATHEMATICS STANDARD(041)
SAMPLE PAPER-4

Time : 3 Hours Maximum Marks : 80


General Instructions :
(i) All questions are compulsory.
(ii) The questions paper consists of 40 questions divided into 4 sections A, B, C and D.
(iii) Section A comprises of 20 questions of 1 mark each. Section B comprises of 6 questions of 2 marks each. Section
C comprises of 8 questions of 3 marks each. Section D comprises of 6 questions of 4 marks each.
(iv) There is no overall choice. However, an internal choices have been provided in two questions of 1 mark each, two
questions of 2 marks each, three questions of 3 marks each, and three questions of 4 marks each. You have to
attempt only one of the alternatives in all such questions.
(v) Use of calculators is not permitted.

Section A
a
Percentage increment = 2
3a # 100
2

1. If n is an even natural number, then the largest = 100 = 33 1 %


natural number by which n ^n + 1h^n + 2h is divisible, 3 2
is [1] 4. If the equation ^m2 + n2h x2 − 2 ^mp + nq h x + p2 + q2 = 0
(a) 6 (b) 8 has equal roots, then [1]
(c) 12 (d) 24 (a) mp = nq (b) mq = np
Ans : (d) 24 (c) mn = pq (d) mq = np
Since n is divisible by 2 therefore ^n + 2h is divisible Ans : (b) mq = np
by 4, and hence n ^n + 2h is divisible by 8.
Also, n, n + 1, n + 2 are three consecutive numbers. b2 = 4ac
So, one of them is divisible by 3.
4 ^mp + nq h2 = 4 ^m2 + n2h ^p2 + q2h
Hence, n ^n + 1h^n + 2h must be divisible by 24.
m2 q2 + n2 p2 − 2mnpq = 0
2. The value of x, for which the polynomials x2 − 1 and
x2 − 2x + 1 vanish simultaneously, is [1] ^mq − np h2 = 0
(a) 2 (b) − 2 mq − np = 0
(c) − 1 (d) 1 mq = np
Ans : (d) 1 5. If the common difference of an AP is 5, then what is
The expressions ^x − 1h ^x + 1h and ^x − 1h ^x − 1h a18 − a13 ? [1]
which vanish if x = 1 (a) 5 (b) 20
3. X ’s salary is half that of Y ’s. If X got a 50% rise in (c) 25 (d) 30
his salary and Y got 25% rise in his salary, then the Ans : (c) 25
percentage increase in combined salaries of both is [1]
Given, the common difference of AP i.e., d = 5
(a) 30 (b) 33 1
3 Now, a18 − a13 = a + ^18 − 1h d − 6a + ^13 − 1h d @
(c) 37 1
6Since, an = a + ^n − 1h d @
(d) 75
2
Ans : (b) 33 1 = a + 17 # 5 − a − 12 # 5
3
Let yls initial salary = a = 85 − 60 = 25
6. If x = p sec θ and y = q tan θ , then [1]
Xls initial salary = a
2
(a) x − y = p q
2 2 2 2
(b) x q − y p = pq
2 2 2 2
Total salary = a + a = 3a
2 2
After increases, (c) x2 q2 − y2 p2 = 21 2 (d) x2 q2 − y2 p2 = p2 q2
pq
Xls salary = a + c 50 # a m = a + a = 3a Ans : (d) x2 q2 − y2 p2 = p2 q2
2 100 2 2 4 4

Y ls salary = a + c 25 # 4 m = a + a = 5a We know, sec2 θ − tan2 θ = 1


2 100 4 4
and sec θ = x
Total salary of both = 3a + 5a = 8a = 2a p
4 4 4
y
tan θ =
Increment in salary = 2a − 3a = 4a − 3a = a q
2 2 2

To Get 20 Solved Paper Free PDF by whatsapp add +91 89056 29969 in your class Group Page 1
Mathematics Standard X Solved Sample Paper 4 www.cbse.online

x2 q2 − y2 p2 = p2 q2 Ans : (3, 5)
7. The area of a circular ring formed by two concentric 13. In a right triangle ABC , right angled at B , if
circles whose radii are 5.7 cm and 4.3 cm respectively tan A = 1, sin A cos A = .......... [1]
is (Take π = 3.1416 ) [1]
Ans : 1
(a) 43.98 sq. cm. (b) 53.67 sq. cm. 2
(c) 47.24 sq. cm. (d) 38.54 sq. cm. 14. If the area of a circle is 154 cm2 , then its circumference
Ans : (a) 43.98 sq. cm. is ......... [1]
Let the radii of the outer and inner circles be r1 and r2 Ans : 44 cm
respectively, we have
15. If the volume of a cube is 64 cm3 , then its surface area
Area = π r − π r = π ^r − r h
1
2
2
2
1
2
2
2
is .......... [1]
= π ^r1 − r2h ^r1 + r2h Ans : 96 cm 2

= π ^5.7 − 4.3h ^5.7 + 4.3h


(Q.16-Q.20) Answer the following
= π # 1.4 # 10 sq. cm
16. In TABC, DE | | BC, find the value of x . [1]
= 3.1416 # 14 sq. cm.
= 43.98 Sq. cm.
8. The base radii of a cone and a cylinder are equal.
If their curved surface areas are also equal, then the
ratio of the slant height of the cone to the height of
the cylinder is [1]
(a) 2 : 1 (b) 1 : 2
(c) 1 : 3 (d) 3 : 1
Ans : (a) 2 : 1

πrl = 2πrh
Ans :
l =2
h 1 In the given figure DE || BC , thus
9. For finding the popular size of ready-made garments,
which central tendency is used? [1] AD = AE
(a) Mean DB EC
(b) Median x = x+3
(c) Mode x+1 x+5
(d) Both Mean and Mode x2 + 5x = x2 + 4x + 3
Ans : (c) Mode x =3
For finding the popular size of ready made garments,
mode is the best measure of central tendency. 17. If PQ and PR are two tangents to a circle with center
O. If +QPR = 46º then find +QOR . [1]
10. Out of one digit prime numbers, one number is
selected at random. The probability of selecting an
even number is [1]
(a) 1 (b) 1
2 4
(c) 4 (d) 2
9 5
Ans : (b) 1
4
One digit prime numbers are 2, 3, 5, 7. Out of these
numbers, only the number 2 is even. Ans :
(Q.11-Q.15) Fill in the blanks. We have +QPR = 46º
11. L.C.M. of 96 and 404 is .......... [1] Since +QOR and +QPR are supplementary angles
Ans : 9696
+QOR + +QPR = 180º
12. (1, 2), (4, y), (x, 6) and (3, 5) are the vertices of a +QOR + 46º = 180º
parallelogram taken in order, then the value of x and
+QOR = 180º − 46º = 134º
y are .......... [1]
Ans : (6, 3) 18. To divide a line segment AB in the ratio 5: 7, first
AX is drawn, so that +BAX is an acute angle and
then at equal distance, points are marked on the ray
or
AX , find the minimum number of these points. [1]
If x − y = 2 then point ^x, y h is equidistant from (7,1) Ans :
adn (...........)
Download 20 Solved Sample Papers pdfs from www.cbse.online or www.rava.org.in Page 2
Mathematics Standard X Solved Sample Paper 4 www.rava.org.in

Minimum number of points marked on Thus c1 = b1 = c1


c2 b2 c2
AX = 5 + 7 = 12 Hence, system is consistent and dependent.
22. Find the ratio in which the point ^− 3, k h divides the
line segment joining the points ^− 5, − 4h and ^− 2, 3h .
Also find the value of k . [2]
Ans :
As per question, line diagram is shown below.

or
To divide a line segment AB in the ratio 2: 5, a ray Let AB be divides by P in ratio n: 1.
AX is drawn such that +BAX is acute. Then points x co-ordinate for section formula
are marked at equal intervals on AX . What is the ^− 2h n + 1 ^− 5h
minimum number of these points ? −3 =
n+1
Ans :
− 3 ^n + 1h = − 2n − 5
Minimum number of points marked on AX = 2 + 5 = 7
− 3n − 3 = − 2n − 5
19. In figure, a tower AB is 20 m high and BC, its shadow
5 − 3 = 3n − 2n
on the ground, is 20 3 m long. find the Sun’s
altitude. [1] 2 =n
Ratio n = 2 or 2: 1
1 1
Now, y co-ordinate,
2 ^3 h + 1 ^− 4h
k = = 6−4 = 2
2+1 3 3
23. In the given figure, PQR is a triangle right angled
at Q and XY | | QR. If PQ = 6 cm, PY = 4 cm and
PX: XQ = 1: 2. Calculate the length of PR and QR. [2]

Ans :

Let the +ACB be θ .


tan θ = AB = 20 = 1 = tan 30c
BC 20 3 3
Thus θ = 30c
20. The radius of sphere is r cm. It is divided into two
equal parts. Find the whole surface of two parts. [1]
Ans :
Ans :
Whole surface of each part = 2πr2 + πr2 = 3πr2
Since XY || OR , by BPT we have
Total surface of two parts = 2 # 3πr2 = 6πr2
PX = PY
XQ YR
Section B 1 = PY = 4
2 PR − PY PR − 4
21. Is the system of linear equations 2x + 3y − 9 = 0 and
4x + 6y − 18 = 0 consistent? Justify your answer. [2] PR − 4 = 8
Ans : PR = 12 cm
For the equation 2x + 3y − 9 = 0 , we have In right TPQR we have
a2 = 2, b1 = 3 and c1 = − 9 QR2 = PR2 − PQ2 = 122 − 62
and for the equation, 4x + 6y − 18 = 0 , we have = 144 − 36 = 108
a2 = 4, b2 = 6 and c2 = − 18 Thus QR = 6 3 cm
Here a1 = 2 = 1
a1 4 2 or
b1 = 3 = 1 In an equilateral triangle ABC, AD is drawn
b2 6 2 perpendicular to BC meeting BC in D. Prove that
c1 = − 9 = 1 AD2 = 3BD2 .
and
c2 − 18 2 Ans :

To Get 20 Solved Paper Free PDF by whatsapp add +91 89056 29969 in your class Group Page 3
Mathematics Standard X Solved Sample Paper 4 www.cbse.online

In TABD, from Pythagoras theorem, 25. Find the number of plates, 1.5 cm in diameter and 0.2
cm thick, that can be fitted completely inside a right
circular of height 10 cm and diameter 4.5 cm. [2]
Ans :
Each one of he circular plate is also a cylinder.
Volume of plate Vp = πrr h = π # ^.75h2 ^.2h
= 9π cm3
80
Volume of right circular cylinder
Vc = π ^2.25h2 ^10h = 405 π cm3
8
AB2 = AD2 + BD2 405π
Number of plates = 8
= 405π # 80
Since AB = BC = CA , we get

80
9π 8
BC2 = AD2 + BD2, = 450 plates.
Here, BC = 2BD or
A sphere of diameter 6 cm is dropped in a right
^2BD h = AD + BD
2 2 2

circular cylindrical vessel partly filled with water. The


4BD − BD2 = AD2
2
diameter of the cylindrical vessel is 12 cm. If the sphere
3BD2 = AD2 is completely submerged in water, by how much will
24. One tends to become lazy. Also, starting at your mobile the level of water rise in the cylindrical vessel ?
screen for long hours can affect your eyesight and give Ans :
you headaches . Those who are addicted to playing
PUBG can get easily stressed out or face anxiety Given, Radius of sphere, r1 = d = 6 = 3 cm
2 2
issues in public due to lack of social interaction.
To raise social awareness about ill effects of playing Radius of cylinder vessel, r2 = 12 = 6 cm
2
PUBG, a school decided to start “BAN PUBG”
Let the level of water rise in cylinder be h .
campaign, students are asked to prepare campaign
students are asked to prepare campaign board in the Volume of sphere = 4 πr3
shape of rectangle (as shown in the figure). 3

= 4 #π#3#3#3
3
= 36π cm3
Volume of sphere = Increase volume in cylinder
36π = π ^6 h2 # h
h = 1 cm
26. Milk in a container, which is in the form of frustum of
a cone of height 30 cm and the radii of whose lower an
upper circular ends are 20 cm and 40 cm respectively,
is to be distributed in a camp for flood victims. If
this milk is available at the rate of `35 per litre and
880 litre of milk is needed daily for a camp, find how
(i) Find the area of the board.
many such containers of milk are needed for a camp
(ii) If cost of 1 cm2 of board is `8, then find the cost
and what cost will it put on the donor agency for this.
of board.
What value is indicated through this by the donor
Ans : agency?
(i) From the figure, we have Ans :
AB = ^8 − 2h2 + ^2 − 2h2 Volume of the milk container = Volume of frustum
= ^6 h + ^0h = 6 cm
2 2
= 1 πh 6R2 + r2 + Rr @
3
BC = ]8 − 8g2 + ^6 − 2h2
=^0h2 + 42 = 4 cm = 1 π # 30 ^402 + 202 + 40 # 20h
3
Area of board = Area of rectangle ABCD = 10π ^1600 + 400 + 800h
= AB # BC
= 10 # 22 # 2800
= 6 # 4 = 24 cm2 ...(1) 7

(ii)Total cost of board = 88000 cm3


= Area of board × Rate = 88 litre
Value: Helping the floods victims.
= 24 # 8 = `192

Download 20 Solved Sample Papers pdfs from www.cbse.online or www.rava.org.in Page 4


Mathematics Standard X Solved Sample Paper 4 www.rava.org.in

Section C Sn = n 82a + ^n − 1h dB
2
27. If one the zero of a polynomial 3x2 − 8x + 2k + 1 is
a + 9d = − 37 ...(1)
seven times the other, find the value of k . [3]
Ans : 3 ^2a + 5d h = − 27
2a + 5d = − 9 ...(2)
We have f ^x h = 3x2 − 8x + 2k + 1 Multiplying (1) by 2 and subtracting (2) from it, we
Let α and β be the zeroes of the polynomial, then get
β = 7α ^2a + 18d h − ^2a + 5d h = − 74 + 9
Sum of zeroes, α + β = − b− 8 l 13d = − 65
3
d =− 5
α + 7α = 8α = 8
3 Substituting the value of d in equation (1), we get
a + 9 # − 5 = − 37
So α =1
3
a = − 37 + 45
Product of zeroes, α # 7α = 2k + 1 a =8
3
Now Sn = n 82a + ^n − 1h dB
7α2 = 2k + 1 2
3
= 8 62 # 8 + ^8 − 1h^− 5h@
7 b 1 l = 2k + 1
2
2
3 3
= 4 616 − 35@
7 # 1 = 2k + 1
9 1 = 4 # − 19 = − 76
7 − 1 = 2k Hence, Sn = − 76
3
4 = 2k & k = 2 29. The vertices of TABC are A ^6, − 2h, B ^0, − 6h and
3 3 C ^4, 8h . Find the co-ordinates of mid-points of AB, BC
or and AC. [3]
−3
Ans :
Show that and1
are the zeroes of the polynomial
Let mid-point of AB, BC and AC be D ^x1, y1h,
2 2
4x2 + 4x − 3 and verify relationship between zeroes
and coefficients of the polynomial. E ^x2, y2h and F ^x2, y3h . As per question, triangle is
Ans : shown below.

We have p ^x h = 4x2 + 4x − 3
−3
If 2 and 2 are the zeroes of the polynomial p (x),
1

then these must satisfy p (x) = 0


pb 1 l = 4b 1 l + 4b 1 l − 3
2 4 2
= 1+2−3 = 0
and p b− l = 4 b 9 l + 4 b− 3 l − 3
3
2 2 2
= 9−6−3 = 0
Thus 1 , − 3 are zeroes of polynomial 4x2 + 4x − 3 . Using section formula, the co-ordinates of the points
2 2 D, E, F are
Sum of zeroes = 1 − 3 = − 1 = − 4
2 2 4 For D, x1 = 6 + 0 = 3 y1 = − 2 − 6 = − 4
2 2
= − Coefficient of x2
Coefficient of x For E, x2 = 0 + 4 = 2 y2 = − 6 + 8 = 1
2 2
Product of zeroes = b 1 lb− 3 l = − 3
2 2 4 For F , x3 = 4 + 6 = 5 y3 = − 2 + 8 = 3
2 2
= Constan term 2 Verified The co-ordinates of the mid-points of AB, BC and
Coefficient of x AC are D ^3, − 4h , E ^2, 1h and F ^5, 3h respectively.
28. The tenth term of an A.P., is − 37 and the sum of its
first six terms is − 27 . Find the sum of its first eight or
terms. [3] Find the ratio in which the point p ^m, 6h divides the
Ans : line segment joining the points A ^− 4, 3h and B ^2, 8h .
Let the first term be a , common difference be d , n th Also find the value of m.
term be an and sum of n term be Sn . Ans :
an = a + ^n − 1h d As per question, line diagram is shown below.

To Get 20 Solved Paper Free PDF by whatsapp add +91 89056 29969 in your class Group Page 5
Mathematics Standard X Solved Sample Paper 4 www.cbse.online

other along AD , as shown in figure. Niharika runs 14


th

the distance AD in the 2 nd line and posts a green flag.


Preet runs 15 th the distance AD on the eights line and
posts a red flag. [3]
Let the ratio be k : 1 (i) What is the distance between both the flags?
Using section formula, we have (ii) If Rashmi has to post a blue flag exactly halfway
2k + ^− 4h between the line segment joining the two flags,
m = (1)
k+1 where should she post her flag?
(iii) Which mathematical concept is used in the above
6 = 8k + 3 (2) problem?
k+1
8k + 3 = 6k + 6
2k = 3
k =3
2
Thus ratio is 3
2 : 1 or 3: 2.
Substituting value of k in (1) we have
2 ^ 32 h + ^− 4h
m =
2 +1
3

= 3−
5
4
2

= −5 1 = − 2
2
5
30. In the given figure, OP is equal to the diameter of a
circle with center O and PA and PB are tangents.
Prove that ABP is an equilateral triangle. [3]
Ans :
(i) Considering A as origin (0,0), AB as X -axis and
AD as Y -axis.
Niharika runs in the 2 nd line with green flag and
distance covered (parallel to AD )
= 1 # 100 = 25 m
4
Co-ordinates of green flag are (2, 25) and label it
as P i.e., P ^2, 25h .
Similarly, Preet runs in the eights line with red
Ans :
flag and distance covered (parallel to AD )
We redraw the given figure by joining A to B as
shown below. = 1 # 100 = 20 m
5
Since OA is radius and PA is tangent at A, OA = AP Co-ordinates of red flag are (8,20) and label it as
. Now in right angle triangle ΔOAP , OP is equal to Q , i.e., Q ^8, 20h
diameter of circle, thus Now, using distance formula, distance between
OP = 2OA green flag and blue flag.
^8 − 2h2 + ^20 − 25h
OA = 1 2
PQ =
OP 2
= 62 + ^− 5h2 = 36 + 25
sin θ = 1 & θ = 30c = 61 m
2
Since PO bisect the angle +APB , (ii) Also, Rashmi has to post a blue flag at the
Hence, +APB = 2 # 30c = 60º mid-point of PQ , therefore by using mid-point
Now, in TAPB , formula, we have b 2 + 8 , 25 + 20 l i.e., b 5, 45 l
2 2 2
AP = AB Hence, the blue flag is in the fifth line, at a
+PAB = +PBA distance of 452 i.e., 22.5 m along the direction
parallel to AD .
= 180c − 60c = 60º (iii) Co-ordinate Geometry.
2
Thus TAPB is an equilateral triangle. 32. An aeroplane, when flying at a height of 4000 m from
the ground passes vertically above another aeroplane
31. To conduct Sport Day activities, in your rectangular
at an instant when the angles of elevation of the two
shaped school ground ABCD , lines have been draw
planes from the same point on the ground are 60º and
with chalk power at a distance of 1 m each. 100 flower
45º respectively. Find the vertical distance between
pots have been placed at a distance of 1 m from each

Download 20 Solved Sample Papers pdfs from www.cbse.online or www.rava.org.in Page 6


Mathematics Standard X Solved Sample Paper 4 www.rava.org.in

the aeroplanes at that instant. (Use 3 = 1.73) [3]


Ans : Distance between the two men,

Let the height of first plane be AB = 4000 m and the PQ = BP + BQ = 75 3 + 25 3


height of second plane be BC = x m. As per given in = 100 3 = 100 # 1.73 = 173
question we have drawn figure below. 33. In the fig., PSR, RTQ and PAQ are three semi-circles
of diameters 10 cm, 3 cm and 7 cm region. Use π = 227
. [3]

Ans :
Here +BDC = 45º and +ADB = 60º Perimeter of shaded region = Perimeter of semi-circles
x = tan 45º = 1 & x = y PSR + RTQ + PAQ
In TCBD ,
y Perimeter of shaded region

and in TABD , 4000 = tan 60º = 3 = π ^5 h + π ^1.5h + π ^3.5h


y
= π ^10h
y = 4000 3 = 22 # 10 = 22
3 7 7
= 2309.40 m Perimeter of shaded region = 31.4 cm. (approx)
Thus vertical distance between two, 34. In class 10 A, there are 20 boys and 20 girls. In 10 B,
4000 − y = 4000 − 2309.40 = 1690.59 cm there are 15 boys and 25 girls. One student is to be
selected from each class. [3]
or (i) What is the probability of both being girls?
Two men on either side of a 75 m high building and (ii) What is the probability of both being boys?
in line with base of building observe the angles of (iii) What is the probability of one boy and one girl?
elevation of the top of the building as 30º and 60º. find Ans :
the distance between the two men. (Use 3 = 1.73)
Total student in 10 A = 20 boys + 20 girls
Ans :
= 40
Let AB be the building and the two men are at P
and Q. As per given in question we have drawn figure Total students in 10 B = 15 boys + 25 girsl
below. = 40
P (girl from 10 A) = 20
40

(i) P (both are girls) = 20 # 25


40 40

=1#5 = 5
2 8 16

(ii) P (both are boys) = 20 # 15


40 40

=1#3 = 3
2 8 16
In TABP , tan 30º = AB (iii) Probability of one boy and one girl.
BP
Case I: Boy from 10 A and girl from 10 B.
1 = 75
3 BP It’s probability = 20 # 25 = 5
40 40 16
BP = 75 3 m
Case II: Girl from 10 A and boy from 10 B.
In TABQ, tan 60º = AB
BQ It’s probability = 20 # 15 = 3
40 40 16
3 = 75
BQ P (one boy and one girl) = 5 + 3 = 8 = 1
16 16 16 2
BQ = 75 = 25 3
3
To Get 20 Solved Paper Free PDF by whatsapp add +91 89056 29969 in your class Group Page 7
Mathematics Standard X Solved Sample Paper 4 www.cbse.online

Section D From (2) y = 100 + x


x 0 700
35. Prove that n2 − n is divisible by 2 for every positive
integer n . [4] y 100 800
Ans :
Plotting the above points and drawing lines joining
We have n − n = n ^n − 1h
2 them, we get the following graph.
Thus n2 − n is product of two consecutive positive
integers.
Any positive integer is of the form 2q or 2q + 1, for
some integer q .
Case 1 : n = 2q
If n = 2q we have
n ^n − 1h = 2q ^2q − 1h
= 2m ,
where m = q ^2q − 1h which is divisible by 2.
Case 1 : n = 2q + 1
Clearly, the two lines intersect at point (700, 800)
If n = 2q + 1, we have
Hence X-A contributes 700 Rs and X-B contributes
n ^n − 1h = (2q + 1) ^2q + 1 − 1h 800 Rs.
= 2q ^2q + 1h = 2m 37. Two pipes running together can fill a tank in 11 19
where m = q ^2q + 1h which is divisible by 2. minutes. If one pipe takes 5 minutes more than the
other to fill the tank, find the time in which each pipe
Hence, n2 − n is divisible by 2 for every positive would fill the tank separately. [4]
integer n .
Ans :
or Let time taken by pipe A be x minutes and time
If d is the HCF of 30 and 72, find the value of x and taken by pipe B be x + 5 minutes.
y satisfying d = 30x + 72y . In one minute pipe A will fill 1 tank.
x
Ans :
In one minute pipe B will fill 1 tank.
Using Euclid’s Division Lemma, we have x+5
Thus pipes A + B will fill 1 + 1 tank in one
72 = 30 # 2 + 12 ...(1) x x+5
minute.
30 = 12 # 2 + 6 ...(2)
As per question, two pipes running together can fill
12 = 6 # 2 + 0 ...(3) a tank in 11 19 = 100
9 minutes, in one minute 100
9
tank
Thus HCF(30, 72) = 6 will be filled.
Now according to the question we have
Now 6 = 30 − 12 # 2 From (2)
1+ 1 = 9
6 = 30 − (72 − 30 # 2) # 2 From (1) x x+5 100
6 = 30 − 72 # 2 + 30 # 4 x+5+x = 9
6 = 30 (1 − 4) − 72 # 2 x ^x + 5h 100
6 = 30 # 5 + 72 # (− 2) 100 ^2x + 5h = 9x ^x + 5h
6 = 30x + 72y 200x + 500 = 9x2 + 45x
Thus x = 5 and y =− 2 .Here x and y are not unique. 9x2 − 155x − 500 = 0
36. For Uttarakhand flood victims two sections A and B of 9x2 − 180x + 25x − 500 = 0
class contributed Rs. 1,500. If the contribution of X-A 9x ^x − 20h + 25 ^x − 20h = 0
was Rs. 100 less than that of X-B, find graphically the
amounts contributed by both the sections. [4] ^x − 20h^9x + 25h = 0
Ans : x = 20, − 25
9
Let amount contributed by two sections X-A and X-B As time can’t be negative we take x = 20 minutes
be Rs. x and Rs.y . and x + 5 = 25 minutes
x + y = 1, 500 ...(1) Hence pipe A will fill the tank in 20 minutes and pipe
y − x = 100 ...(2) B will fill it in 25 minutes.
From (1) y = 15000 − x
or
x 0 700 1,500
The perimeter of a right triangle is 60 cm. Its
y 1,500 800 0 hypotenuse is 25 cm. Find the area of the triangle.

Download 20 Solved Sample Papers pdfs from www.cbse.online or www.rava.org.in Page 8


Mathematics Standard X Solved Sample Paper 4 www.rava.org.in

Ans :
As per question statement figure is given below.

Ans :

Since, TFEC , TGBD


Here a + b + c = 60, c = 25 EC = BD ...(1)
a + b = 60 − c = 60 − 25 = 35 Since, +1 = +2 , using isosceles triangle property
Using Pythagoras theorem AE = AD ...(2)
a + b = 25 = 625
2 2 2
From equation (1) and (2), we have
Substituting the values in ^a + b h = a + b + 2ab ,
2 2 2
AE = AD
352 = 625 + 2ab EC BD
1225 − 625 = 2ab DE || BC, (Converse of BPT)
or, ab = 300 Due to corresponding angles, we have
Hence, Area of TABC +1 = +3 and +2 = +4
1 ab = 150 cm2 . Thus in TADE and TABC,
2
+A = +A
38. In TABC, if +ADE = +B, then prove that
TADE ~TABC. Also, if AD = 7.6 cm, AE = 7.2 cm, +1 = +3
BE = 4.2 cm and BC = 8.4 cm, then find DE. [4] +2 = +4
Sy by AAA criterion of similarity,
TADE~TABC Hence proved
39. If cos θ + sin θ = p and sec θ + cosec θ = q, prove that
q ^p2 − 1h = 2p [4]
Ans :
We have cos θ + sin θ = p and sec θ + cosec θ = q
q ^p2 − 1h = ^sec θ + cosec θh [^cos θ + sin θh2 − 1]
= ^sec θ + cosec θh^cos2 θ + sin2 θ + 2 sin θ cos θ − 1h
Ans : = ^sec θ + cosec θh [1 + 2 sin θ cos θ − 1]

In TADE and TABC, +A is common = b 1 + 1 l^2 sin θ cos θh


cos θ sin θ
and +ADE = +ABC (Given)
= sin θ + cos θ # 2 sin θ cos θ
Due to AA similarity cos θ sin θ
TADE ~TABC = 2 ^sin θ + cos θh = 2p Hence Proved.
AD = DE 40. Find the value of x and y , if the median for the
AB BC following data is 31. [4]
AD = DE Classes 0- 10- 20- 30- 40- 50- Total
AE + BE BC 10 20 30 40 50 60
7.6 = DE Frequency 5 x 6 y 6 5 40
7.2 + 4.2 8.4
Ans :
DE = 7.6 # 8.4 = 5.6 cm
11.4 C.I. f c.f.
or
0-10 5 5
In the following figure, TFEC , TGBD and
10-20 x 5+x
+1 = +2 . Prove that TADE , TABC.
20-30 6 11+ x
30-40 y 11+ x+y
40-50 6 17+ x+y
50-60 5 22+ x+y

To Get 20 Solved Paper Free PDF by whatsapp add +91 89056 29969 in your class Group Page 9
Mathematics Standard X Solved Sample Paper 4 www.cbse.online

Here from table, N = 22 + x + y = 40


& x + y = 18 ...(1)
Since, Median = 31, which lies between 30-40
` Median class = 30 − 40
N − c.f.
Median = l + 2f p# h
f
20 − ^11 + x h
& 31 = 30 + = G # 10
y
^9 − x h # 10
& 1 =
y
& y = 90 − 10x
10x + y = 90 ...(2)
From equation (1),
y = 18 − x
From equation
10x + 18 − x = 90
9x = 72
x = 72 = 8
9
Similarly, from equation (1), we get
8 + y = 18
y = 10
WWW.CBSE.ONLINE

Download unsolved version of this paper from


www.cbse.online

This sample paper has been released by website www.cbse.online for the benefits of the students. This paper has been prepared by
subject expert with the consultation of many other expert and paper is fully based on the exam pattern for 2019-2020. Please note
that website www.cbse.online is not affiliated to Central board of Secondary Education, Delhi in any manner. The aim of website is
to provide free study material to the students.
Download 20 Solved Sample Papers pdfs from www.cbse.online or www.rava.org.in Page 10
Mathematics Standard X Solved Sample Paper 5 www.rava.org.in

CLASS X (2019-20)
MATHEMATICS STANDARD(041)
SAMPLE PAPER-5

Time : 3 Hours Maximum Marks : 80


General Instructions :
(i) All questions are compulsory.
(ii) The questions paper consists of 40 questions divided into 4 sections A, B, C and D.
(iii) Section A comprises of 20 questions of 1 mark each. Section B comprises of 6 questions of 2 marks each. Section
C comprises of 8 questions of 3 marks each. Section D comprises of 6 questions of 4 marks each.
(iv) There is no overall choice. However, an internal choices have been provided in two questions of 1 mark each, two
questions of 2 marks each, three questions of 3 marks each, and three questions of 4 marks each. You have to
attempt only one of the alternatives in all such questions.
(v) Use of calculators is not permitted.

Section A Now 5 (10x + y) = 10y + x


6

Q.1-Q.10 are multiple choice questions. Select the Solving, we get 44x = 55y
most appropriate answer from the given options. x =5 ...(1)
y 4
1. Two positive numbers have their HCF as 12 and their Also x − y = 1. Solving them, we get x = 5 and y = 4
product as 6336. The number of pairs possible for the
. Therefore, number is 54.
numbers, is [1]
(a) 2 (b) 3 4. If one root of the quadratic equation ax2 + bx + c = 0
(c) 4 (d) 5 is the reciprocal of the other, then [1]
(a) b = c (b) a = b
Ans : (a) 2
(c) ac = 1 (d) a = c
Let the numbers be 12x and 12y where x and y are
Ans : (d) a = c
co-primes.
Product of these numbers = 144xy If one root is α , then the other 1 .
α
Hence, 144xy = 6336 & xy = 44 α$ 1 = product of roots = c
α a
Since, 44 can be written as the product of two factors
in three ways. i.e. 1 # 44 , 2 # 22 , 4 # 11. As x and 1 =c
a
y are co-prime, so ^x, y h can be (1,44) or (4,11) but
a =c
not (2,22).
Hence, two possible pairs exist. 5. If the common difference of an AP is 5, then what is
a18 − a13 ? [1]
2. If α and β are zeroes and the quadratic polynomial
(a) 5 (b) 20
f ^x h = x2 − x − 4 , then the value of 1 + 1 − αβ is [1]
α β (c) 25 (d) 30
(a) 15 (b) − 15 Ans : (c) 25
4 4
Given, the common difference of AP i.e., d = 5
(c) 4 (d) 15
Now, a18 − a13 = a + ^18 − 1h d − 6a + ^13 − 1h d @
Ans : (a) 15
4 6Since, an = a + ^n − 1h d @
Given that, f ^x h = x − x − 4
2
= a + 17 # 5 − a − 12 # 5
α + β = 1 and αβ = − 4 = 85 − 60 = 25
We have, + 1 − αβ =
1 α+β
− αβ = − 1 + 4 = 15 6. If f ^x h = cos x + sec x , then f ^x h
2 2
[1]
α β αβ 4 4
(a) $ 1 (b) # 1
3. The 2 digit number which becomes (5/6)th of itself
when its digits are reversed. The difference in the (c) $ 2 (d) # 2
digits of the number being 1, then the two digits Ans : (c) $ 2
number is [1]
(a) 45 (b) 54 Given, f ^x h = cos2 x + sec2 x
(c) 36 (d) None of these = cos2 x + sec2 x − 2 + 2
Ans : (b) 54 [adding and subtracting 2]
If the two digits are x and y , then the number is = cos x + sec x − 2 cos x $ sec x + 2
2 2

10x + y . 6cos x $ sec x = 1@


To Get 20 Solved Paper Free PDF by whatsapp add +91 89056 29969 in your class Group Page 1
Mathematics Standard X Solved Sample Paper 5 www.cbse.online

= ^cos x − sec x h + 2 Hence, Favourable number of elementary events = 2


2

8a + b − 2ab = ^a − b h B
2 2 2
Hence, P (getting a blue marble) = 2
5
We know that, square of any expression is always
(Q.11-Q.15) Fill in the blanks.
greater than equal to zero.
11. If a = bq + r , least value of r is .......... [1]
f ^x h $ 2 Hence proved.
Ans : Zero
7. A sector is cut from a circular sheet of radius 100 cm,
the angle of the sector being 240c. If another circle of 12. Area of a rhombus if its vertices are (3, 0), (4, 5),
the area same as the sector is formed, then radius of (− 1, 4) and (− 2, − 1) taken in order is .......... [1]
the new circle is [1] Ans : 24. Sq. units
(a) 79.5 cm (b) 81.5 cm
(c) 83.4 cm (d) 88.5 cm or
Ans : (b) 81.5 cm Points (3, 2), (− 2, − 3) and (2, 3) form a ..........
triangle.
Area of sector = 240/360 # π ^100h 2

Ans : Right angle


= 20933 cm2
Let r be the radius of the new circle, then 13. In TABC , right-angled at B , AB = 24 cm , BC = 7 cm
20933 = πr 2 . sin A = .......... [1]
20933 = 81.6 cm Ans : 7/25
r =
π
8. A slab of ice 8 inches in length, 11 inches in breadth, 14. Length of an arc of a sector of a circle with radius r
and 2 inches thick was melted and re-solidified in the and angle with degree measure θ is .......... [1]
form of a rod of 8 inches diameter. The length of such Ans : θ 2πr
360 #
a rod, in inches, is nearest to [1]
(a) 3 (b) 3.5 15. The volume of a cube with diagonal d is .......... [1]
3
(c) 4 (d) 4.5 Ans : d cu units.
3 3
Ans : (b) 3.5
(Q.16-Q.20) Answer the following
Volume of the given ice cuboid = 8 # 11 # 2 16. In given figure DE | | BC. If AD = 3 cm, DB = 4 cm
= 176 and AE = 6cm, then find EC. [1]
Let the length of the required rod be l .
2
πl 8 = 176
4
l = 3.5 inches
9. If the difference of mode and median of a data is 24,
then the difference of median and mean is [1]
(a) 12 (b) 24
(c) 08 (d) 36
Ans : (a) 12
Ans :
We have,
Mode − Median = 24 In the given figure DE || BC , thus
We know that, Mode = 3 (Median) − 2 Mean AD = AE
AB EC
Mode − Median = 2 Median − 2 Mean
3 = 6
24 = 2 (Median − Mean) 4 EC
Median − Mean = 12 EC = 8 cm
10. A bag contains 3 red and 2 blue marbles. If a marble 17. In the given figure, AOB is a diameter of the circle
is drawn at random, then the probability of drawing a with centre O and AC is a tangent to the circle at A
blue marble is: [1] . If +BOC = 130° , the find +ACO. [1]
(a) 1 (b) 2
5 5
(c) 3 (d) 4
5 5
Ans : (b) 2
5
There 5 marbles in the bag. Out of these 5 marbles
one can be choose in 5 ways.
Hence, Total number of possible outcomes = 5
Since, the bag contains 2 blue marbles. Therefore, one Ans :
blue marble can be drawn in 2 ways.
Download 20 Solved Sample Papers pdfs from www.cbse.online or www.rava.org.in Page 2
Mathematics Standard X Solved Sample Paper 5 www.rava.org.in

Here OA is radius and AC is tangent at A, since 19. To divide a line segment AB in the ratio 2: 5, a ray
radius is always perpendicular to tangent, we have AX is drawn such that +BAX is acute. Then points
+OAC = 90º are marked at equal intervals on AX . What is the
From exterior angle property, minimum number of these points ? [1]
Ans :
+BOC = OAC + +ACO
Minimum number of points marked on, AX = 2 + 5 = 7
130º = 90° + +ACO
+ACO = 130º − 90º = 40º 20. What is the volume of a right circular cylinder of base
radius 7 cm and height 10 cm ? (Use π = 227 ) [1]
18. A ladder 15 m long leans against a wall making an Ans :
angle of 60º with the wall. Find the height of the point
where the ladder touches the wall. [1] Here r = 7 cm, h = 10 cm,
Ans : Volume of cylinder = πr2 h
Let the height of wall be h . As per given in question = 22 # ^7 h2 # 10
we have drawn figure below. 7
= 1540 cm3

Section B
21. For what value of k , the pair of linear equations
kx − 4y = 3, 6x − 12y = 9 has an infinite number of
solutions ? [2]
Ans :

We have kx − 4y − 3 = 0
h = cos 60º
15 and 6x − 12y − 9 = 0
where, a1 = k, b1 = 4, c1 = − 3
h = 15 # cos 60º
a2 = 6, b2 = − 12, c2 − 9
= 15 # 1
2 Condition for infinite solutions:
= 7.5 m
a1 = b1 = c1
a2 b2 c2
or
k = −4 = 3
An observer, 1.7 m tall, is 20 3 m away from a tower. 6 − 12 9
The angle of elevation from the eye of observer to the
top of tower is 30º. Find the height of tower. Hence, k =2
Ans :
22. The x-coordinate of a point P is twice its y-coordinate.
Let height of the tower AB be h . As per given in If P is equidistant from Q ^2, − 5h and R ^− 3, 6h , find
question we have drawn figure below. the co-ordinates of P. [2]
Ans :
Let the point P ^2y, y h ,
Since PQ = PR , we have

^2y − 2h2 + ^y + 5h2 = ^2y + 3h2 + ^y − 6h2

^2y − 2h2 + ^y + 5h2 = ^2y + 3h2 + ^y − 6h2


− 8y + 4 + 10y + 25 = 12y + 9 − 12y + 36
2y + 29 = 45
y =8
Here AE = h − 1.7
Hence, coordinates of point P are ^16, 8h
and BC = DE = 20 3
23. Find the altitude of an equilateral triangle when each
In TADE, +E = 90º of its side is 'a' cm. [2]
tan 30º = h − 1.7 Ans :
20 3
Let TABC be an equilateral triangle of side a and
1 = h − 1.7 AD is altitude which is also a perpendicular bisector
3 20 3 of side BC. This is shown in figure given below.
h − 1.7 = 20
or h = 20 + 1.7 = 21.7 m

To Get 20 Solved Paper Free PDF by whatsapp add +91 89056 29969 in your class Group Page 3
Mathematics Standard X Solved Sample Paper 5 www.cbse.online

= 4 πr 3
3
3
(b) Volume of the sphere = 4 π d r n
3 3
3
= 4π r
3 27
4
πr 3
Number of spheres = 3
= 27
π 27
3
4 r
3

25. A sphere of maximum volume is cut out from a solid


hemisphere of radius 6 cm. Find the volume of the cut
a2 = a a k + h2
2
In TABD, out sphere. [2]
2
2 2 Ans :
h2 = a2 − a = 3a
4 4 Diameter of sphere = Radius of hemisphere
Thus h = 3a = 6 cm
2
Radius of sphere = 3 cm
or
Volume, V = 4 πr3 = 4 # 22 # 33 cm3.
In an equilateral triangle of side 3 3 cm find the 3 3 7
length of the altitude.
= 113.14 cm3.
Ans :
or
Let TABC be an equilateral triangle of side 3 3 cm
and AD is altitude which is also a perpendicular bisector A cone of height 24 cm and radius of base 6 cm is
of side BC. This is shown in figure given below. made up of clay. If we reshape it into a sphere, find
the radius of sphere.
Ans :

Volume of sphere =Volume of cone


4 πr 3 = 1 πr 2 h
3 1 3 2

^ h# 3
4 r 3 = 6 2 24
3# 1
4r13 = 36 # 24
r13 = 63
r1 = 6 cm
2
^3 3 h = h + c 2 m
Now
2 2 3 3 Hence, radius of sphere is 6 cm.
26. Ramesh, a juice seller has set up his juice shop. He has
27 = h2 + 27 three types of glasses of inner diameter 5 cm to serve
4
the customers. The height of the glasses is 10 cm. (Use
h2 = 27 − 27 = 81 π = 3.14 ). [2]
4 4

h = 9 = 4.5 cm
2
24. A hemisphere and a cone both have same diameter.
These two metal solids are joined by putting their
bases together. The height of the cone is equal to the
diameter of the sphere. This solid is melted and recast
into a sphere of a diameter equal to one third of the
diameter of the hemisphere. [2]
(a) If radius of the hemisphere is r , find the volume
of the combined solid.
(b) Find the number of spheres.
Ans :

(a) Radius of the hemisphere = r


Height of the cone = 2r
He decided to serve the customer in A” type of glasses.
Volume of the solid = 1 πr2 # 2r + 2 πr3 Find the volume of glass of type A and which glass
3 3
has the minimum capacity.
= 2 πr 3 + 2 πr 3 Ans :
3 3

Download 20 Solved Sample Papers pdfs from www.cbse.online or www.rava.org.in Page 4


Mathematics Standard X Solved Sample Paper 5 www.rava.org.in

Given, Diameter = 5 cm Ans :


Radius = 2.5 cm We have p ^x h = 5x2 + 8x − 4 = 0
Height = 10 cm = 5x2 + 10x − 2x − 4 = 0
Volume of glass of type A = πr h 2
= 5x ^x + 2h − 2 ^x + 2h = 0
= 3.14 # 2.5 # 2.5 # 10 = ^x + 2h^5x − 2h
= 196.25 cm3 Substituting p (x) = 0 we get zeroes as − 2 and 25 .
Volume of hemisphere Verification :
= 2 πr 3 Sum of zeroes = − 2 + 2 = − 8
3 5 5
= 2 # 3.14 # 2.5 # 2.5 # 2.5 Product of zeroes = ^− 2h # b 2 l = − 4
3 5 5
= 32.71 cm3 Now from polynomial we have
Volume of glass of type B
= 196.25 − 32.71 Sum of zeroes − b = − Coefficient of x2 = − 8
a Coefficient of x 5
= 163.54 cm3
Product of zeroes c = Constan term 2 = − 4
a Coefficient of x 5
Volume of cone = 1 πr2 h
3
28. If 7th term of an A.P. is 1 and 9th term is 1 , find 63rd
9 7
= 1 # 3.14 # 2.5 # 2.5 # 1.5 term. [3]
3
Ans :
= 3.14 # 2.5 # 2.5 # 0.5
Let the first term be a , common difference be d and
= 9.81 cm3 n th term be an .
Volume of glass of type C
We have a7 = 1 & a + 6d = 1 (1)
= 196.25 − 9.81 9 9
= 186.44 cm3 a9 = 1 & a + 8d = 1 (2)
Hence the glass of type B has the minimum capacity. 7 7
Subtracting equation (1) from (2), we get
Section C 2d = 1 − 1 = 2 = d
7 9 63
27. Verify whether 2, 3 and 12 are the zeroes of the
polynomial p ^x h = 2x3 − 11x2 + 17x − 6 . [3] d = 1
63
Ans : Substituting the value of d in (2) we get
If 2, 3 and 12 are the zeroes of the polynomial p (x),
then these must satisfy p (x) = 0 a+8# 1 = 1
63 7
(1) p ^x h = 2x2 − 11x2 + 17x − 6
a = 1 − 8 = 9−8 = 1
p ^2 h = 2 ^2 h3 − 11 ^2 h2 + 17 ^2 h − 6 7 63 63 63

= 16 − 44 + 34 − 6 Thus a63 = a + (63 − 1) d


= 50 − 50 = 1 + 62 # 1 = 1 + 62
63 63 63
or p ^2 h = 0
= 63 = 1
(2) p ^3 h = 2 ^3 h3 − 11 ^3 h2 + 17 ^3 h − 6 63
Hence, a63 = 1
= 54 − 99 + 51 − 6
= 105 − 105 29. If the co-ordinates of points A and B are ^− 2, − 2h
and ^2, − 4h respectively, find the co-ordinates of
or p ^3 h = 0
P such that AP = 3 AB, where P lies on the line
3 2 7
(3) p b 1 l = 2 b 1 l − 11 b 1 l + 17 b 1 l − 6 segment AB. [3]
2 2 2 2
Ans :
= 1 − 11 + 17 − 6
4 4 2
We have AP = 3 AB & AP : PB = 3: 4
7
or pb 1 l = 0 As per question, line diagram is shown below.
2
Hence, 2, 3, and 1
2 are the zeroes of p ^x h .

or
Find the zeroes of the quadratic polynomial Section formula :
5x2 + 8x − 4 and verify the relationship between the my2 + nx1
x = mx2 + nx1 and y =
zeroes and the coefficients of the polynomial. m+n m+n
To Get 20 Solved Paper Free PDF by whatsapp add +91 89056 29969 in your class Group Page 5
Mathematics Standard X Solved Sample Paper 5 www.cbse.online

Ans :
Applying section formula, we get
x = 3 # 2 + 4 #− 2 =− 2
3+4 7

y = 3 # − 4 + 4 # − 2 = − 20
3+4 7

Hence P is b− 2 , − 20 l
7 7
or
If the distance of P ^x, y h from A ^6, 2h and B ^− 2, 6h
are equal, prove that y = 2x.
Ans :
We have P ^x, y h, A ^6, 2h, B ^− 2, 6h
Now PA = PB
PA2 = PB2
^x − 6h2 + ^y − 2h2 = ^x + 2h2 + ^y − 6h2
− 12x + 36 − 4y + 4 = 4x + 4 − 12y + 36
(i)Here, radius of two cones and cylinder
− 12x − 4y = 4x − 12y
= 3 cm
12y − 4y = 4x + 12x 2
8y = 16x = 1.5 cm
y = 2x Hence Proved Height of each cone = 2 cm
30. In the figure, PQ is a tangent to a circle with center Height of cylindrical portion = 12 − 2 − 2 = 8 cm
O . If +OAB = 30º , find +ABP and +AOB. [3] Volume of the air = Volumes of cylindrical part
+2×Volume of cone
= π ^1.5h2 # 8 + 2 1 π ^1.5h2 # 2
3

= 22 # ^1.5h :8 + 4 D
2
7 3

= 22 # 2.25 # 28
7 3
= 66 cm3
(ii) Volume (Mensuration).
32. In the given figure, a chord AB of the circle with
Ans :
centre O and radius 10 cm, that subtends a right
Here OB is radius and QT is tangent at B , OB = PQ angle at the centre of the circle. Find the area of the
+OBP = 90º minor segment AQBP . Hence find the area of major
Since the tangent is perpendicular to the end point segment A+LBQA . (Use π = 3.14 ) [3]
of radius,
Here OA and OB are radius of circle and equal. Since
angles opposite to equal sides are equal,
+OAB = +OBA = 30º
Now +AOB = 180º − ^30º + 30ºh
= 120º
+ABP = +OBP − +OBA
= 90º − 30º = 60º
31. Rachel, an engineering student, was asked to make a
model shaped like a cylinder with two cones attached Ans :
at its two ends by using a thin aluminium sheet. The
diameter of the model is 3 cm and its length is 12 cm. Area of sector OAPB ,
If each cone has a height of 2 cm. [3] A1 = 90 π ^10h2 = 25π cm2
(i) Find the volume of air contained in the model 360
that Rachel made. (Assume the outer and inner
Area of ΔAOB , A2 = 1 # 10 # 10 = 50 cm2
dimensions of the model to be nearly the same). 2
(ii) Which mathematical concept is used in the above Area of minor segment AQBP ,
problem ?
= ^25π − 50h cm2
Download 20 Solved Sample Papers pdfs from www.cbse.online or www.rava.org.in Page 6
Mathematics Standard X Solved Sample Paper 5 www.rava.org.in

= 25 # 3.14 − 50 journey was 48 kilometre/hour. [3]


= 78.5 − 50 (a) If the distance from home to town is 5 kilometre,
find his total journey time.
= 28.5 cm2 cm2 (b) By taking the speed of his journey from home to
Also area of circle = π ^10h2 town as x , form a second degree equations.
= 3.14 # 100 = 314 cm2 Ans :
Area of major segment ALBQA = 314 − 28.5
= 285.5 cm2

or
Find the area of minor segment of a circle of radius 14
cm, when its centre angle is 60c. Also find the area of (a) Given, Average speed = 48 km/hr
corresponding major segment. Use π = 227 .
Total distance = 10 km
Ans :
Here, r = 14 cm, θ = 60c Total time = 10 km
48
Area of minor segment = πr2 θ − 1 r2 sin θ = 10 # 60 min
360 2 48
= 22 # 14 # 14 # 60 − 1 # 14 # 14 # 3 = 12.5 min.
7 360c 2 2
(b) Time = Distance
= b 308 − 49 3 l = 17.9 cm2 approx. Speed
3
Time t1 (Home to town) = 5
Area of major segment = πr − b 308 − 49 3 l
2 x
3
Time t2 (town to home) = 5
= 1540 + 49 3 = 598.10 x + 20
3
Total Distance = Total time
= 598 cm2 approx.
×Average speed
33. An electric pole is 10 m high. A steel wire tied to top
of the pole is affixed at a point on the ground to keep 10 = c 5 + 5 m 48
x x + 20
the pole up right. If the wire makes an angle of 45º
with the horizontal through the foot of the pole, find 5 ^x + 20 + x h
10 =
x ^x + 20h
# 48
the length of the wire. [Use 2 = 1.414 ] [3]
Ans : 2x ^x + 20h = 48 ^2x + 20h
Let OA be the electric pole and B be the point on the x2 + 20x = 48x + 480
ground to fix the pole. Let BA be x . x2 − 28x − 480 = 0
As per given in question we have drawn figure below.

Section D
35. 4 chairs and 3 tables cost Rs 2100 and 5 chairs and 2
tables cost Rs 1750. Find the cost of none chair and
one table separately. [4]
Ans :
Let cost of 1 chair be Rs x and cost of 1 table be Rs
y According to the question,
4x + 3y = 2100 ...(1)
In TABO, we have 5x + 2y = 1750 ...(2)
sin 45º = AO Multiplying equation (1) by 2 and equation (2) by 3
AB
we get,
1 = 10
x 8x + 6y = 4200 ...(3)
2
15x + 6y = 5250 ...(4)
x = 10 2 = 10 # 1.414
= 14.14 m Subtracting equation (3) from (4) we have
Hence, the length of wire is 14.14 m 7x = 1050
34. A man travels from home to town and back in a x = 150
motor cycle. He travels to home from town at a speed Substituting the value of x in (1), y = 500
which is 20 kilometer/hour more than his journey to Thus cost of chair and table is Rs 150, Rs 500
the town from home. The average speed of his total respectively.

To Get 20 Solved Paper Free PDF by whatsapp add +91 89056 29969 in your class Group Page 7
Mathematics Standard X Solved Sample Paper 5 www.cbse.online

or x = 20
If a bag containing red and white balls, half the number Speed while going is 20 km/h and speed while
of white balls is equal to one-third the number of red returning will be = 20 + 10 = 30 km/h
balls. Thrice the total number of balls exceeds seven
times the number of white balls by 6. How many balls or
of each colour does the bag contain ? A motorboat whose speed in still water is 18 km/h,
Ans : takes 1 hour more to go 24 km upstream than to
Let the number of red balls be x and white balls be y . return downstream to the same spot. Find the speed
According to the question, of the stream.
1 y = 1 x or 2x − 3y = 0 Ans :
...(1)
2 3 Let the speed of stream be x km/h
and 3 ^x + y h − 7y = 6 Then the speed of boat upstream = ^18 − x h km/h
Speed of boat downstream = ^18 + x h km/h
or 3x − 4y = 6 ...(2)
According to the question,
Multiplying equation (1) by 3 and equation (2) we
have 24 − 24 = 1
18 − x 18 + x
6x − 9y = 0 ...(3)
24 [(18 + x) − (18 − x)]
6x − 8y = 12 ...(4) =1
182 − x2
Subtracting equation (3) from (4) we have
48x = 324 − x2
y = 12
x2 + 48x − 324 = 0
Substituting y = 12 in equation (1),
x2 + 54x − 6x − 324 = 0
2x − 36 = 0
x ^x + 54h − 6 ^x + 54h = 0
x = 18
^x + 54h^x − 6h = 0
Hence, number of red balls = 18
x + 54 = 0, x − 6 = 0
and number of white balls = 12
x = − 54, x = 6
36. Find HCF and LCM of 378, 180 and 420 by prime
factorization method. Is HCF × LCM of these numbers Since speed cannot be negative, we reject x = − 54 .
equal to the product of the given three numbers? [4] The speed of steam is 6 km/h.
Ans : 38. In the right triangle, B is a point on AC such
that AB + AD = BC + CD. If AB = x, BC = h and
Finding prime factor of given number we have,
CD = d, then find x (in term of h and d). [4]
378 = 2×33 ×7 Ans :
180 = 22 ×32 ×5
420 = 22 ×3×7×5
HCF(378, 180, 420) = 2×3 = 6
LCM(378, 180, 420) = 22 ×33 ×5×7
= 22 ×33 ×5×7 = 3780
HCF×LCM = 6×3780 = 22680
Product of given numbers
= 378×180×420
= 28576800 We have AB + AD = BC + CD
Hence, HCF × LCM ! Product of three numbers. AD = BC + CD − AB
37. The time taken by a person to cover 150 km was 2 12 AD = h + d − x
hours more than the time taken in the return journey.
In right angled triangle TACD,
If he returned at a speed of 10 km/hour more than
the speed while going, find the speed per hour in each AD2 = AC2 + DC2
direction. [4] ^h + d − x h2 = ^x + h h2 + d
2

Ans :
^h + d − x h2 − ^x + h h2 = d
2

Let the speed while going be x km/h


^h + d − x − x − h h^h + d − x + x + h h = d
2

Speed while returning = ^x + 10h km/h


^d − 2x h^2h + d h = d
2
According to question we have
150 − 150 = 5 2hd + d2 − 4hx − 2xd = d2
x x + 10 2 2hd = 4hx + 2xd
x2 + 10x − 600 = 0 = 2 ^2h + d h x
^x + 30h^x − 20h = 0
Download 20 Solved Sample Papers pdfs from www.cbse.online or www.rava.org.in Page 8
Mathematics Standard X Solved Sample Paper 5 www.rava.org.in

or, x = hd tan 55º tan 60º tan 85º.


2h + d
= cot 85º tan 85º cot 55º tan 55º $ 3
or
= 1#1# 3= 3
Prove that ratio of the areas of two similar triangles is
equal to the square of the ratio of their corresponding Now given expression = 1 − 1 − 1 + 3 = 3 −1
sides. 40. The median of the following data is 525. Find the
Ans : values of x and y if the total frequency is 100. [4]
As per given condition we have drawn the figure Class Interval Frequency
below. Here TABC ~TPQR 0-100 2
100-200 5
200-300 x
300-400 12
400-500 17
500-600 20
600-700 y
We have drawn AD = BC and PE = QR
700-800 9
Since TABC ~TPQR , due to corresponding sides of
similar triangles 800-900 7
AB AC = AC ...(1) 900-1000 4
PQ QR PR
N = 100
+B = +Q Ans :
In TADB and TPEQ,
Class Interval Frequency Cumulative frequency
+B = +Q (Proved)
+ADB = +PEQ [each 90º] 0-100 2 2

TADB ~TPEQ (AA Similarity) 100-200 5 7


Corresponding sides of similar triangle, 200-300 x 7+x
AD = AB ...(2) 300-400 12 19+ x
PE PQ
From eq. (1) and eq. (2), 400-500 17 36+ x
AB = BC = AC = AD 500-600 20 56+ x
...(3)
PQ QR PR PE
600-700 y 56+ x+y
ar ^TABC h 1
# BC # AD
Now, = 2 700-800 9 65+ x+y
ar ^TPQRh 1
# QR # PE
2
800-900 7 72+ x+y
= c BC m # b AD l = BC # BC
QR PE QR QR 900-1000 4 76+ x+y
From equation (3) we have N = 100
ar ^TABC h 2 Here, 76 + x + y = 100
= BC2 ...(4)
ar ^TPQRh QR & x + y = 100 − 76 = 24 ...(i)
From equation (3) and equation (4) we have Given, Median = 525 , which lies between class 500-600
ar ^TABC h 2 2 2
Median class = 500 − 600
= c AB m = c BC m = b AC l &
ar ^TPQRh PQ QR PR n − c.f.
Now, Median = l + 2 #h
39. Evaluate : cos 65º − tan 20º − sin 90º + tan 5º tan 35º f
sin 25º cot 70º
^ hH
100 − 36 + x
tan 60º tan 55º tan 85º. [4] & 525 = 500 + > 2 # 100
20
Ans :
& 25 = ^50 − 36 − x h 5
cos 65º = cos 65º = cos 65º = 1, ^14 − x h = 5 = 5
We have 25
sin 25º sin ^90º − 65ºh cos 65º &

tan 20º = tan ^90º − 70ºh = cot 70° = 1 & x = 14 − 5 = 9


cot 70º cot 70º cot 70º
Substituting the value of x in equation (i),
and sin 90º = 1 y = 24 − 9 = 15
tan 5º tan 35º tan 60º tan 55º tan 85º Hence, x = 9 and y = 15
= tan ^90º − 85ºh tan ^90º − 55ºh WWW.CBSE.ONLINE

To Get 20 Solved Paper Free PDF by whatsapp add +91 89056 29969 in your class Group Page 9
Mathematics Standard X Solved Sample Paper 6 www.rava.org.in

CLASS X (2019-20)
MATHEMATICS STANDARD(041)
SAMPLE PAPER-6

Time : 3 Hours Maximum Marks : 80


General Instructions :
(i) All questions are compulsory.
(ii) The questions paper consists of 40 questions divided into 4 sections A, B, C and D.
(iii) Section A comprises of 20 questions of 1 mark each. Section B comprises of 6 questions of 2 marks each. Section
C comprises of 8 questions of 3 marks each. Section D comprises of 6 questions of 4 marks each.
(iv) There is no overall choice. However, an internal choices have been provided in two questions of 1 mark each, two
questions of 2 marks each, three questions of 3 marks each, and three questions of 4 marks each. You have to
attempt only one of the alternatives in all such questions.
(v) Use of calculators is not permitted.

Section A Uprate = 9 ' 6 = 1.5 km/hr


Speed of the boat = (4.5 + 1.5) ' 2 = 3 km/hr
Q.1-Q.10 are multiple choice questions. Select the Speed of the current = (4.5 − 1.5) ' 2 = 1.5 km/hr
most appropriate answer from the given options. 4. One of the two students, while solving a quadratic
1. Which of the following will have a terminating decimal equation in x , copied the constant term incorrectly
expansion? [1] and got the roots 3 and 2. The other copied the
(a) 77 (b) 23 constant term and coefficient of x2 correctly as − 6
210 30 and 1 respectively. The correct roots are [1]
(c) 125 (d) 23 (a) 3, − 2 (b) − 3, 2
441 8
23 (c) − 6, − 1 (d) 6, − 1
Ans : (d)
8 Ans : (d) 6, − 1
For terminating decimal expansion, denominator must Let α, β be the roots of the equation.
have only 2 or only 5 or 2 and 5 as factor. Then, α+β = 5
Here, 23 = 23
8 and αβ = − 6 .
(2) 3
So, the equation is
(only 2 as factor of denominator so terminating)
x2 − 5x − 6 = 0
2. The value of the polynomial x8 − x5 + x2 − x + 1 is [1]
The roots of the equation are 6 and –1.
(a) positive for all the real numbers
(b) negative for all the real numbers 5. Five distinct positive integers are in a arithmetic
progression with a positive common difference. If their
(c) 0
sum is 10020, then the smallest possible value of the
(d) depends on value of x last term is [1]
Ans : (a) positive for all the real numbers (a) 2002 (b) 2004
(c) 2006 (d) 2007
Let f ^x h = x 8 − x 5 + x 2 − x + 1
Ans : (c) 2006
For x = 1 or 0
Let the five integers be a − 2d , a − d , a , a + d , a + 2d.
f ^x h = 1 > 0
Then, we have,
For x <0 ^a − 2d h + ^a − d h + a + ^a + d h + ^a + 2d h
each term of f ^xh is Positive and so first f ^xh > 0 . = 10020

Hence, f ^xh is Positive for all real x. 5a = 10020 & a = 2004


Now, as smallest possible value of d is 1.
3. A motor boat takes 2 hours to travel a distance 9 km. Hence, the smallest possible value of a + 2d is 2004+2
down the current and it takes 6 hours to travel the = 2006
same distance against the current. The speed of the
boat in still water and that of the current (in km/ 6. If x sin3 θ + y cos3 θ = sin θ cos θ and x sin θ = y cos θ ,
hour) respectively are [1] than x2 + y2 is equal to [1]
(a) 3, 1.5 (b) 3, 2 (a) 0 (b) 1/2
(c) 3.5, 2.5 (d) 3, 1 (c) 1 (d) 3/2

Ans : (a) 3, 1.5 Ans : (c) 1

Down-rate = 9 ' 2 = 4.5 km/hr We have, x sin3 θ + y cos3 θ = sin θ cos θ

To Get 20 Solved Paper Free PDF by whatsapp add +91 89056 29969 in your class Group Page 1
Mathematics Standard X Solved Sample Paper 6 www.cbse.online

^x sin θh sin2 θ + ^y cos θh cos2 θ = sin θ cos θ of getting at most one head is [1]
x sin θ ^sin2 θh + ^x sin θh cos2 θ = sin θ cos θ (a) 1
4
(b) 1
2
x sin θ ^sin2 θ + cos2 θh = sin θ cos θ
(c) 3 (d) 1
x sin θ = sin θ cos θ & x = cos θ 4
Ans : (c) 3
Now, x sin θ = y cos θ 4
cos θ sin θ = y cos θ Total outcomes = HH, HT, TH, TT
y = sin θ Favourable outcomes = HT, TH, TT
Hence, x + y = cos θ + sin θ = 1
2 2 2 2
P (at most one head) = 3
4
7. If the area of a semi-circular field is 15400 sq m, then (Q.11-Q.15) Fill in the blanks.
perimeter of the field is: [1]
(a) 160 2 m (b) 260 2 m 11. An algorithm which is used to find HCF of two positive
numbers is .......... [1]
(c) 360 2 m (d) 460 2 m
Ans : Euclid’s division algorithm
Ans : (c) 360 2 m
Let the radius of the field be r . 12. The fourth vertex D of a parallelogram ABCD whose
three vertices are A ^− 2, 5h , B ^6, 9h and C ^8, 5h is
Then, πr2 = 15400 .......... [1]
2
Ans : ^0, 1h
1 22 r2 = 15400
2# 7 #
or
r2 = 15400 # 2 # 7
22 (5, − 2) (6, 4) and (7, − 2) are the vertices of an ..........
= 9800
triangle.
r = 70 2 m Ans : Isosceles
Thus, perimeter of the field
= πr + 2r 13. In TPQR , right-angled at Q , PR + QR = 25 cm and
PQ = 5 cm . The value of tan P is .......... [1]
= 22 # 70 2 + 2 # 70 # 2 Ans : 12/5
7
= 220 2 + 140 2 14. The region enclosed by an arc and a chord is called
= 2 ^220 + 140h the ......... of the circle. [1]
Ans : Segment
= 360 2 m
8. If the perimeter of one face of a cube is 20 cm, then its 15. The total surface area of a solid hemisphere having
surface area is [1] radius r is .......... [1]
(a) 120 cm2 (b) 150 cm2 Ans : 3πr 2

(c) 125 cm2 (d) 400 cm2


(Q.16-Q.20) Answer the following
Ans : (b) 150 cm2
16. If ratio of corresponding sides of two similar triangles
Edge of cube = 20 cm = 5 cm is 5 : 6, then find ratio of their areas. [1]
4
Ans :
Surface area = 6 # 52 cm2 = 150 cm2 Let the triangles be TABC and TDEF
9. The median of a set of 9 distinct observations is ar ^TABC h 2
20.5. If each of the largest 4 observation of the set is = b 5 l = 25
ar ^TDEF h 6 36
increased by 2, then the median of the new set [1]
(a) Is increased by 2 25 : 36
(b) Is decreased by 2 17. Two concentric circles are of radii 5 cm and 3 cm.
(c) Is two times the original median Find the length of the chord of larger circle (in cm)
(d) Remains the same as that of the original set which touches the smaller circle. [1]
Ans :
Ans : (d) Remains the same as that of the original set
As per the given question we draw the figure as below.
Since, n =9

median term = b 9 + 1 l = 5 th item.


th
then,
2
Now, last four observations are increased by 2.
The median is 5th observation, which is remaining
unchanged.
There will be no change in median.
10. Two coins are tossed simultaneously. The probability

Download 20 Solved Sample Papers pdfs from www.cbse.online or www.rava.org.in Page 2


Mathematics Standard X Solved Sample Paper 6 www.rava.org.in

Here AB is the chord of large circle which touch


the smaller circle at point C . We can see easily that θ = 45c
TAOC is right angled triangle. Hence angle of elevation is 45c
Here, AO = 5 cm, OC = 3 cm
19. A line Segment AB is divided at point P such that
AC = AO2 − OC2 PB = 3 , then find the ratio AP | PB . [1]
AB 7
= 52 − 32
Ans :
= 25 − 9 = 16 = 4 cm
Length of chord, AB = 8 cm. Here, AB = 7, PB = 3
18. A pole casts a shadow of length 2 3 m on the ground, ` AP = AB − PB = 7 − 3 = 4
when the Sun’s elevation is 60º. Find the height of the ` AP | PB = 4 | 3
pole. [1]
20. If the radius of the base of a right circular cylinder
Ans : is halved, keeping the height same, find the ratio of
Let the height of pole be h. As per given in question the volume of the reduced cylinder to that of original
we have drawn figure below. cylinder. [1]
Ans :

Volume of reduced cylinder π # ^ r2 h2 h


=
Volume of original cylinder πr 2 h
= 1 = 1|4
4

Section B
21. For what value of 'k' , the system of equations
kx + 3y = 1, 12x + ky = 2 has no solution. [2]
h = tan 60º Ans :
2 3 The given equations can be written as
h = 2 3 tan 60º kx + 3y − 1 = 0 and 12x + ky − 2 = 0

=2 3# 3 =6 m Here, a1 = k, b1 = 3, c1 = − 1
and a2 = 12, b2 = k, c2 = − 2
or
The equation for no solution if
An observer 1.5 m tall is 28.5 m away from a tower 30 a1 = b1 ! c1
m high. Find the angle of elevation of the top of the a2 b2 c2
tower from his eye.
or, k = 3 ! −1
Ans : 12 k −2
As per given in question we have drawn figure below. From k = 3 we have k2 = 36 & k ! 6
12 k

From 3 ! − 1 we have k ! 6
k −2
Thus k = − 6
22. Prove that the point ^3, 0h , ^6, 4h and ^− 1, 3h are the
vertices of a right angled isosceles triangle. [2]
Ans :
We have A ^3, 0h , B ^6, 4h and C ^− 1, 3h
Now AB2 = ^3 − 6h2 + ^0 − 4h2
= 9 + 16 = 25
BC = ^6 + 1h2 + ^4 − 3h2
2

= 49 + 1 = 50
CA = ^− 1 − 3h2 + ^3 − 0h2
2

Here AE = 1.5 m is height of observer and BD = 30 = 16 + 9 = 25


m is tower. AB = CA2 or, AB = CA
2

Now BC = 30 − 1.5 = 28.5 m Hence triangle is isosceles.


In TBAC, tan θ = BC
AC

tan θ = 28.5 = 1 = tan 45c


28.5
To Get 20 Solved Paper Free PDF by whatsapp add +91 89056 29969 in your class Group Page 3
Mathematics Standard X Solved Sample Paper 6 www.cbse.online

Also, 25 + 25 = 50
Now BD = BC = 24 = 12 cm
or, AB + CA2 = BC2
2 2 2
Since pythagoras theorem is verified, therefore triangle AB = 24 cm
is a right angled triangle.
AD = AB2 − BD2
23. In the given figure, DE | | BC. If AD = 1.5 cm
= ^24h2 − ^12h2
ar ^TADE h
BD = 2AD, then find [2] = 576 − 144
ar ^trapezium BCED h
= 432 = 12 3
Thus AD = 12 3 cm
` The length of the altitude is 12 3 cm.
24. The radius and height of a wax made cylinder are
6 cm and 12 cm respectively. A cone of same base
radius and height has been made from this cylinder
by cutting out. [2]
(a) Find the volume of cone
(b) How many candles with 1 cm radius and 12 cm
height can be made using the remaining wax.
Ans : Ans :

We have AD = 1.5 cm, BD = 3 Volume of the cone = 1 π # ^6 h # 12


2
(a)
3
and AB = AD + BD = 1.5 + 3.0 = 4.5 cm
= 144π cubic centimetre
In triangle ADE and ABC, +A is common and
DE | | BC (b) Volume of the cylinder = πr2 h = 144π # 3

Thus +ADE = +ABC = 432π cubic centimetre

+AED = +ACB Volume of the remaining wax = 288π cubic centimetre

(corresponding angles) Volume of one candle = π # 12 # 12

By AA similarity we have = 12π cubic centimetres

TADE ~TABC Number of candles = 288π = 24


12π
ar ^TADE h 2 ^1.5h2 1 25. A cubical block of side 7 cm is surmounted by a
Now = AD2 = =
ar ^TABC h AB ^4.5h2 9 hemisphere. What is the greatest diameter the
hemisphere can have ? Find the surface area of the
ar ^TADE h
= 1 solid. [2]
ar ^TABC h − ar ^TADE h 9−1
Ans :
ar ^TADE h
=1
ar ^trapezium BCED h 8 Diameter of hemisphere = Side of cubical block
or 2r = 7
In an equilateral triangle of side 24 cm, find the length or, r =7
2
of the altitude.
Surface area of solid
Ans :
= Surface area of the cube
Let TABC be an equilateral triangle of side 24 cm and
− Area of base of hemisphere
AD is altitude which is also a perpendicular bisector
of side BC. This is shown in figure given below. + curved surface area of hemisphere
= 6l2 − πr2 + 2πr2
= 6 # 49 − 11 # 7 + 77 = 332.5 cm2
2
or

Download 20 Solved Sample Papers pdfs from www.cbse.online or www.rava.org.in Page 4


Mathematics Standard X Solved Sample Paper 6 www.rava.org.in

A metallic solid sphere of radius 4.2 cm is melted and Thus 5 = 10 ...(1)


recast into the shape of a solid cylinder of radius 6 a
cm. Find the height of the cylinder. c = 10
and ...(2)
Ans : a
Dividing (2) by eq. (1) we have
Volume of sphere = Volume of cylinder
c =1 &c=5
4 πR 3 = πr 2 h 5
3
Substituting c = 5 in (2) we get a = 1
2

3 #^ h
4 4.2 3 = 62 # h Hence a = 1
and c = 5 .
2

or
h = 4 # 4.2 # 4.2 # 4.2
3#6#6
If α and β are the zeroes of a quadratic polynomial
Hence, height of cylinder h = 2.744 cm. such that α + β = 0 and α − β = 8 . Find the quadratic
26. There are two covers A and B each containing paper polynomial having α and β as its zeroes.
slips with natural numbers from 1 to 7 written on Ans :
them. One slip is drawn from each cover. Using them,
We have α + β = 24 ...(1)
a two digit number is formed with a number from
A in the units place and the number from B in the α−β = 8 ...(2)
tens place. How many such two digit numbers can Adding equations (1) and (2) we have
be formed? What is the probability that a two digit
2α = 32 & α = 16
number so formed is even? [2]
Ans : Subtracting (1) from (2) we have
2β = 24 & β = 12
Number of slips in cover A = 7
Hence, the quadratic polynomial
Number of slips in cover B = 7
Numbers formed with a number from cover A in the p (x) = x2 − ^α + βh x + αβ
units place and a number from cover B in the tens = x2 − ^16 + 8h x + ^16h^8h
place are as follows: = x2 − 24x + 128
11 12 13 14 15 16 17 28. Determine an A.P. whose third term is 9 and when
31 22 23 24 25 26 27 fifth term is subtracted from 8th term, we get 6. [3]
Ans :
31 32 33 34 35 36 37
Let the first term be a , common difference be d and
41 42 43 45 45 46 47
n th term be an .
51 52 53 54 55 56 57
We have a3 = 9
61 62 63 64 65 66 67 a + 2d = 9 ...(1)
71 72 73 74 75 76 77 and a8 − a5 = 6
Thus, total number of two digit numbers
^a + 7d h − ^a + 4d h = 6
= 7 # 7 = 49
Number of two digit even numbers 3d = 6

= 7 # 3 = 21 d =2

P (two digit number so formed is even) Substituting this value of d in equation (1), we get
a + 2 ^2 h = 9
= 21 = 3
49 7 a =5

Section C So, A.P. is 5, 7, 9, 11, ...


29. Find the co-ordinate of a point P on the line segment
27. If the sum and product of the zeroes of the polynomial joining A ^1, 2h and B ^6, 7h such that AP = 25 AB [3]
ax2 − 5x + c are equal to 10 each, find the value of 'a' Ans :
and 'c' . [3]
As per question, line diagram is shown below.
Ans :

We have f ^x h = ax2 − 5x + c
Let the zeroes of f ^x h be α and β , then,
Sum of zeroes α + β =−−5 = 5 We have AP = 2 AB & AP: PB = 2: 3
a a 5
By using section formula,
Product of zeroes αβ = c my2 + nx1
a x = mx2 + nx1 and y =
m+n m+n
According to question, the sum and product of the
zeroes of the polynomial f ^x h are equal to 10 each. Applying section formula we get

To Get 20 Solved Paper Free PDF by whatsapp add +91 89056 29969 in your class Group Page 5
Mathematics Standard X Solved Sample Paper 6 www.cbse.online

x = 2 # 6 + 3 # 1 = 12 + 3 = 3 an elevation of 70c from the other bank. Stepping 20


2+3 5 metres back, he sees the top of the tree at an elevation
of 55c. Height of the person is 1.4 metres. [3]
and y = 2 # 7 + 3 # 2 = 14 + 6 = 4
2+3 5 (a) Draw a rough figure and mark the measurements.
(b) Find the height of the tree.
Thus P ^x, y h = ^3, 4h
(c) Find the width of the river.
or [ tan 70c = 2.75 ; tan 55c = 1.43 ]
Ans :
Find the ratio in which the line segment joining the
points A ^3, − 3h and B ^− 2, 7h is divided by x-axis. (a)
Also find the co-ordinates of point of division.
Ans :
y co-ordinate of any point on the x will be zero. Let
(x, 0) be point on x axis which cut the line. As per
question, line diagram is shown below.

Let the ratio be k : 1.


Using section formula for y co-ordinate we have
1 ^− 3h + k ^7 h
0 = (b) In TABC ,
1+k
tan 70c = AB
k =3 x
7 AB = x tan 70c
Using section formula for x co-ordinate we have (c)
1 ^3 h + k ^− 2h 3 − 2 # 73
x = = =3 In TABD , tan 55c = AB
1+k 1 + 73 2 x + 20
AB = ^x + 20h tan 55c
Thus co-ordinates of point are ^ 32 , 0h .
x tan 70c = ^x + 20h tan 55c
30. ABC is a triangle. A circle touches sides AB and AC
produced and side BC at BC at X , X,Y and Z x tan 70c = x tan 55c + 20 tan 55c
respectively. Show that AX = 12 perimeter of TABC . x ^tan 70c − tan 55ch = 20 tan 55c
[3]
Ans : x = 20 # tan 55c
tan 75c − tan 55c
As per question we draw figure shown below.
= 20 # 1.43 = 28.6
2.75 − 1.43 1.32
= 21.67
AB = x tan 70 = 21.67 # 2.75
= 59.59 m
Height of the tree = 59.59 + 1.4
= 60.99 m
Width of the river = 21.67 m
32. In the given figure, AOB is a sector of angle 60c of a
Since length of tangents from an external point to a circle with centre O and radius 17 cm. If AP = OB and
circle are equal, AP = 15 cm, find the area of the shaded region. [3]
At A, AX = AY (1)
At B BX = BZ cm (2)
At C CY = CZ (3)
Perimeter of TABC ,
p = AB + AC + BC
= (AX − BX) + (AY − CY) + (BZ + ZC)
= AX + AY − BX + BZ + ZC − CY
From eq. (1), (2) and (3), we get Ans :
= AX + AY = 2AX Here OA = 17 cm, AP = 15 cm and ΔOPA is right
Thus AX = 1 p Hence Proved triangle
2
31. One sees the top of a tree on the bank of a river at Using Pythagoras theorem, we have

Download 20 Solved Sample Papers pdfs from www.cbse.online or www.rava.org.in Page 6


Mathematics Standard X Solved Sample Paper 6 www.rava.org.in

OP = 172 − 152 = 8 cm x = tan 45º = 1 & x = y


y
Area of the shaded region
x + 7 = tan 60º = 3
= Area of the sector TOAB x
– Area of ΔOPA 7 = ^ 3 − 1h x
= 60 # πr2 − 1 # b # h
360 2 7 ^ 3 + 1h 7 (2.73)
x = = = 9.6 m
2 2
= 60c # 22 # 17 # 17 − 1 # 8 # 15 34. From the top of a tower of height 50 cm, the angles
360c 7 2
of depression of the top and bottom of a pole are 30c
= 151.38 − 60 = 91.38 cm2 and 45c respectively find: [3]
(i) How far the pole is from the bottom of a tower?
or (ii) The height of the pole (Use 3 = 1.732 )
A memento is made as shown in the figure. Its base Ans :
PBCR is silver plate from the Front side. Find the
area which is silver plated. Use π = 227 . Here, AB = 50 m
+ADB = 45c
+ACM = 30c
(i) In right TABD ,
tan 45c = AB = 1
BD
BD = AB = 50 m

Ans :
From the given figure
Area of right-angled ΔABC = 1 # 10 # 10 = 50
2
Area of quadrant APR of the circle of radii 7 cm
= 1 # π # ^7 h2
4

= 1 # 22 # 49 = 38.5 cm2
4 7 Distance of pole from bottom of water = 50 m
(ii) In right TAMC ,
Area of base PBCR
= Area of ΔABC − Area of quadrant APR AM = MC tan 30c = 50
3
= 50 − 38.5 = 11.5 cm2 Height of pole = CD = BM = AB − AM
33. A 7m long flagstaff is fixed on the top of a tower
standing on the horizontal plane. From point on the = 50 − 50 = 50 − 50
3 1.732
ground, the angles of elevation of the top and bottom
= 50 − 28.87 = 21.13 m
of the flagstaff are 60º and 45º respectively. Find the
height of the tower correct to one place of decimal.
(Use 3 = 1.73) [3] Section D
Ans :
35. Solve for x and y : [4]
As per given in question we have drawn figure below.
2x − y + 3 = 0
3x − 5y + 1 = 0
Ans :

We have 2x − y + 3 = 0 ...(1)
3x − 5y + 1 = 0 ...(2)
Multiplying equation (1) by 5,and subtracting (2)
from it we have
7x = − 14
x = − 14 = − 2
7
Substituting the value of x in equation (1), we get

To Get 20 Solved Paper Free PDF by whatsapp add +91 89056 29969 in your class Group Page 7
Mathematics Standard X Solved Sample Paper 6 www.cbse.online

2x − y + 3 = 0 is 34 , find the fraction. [4]


15
2 ^− 2h − y + 3 = 0 Ans :
−4 − y + 3 = 0 Let numerator be x , then denominator will be x + 2 .
−y − 1 = 0
and fraction = x
y =− 1 x+2

Hence, x = − 2 and y = − 1. Now x + x + 2 = 34


x+2 x 15
or 15 ^x2 + x2 + 4x + 4h = 34 ^x2 + 2x h
A two digit number is obtained by either multiplying 30x2 + 60x + 60 = 34x2 + 68x
the sum of digits by 8 and then subtracting 5 or by
4x2 + 8x − 60 = 0
multiplying the difference of digits by 16 and adding
3. Find the number. x2 + 2x − 15 = 0
Ans : x2 + 5x − 3x − 15 = 0
Let the digits of number be x and y , then number x ^x + 5h − 3 ^x + 5h = 0
will 10x + y
^x + 5h^x − 3h = 0
According to the question, we have
We reject the x = − 5 . Thus x = 3 and fraction = 3
8 ^x + y h − 5 = 10x + y 5
2x − 7y + 5 = 0 ...(1) or
also 16 ^x − y h + 3 = 10x + y A motor boat whose speed is 24 km/h in still water
6x − 17y + 3 = 0 ...(2) takes 1 hour more to go 32 km upstream than to
return downstream to the same spot. Find the speed
Comparing the equation with ax + by + c = 0 we get of the stream.
a1 = 2, b1 = − 1, c1 = 5 Ans :
a2 = 6, b2 = − 17, c2 = 3 Let the speed of stream be x km/h
x y 1 Then the speed of boat upstream = ^24 − x h km/h
Now = =
b2 c1 − b1 c2 c1 a2 − c2 a1 c1 b2 − a2 b1 Speed of boat downstream = ^24 + x h km/h
x y According to the question,
=
^ h^ h ^
− 7 3 − − 17 h^ h
5 ^ h^ h ^2 h^3 h
5 6 − 32 − 32 = 1
24 − x 24 + x
= 1
^2h^− 17h − ^6 h^− 7h 32 :
24 − x 24 + x D
1 − 1 =1
x y 1
= =
− 21 + 85 30 − 6 − 34 + 42 32 :24 + x − 242+ x D = 1
576 − x
x = y =1
64 24 8 32 ^24 + x − 24 + x h = 576 − x2
x =y =1 64x = 576 − x2
8 3 x2 + 64x − 576 = 0
Hence, x = 8, y = 3 x2 + 72x − 8x − 576 = 0
So, required number = 10 # 8 + 3 = 83 . x ^x + 72h − 8 ^x + 72h = 0
36. Find the HCF of 256 and 36 using Euclid’s Division ^x − 8h^x + 72h = 0
Algorithm. Also, find their LCM and verify that HCF
x = 8, − 72
× LCM = Product of the two numbers. [4]
Ans : Since speed cannot be negative, we reject x = − 72 .

By using Euclid’s Division Lemma, we have The speed of steam is 8 km/h.

256 = 36×7 + 4 38. ABCD is a rhombus whose diagonal AC makes an


angle α with AB. If cos α = 23 and OB = 3 cm, find
36 = 4×9 + 0 the length of its diagonals AC and BD . [4]
Hence, the HCF of 256 and 36 is 4.
LCM : 256 = 28
36 = 22 ×32
LCM (36, 256) = 28 ×32 = 256×9 = 2304
HCF × LCM = Product of the two number
4×2, 304 = 256×36
9216 = 9, 216 Hence verified.
37. The denominator of a fraction is two more than its
numerator. If the sum of the fraction and its reciprocal
Download 20 Solved Sample Papers pdfs from www.cbse.online or www.rava.org.in Page 8
Mathematics Standard X Solved Sample Paper 6 www.rava.org.in

Ans : Due to AA similarity,


TABC ~TPQR
We have cos α = 2 and OB = 3 cm
3 ar ^TABC h 2
Now = AD2
ar ^TPQRh PE
In TAOB, cos α = 2 = AO
3 AB
16 = AD2
Let OA = 2x then AB = 3x 25 PE2
Now in right angled triangle TAOB we have 4 = AD
5 PE
AB2 = AO2 + OB2
AD = 4
^3x h2 = ^2x h2 + ^3 h2 PE 5
9x2 = 4x2 + 9
39. In an acute angled triangle ABC, if sin ^A + B − C h = 1
5x2 = 9 2
and cos ^B + C − Ah = 1 , find +A, +B and +C .[4]
x = 9 = 3 2
5 Ans :
5
Hence, OA = 2x = 2 c 3
m = 6 cm We have sin ^A + B − C h = 1 = sin 30º
5 5 2

AB = 3x = 3 c 3 9 or, A + B − C = 30º ...(1)


5m
and = cm
5
and cos ^B + C − Ah = 1 = cos 45º
Diagonal BD = 2 # OB = 2 # 3 = 6 cm 2
and AC = 2AO or, B + C − A = 45º ...(2)
= 2 # 6 = 12 cm Adding equation (1) and (2), we get
5 5 2B = 75º
or
or, B = 37.5º
Vertical angles of two isosceles triangles are equal. If Now subtracting equation (2) from equation (1) we
their areas are in the ratio 16 : 25, then find the ratio get,
of their altitudes drawn from vertex to the opposite
side. 2 ^A − C h = − 15º
Ans : or, A − C = 7.5º ...(3)
As per given condition we have drawn the figure Now A + B + C = 180º
below. A + B + C = 180º
A + C = 180c − 37.5c = 142.5º ...(4)
Adding equation (3) and (4), we have
2A = 135º
or, A = 67.5º
and, C = 75º
Hence, +A = 67.5º, +B = 37.5º, +C = 75º

Here +A = +P 40. Find the median of the following data :


+B = +C, +Q = +R Class 0- 20- 40- 60- 80- 100- 120-
Interval 20 40 60 80 100 120 140
Let +A = +P be x .
Frequency 6 8 10 12 6 5 3
In TABC, +A + +B + +C = 180º
x2 + +B + +B = 180º (+B = +C ) How can we find the median graphically ? [4]
2+B = 180º − x Ans :
(i)
+B = 180º − x ...(1)
2
Classes c.f.
Now, in TPQR
+P + +Q + +R = 180º (+Q = +R ) More than 0 50

x2 + +Q + +Q = 180º More than 20 44


2+Q = 180º − x More than 40 36

+Q = 180º − x ...(2) More than 60 26


2
In TABC and TPQR, More than 80 14
+A = +P [Given] More than 100 8
+B = +Q [From eq. (1) and (2)] More than 120 3

To Get 20 Solved Paper Free PDF by whatsapp add +91 89056 29969 in your class Group Page 9
Mathematics Standard X Solved Sample Paper 6 www.cbse.online

To draw on ogive we take the indeces : (0,50),(20, 44),


(40, 36), (60, 26), (80, 14), (100, 8), (120, 3)

From graph, N = 50 = 25
2 2
` Median = 61.6
(ii) By Formula Method :
Classes f c.f.
0-20 6 6
20-40 8 14
40-60 10 24
60-80 12 36 Median Class
80-100 6 42
100-120 5 47
120-140 3 50
Median = N th term
2

= 50 = 25 th term
2
` Median class = 60 − 80
N − c.f.
Median = l + f 2 p# h
f

= 60 + 1 # 20
12

= 60 + 5
3

= 185
3
= 61.67
WWW.CBSE.ONLINE

Download unsolved version of this paper from


www.cbse.online

This sample paper has been released by website www.cbse.online for the benefits of the students. This paper has been prepared by
subject expert with the consultation of many other expert and paper is fully based on the exam pattern for 2019-2020. Please note
that website www.cbse.online is not affiliated to Central board of Secondary Education, Delhi in any manner. The aim of website is
to provide free study material to the students.
Download 20 Solved Sample Papers pdfs from www.cbse.online or www.rava.org.in Page 10
Mathematics Standard X Solved Sample Paper 7 www.rava.org.in

CLASS X (2019-20)
MATHEMATICS STANDARD(041)
SAMPLE PAPER-7

Time : 3 Hours Maximum Marks : 80


General Instructions :
(i) All questions are compulsory.
(ii) The questions paper consists of 40 questions divided into 4 sections A, B, C and D.
(iii) Section A comprises of 20 questions of 1 mark each. Section B comprises of 6 questions of 2 marks each. Section
C comprises of 8 questions of 3 marks each. Section D comprises of 6 questions of 4 marks each.
(iv) There is no overall choice. However, an internal choices have been provided in two questions of 1 mark each, two
questions of 2 marks each, three questions of 3 marks each, and three questions of 4 marks each. You have to
attempt only one of the alternatives in all such questions.
(v) Use of calculators is not permitted.

Then the present age of father (in years) is [1]


Section A (a) 42 (b) 56
(c) 70 (d) 77
Q.1-Q.10 are multiple choice questions. Select the
most appropriate answer from the given options. Ans : (c) 70
Let the ages of father and son be 7x , 3x
The value of ^12h3 + ^18h3 , x dN , end with the
x x
1.
digit. [1] Hence, (7x + 10) : (3x + 10) = 2 : 1
(a) 2 7x + 10 = 6x + 20
(b) 8 7x − 6x = 20 − 10
(c) 0
or x = 10
(d) Cannot be determined Age of the father is 70 years.
Ans : (c) 0 4. Each root of x2 − bx + c = 0 is decreased by 2. The
For all x d N , ^12h ends with either 8 or 2 and ^18h resulting equation is x2 − 2x + 1 = 0 , then
3x
3x [1]
ends with either 2 or 8. (a) b = 6, c = 9 (b) b = 3, c = 5
If ^12h3 ends with 8, then ^18h ends with 2.
3x
(c) b = 2, c = − 1 (d) b = − 4, c = 3
x

If ^12h3 ends with 2, then ^18h ends with 8.


x 3x
Ans : (a) b = 6, c = 9
Thus, ^12h3 + ^18h3 ends with 0 only.
x x

α+β = b
2. On dividing x3 − 3x2 + x + 2 by a polynomial g ^x h ,
the quotient and remainder were x − 2 and − 2x + 4 αβ = c
respectively, then g ^x h is equal to [1] According to the question
(a) x2 + x + 1 (b) x2 + 1 ^α + β − 4h = b − 4
(c) x2 − x + 1 (d) x2 − 1 ^α − 2h^β − 2h = αβ − 2 ^α + βh + 4
Ans : (c) x2 − x + 1 = c − 2b + 4
Here, Dividend = x3 − 3x2 + x + 2 Now 2 = b−4
Quotient = x − 2 b =6
Remainder = − 2x + 4 and 1 = c − 2b + 4
Divisor = g ^x h 1 = c − 2 # 6 + 4 = c − 12 + 4
Since, c = 1 + 12 − 4 = 9
dividend = Divisor # Quotient +Remainder 5. What is the common difference of four terms in A.P.
such that the ratio of the product of the first fourth
So, x − 3x + x + 2 = g ^x h # ^x − 2h + ^− 2x + 4h
3 2
term to that of the second and third term is 2 : 3 and
g ^x h # ^x − 2h = x3 − 3x2 + x + 2 + 2x − 4 the sum of all four terms is 20? [1]
(a) 3 (b) 1
g ^x h = x − 3x + 3x − 2
3 2

x−2 (c) 4 (d) 2


^x − 2h^x2 − x + 1h Ans : (d) 2
=
^x − 2h Take the four terms as a − 3x , a − x , a + x , a + 3x
= x −x+1
2
The sum = 4a = 20
3. At present ages of a father and his son are in the ratio a =5
7 : 3, and they will be in the ratio 2 : 1 after 10 years.

To Get 20 Solved Paper Free PDF by whatsapp add +91 89056 29969 in your class Group Page 1
Mathematics Standard X Solved Sample Paper 7 www.cbse.online

Also, 3 (a2 − (3x) 2) = 2 (a2 − x2) x+y


Hence, = 10
2
x =1
However, the common difference is 2x and not x x + y = 20 ...(1)
When, x = 1, d = 2x = 2 and x − y = 6 (width of the class = 6 )..(2)
6. The ratio in which the point (2, y) divides the join of By solving (1) and (2), we get y = 7
(− 4, 3) and (6, 3). The value of y is [1] Hence, lower limit of the class is 7.
(a) 2 : 3, y = 3 (b) 3 : 2, y = 4
10. The probability of getting a number greater than 2 in
(c) 3 : 2, y = 3 (d) 3 : 2, y = 2 throwing a dice is [1]
Ans : (c) 3 : 2, y = 3 (a) 2/3 (b) 1/3
Let the required ratio be k : 1 (c) 4/3 (d) 1/4
6k − 4 (1) Ans : (a) 2/3
Then, 2 =
k+1
Required probability = 4 = 2
6 3
or k =3
2
(Q.11-Q.15) Fill in the blanks.
3
The required ratio is : 1 or 3 : 2
2 11. The ratio of the areas of two similar triangles is equal
3 (3) + 2 (3) to the square of the ratio of their .......... [1]
Also, y = =3 Ans : corresponding sides
3+2
7. If the angle of depression of an object from a 75 m
high tower is 30c, then the distance of the object from 12. Point (− 4, 6) divide the line segment joining the
the tower is [1] points A (− 6, 10) and B (3, − 8) in the ratio .......... [1]
(a) 25 3 m (b) 50 3 m Ans : 2 : 7
(c) 75 3 m (d) 150 m or
Ans : (c) 75 3 m All the points equidistant from two given points
A and B lie on the .......... of the line segment AB .
Ans : perpendicular bisector

13. It tan A = 4/3 then sin A .......... [1]


Ans : 4/5

14. A line that intersects a circle in one point only is


called .......... [1]
Ans : tangent

15. Two points on a line segment are marked such that


tan 30c = AB the three parts they make are equal then we say that
OB
the two points .......... the line segment. [1]
1 = 75
OB Ans : Trisect
3
OB = 75 3 m (Q.16-Q.20) Answer the following
8. Ratio of volumes of two cones with same radii is [1] 16. If the length of the ladder placed against a wall is
(a) h1 : h2 (b) s1 : s2 twice the distance between the foot of the ladder and
(c) r1 : r2 (d) None of these the wall. Find the angle made by the ladder with the
horizontal. [1]
Ans : (a) h1 : h2
Ans :
1 πr 2 h : 1 πr 2 h
3 1 1 3 2 2 Let the distance between the foot of the ladder and
the wall is x , then length of the ladder will be 2x . As
1 πr 2 h : 1 πr 2 h (r1 = r2) per given in question we have drawn figure below.
3 1 1 3 1 2
h1 : h2
9. In a frequency distribution, the mid value of a class is
10 and the width of the class is 6. The lower limit of
the class is [1]
(a) 6 (b) 7
(c) 8 (d) 12
Ans : (b) 7
Let x be the upper limit and y be the lower limit.
Since the mid value of the class is 10.
In TABC, +B = 90º
Download 20 Solved Sample Papers pdfs from www.cbse.online or www.rava.org.in Page 2
Mathematics Standard X Solved Sample Paper 7 www.rava.org.in

cos A = x median class : [1]


2x
Cost of living index 1400- 1550- 1700- 1 8 5 0 -
= 1 = cos 60º 1500 1700 1850 2000
2
Number of weeks 8 15 21 8
A = 60º
Ans :
17. What is the perimeter of the sector with radius 10.5
cm and sector angle 60º. [1] C.I. 1400- 1550- 1700- 150-2000
Ans : 1550 1700 1850
As per question the digram is shown below. f 8 15 21 8
c.f. 8 23 44 52
Σf
= 26 & Median class = 1700 − 1850
2
20. Out of 200 bulbs in a box, 12 bulbs are defective. One
bulb is taken out at random from the box. What is the
probability that the drawn bulb is not defective? [1]
Ans :
Perimeter of the sector,
Total No. of cases = 200
p = 2r + 2πrθ
360º Favourable cases = 200 − 12
= 10.5 # 2 + 2 # 22 # 10.5 # 60 = 188
7 360
= 21 + 11 = 32 cm Required probability = 188
200

18. Two cubes each of volume 8 cm3 are joined end to end, = 47
50
then what is the surface area of resulting cuboid. [1]
Ans : Section B
Given
21. Complete the following factor tree and find the
Side of the cube, a = 3 8 = 2 cm
composite number x [2]
Now the length of cuboid
l = 4 cm
Breadth, b = 2 cm
Height, h = 2 cm
Surface area of cuboid
= 2 ^l # b + b # h + h # l h
= 2 ^4 # 2 + 2 # 2 + 2 # 4h
= 2 # 20 = 40 cm2

or
A solid metallic object is shaped like a double cone as
shown in figure. Radius of base of both cones is same
but their heights are different. If this cone is immersed
in water, find the quantity of water it will displace.
Ans :

Volume of the upper cone = 1 πr2 h Ans :


3
We have z = 371 = 53
Volume of the lower cone = 1 πr2 H 7
3
y = 1855 # 3 = 5565
Total volume of both the cones = 1 πr2 h + 1 πr2 H
3 3 x = 2 # y = 2 # 5565 = 11130

= 1 πr2 ^h + H h Thus complete factor three is as given below.


3

The quantity of water displaced will 1 πr2 ^h + H h


3
cube units.
19. Find the following frequency distribution, find the

To Get 20 Solved Paper Free PDF by whatsapp add +91 89056 29969 in your class Group Page 3
Mathematics Standard X Solved Sample Paper 7 www.cbse.online

TABC ~TDEF
ar ^TABC h
= b AB l = ^3 h2 = 9
2

ar ^TDEF h DE
or
In the given figure, +A = +B and AD = BE. Show
that DE | | AB.

Ans :
22. If x = − 12 , is a solution of the quadratic equation
3x2 + 2kx − 3 = 0 , find the value of k . [2] In TCAB, we have
+A = +B (1)
Ans :
By isoscales triangle property, we have
We have 3x2 + 2kx − 3 = 0 AC = CB
Putting x = − 1 , we get
2 But, we have been given
2 AD = BE (2)
3 b− 1 l + 2k b− 1 l − 3 = 0
2 2
Dividing equation (2) by (1) we get,
3 −k−3 = 0 CD = CE
4 AD BE
k = 3 −3
4 By converse of BPT,
= 3 − 12 = − 9 DE | | AB. Hence Proved
4 4
24. Two slips of paper marked 5 and 10 are put in a box
Hence k = − 9
4 and three slips marked 1, 3, 5 are in another. One slip
from each box is drawn. [2]
23. The sides AB and AC and the perimeter P1 of TABC
(a) What is the probability that both show odd
are respectively three times the corresponding sides
number?
DE and DF and the parameter P2 of TDEF. Are the
(b) What is the probability of getting one odd number
ar ^TABC h and one even number?
two triangles similar? If yes, find [2]
ar ^TDEF h Ans :
Ans : One box contains (5,10)
As per given condition we have drawn the figure Other box contains (1, 3, 5)
below. (a) For I box probability for odd number 1
2
Fro II box probability for odd number = 3 = 1
3
1
Required probability = # 1 = 1
2 2
(b) P (one odd and one even)
= P (one odd from box I)
# P (one even from box II)
In TABC and TDEF , + P (one even from box I)

AB = 3DE # P (one odd from box II)


and AC = 3DF =1#0+1#3
2 3 2 3
AB = 3 ; AC = 3;
DE DF = 0+1 = 1
2 2
Since P1 = 3P2 , BC = 3EF 25. The data regarding marks obtained by 48 students
AB = AC = BC = 3 of a class in a class test is given below. Calculate the
Thus
DE DF EF
By SSS criterion we have
Download 20 Solved Sample Papers pdfs from www.cbse.online or www.rava.org.in Page 4
Mathematics Standard X Solved Sample Paper 7 www.rava.org.in

modal marks of students. [2] tan 30c = AB & 1 = 40


BC 3 BC
Marks 0-5 5- 10- 15- 20- 25- 30- 35- 40- 45-
obtained 10 15 20 25 30 35 40 45 50 BC = 40 3
Number of 1 0 2 0 0 10 25 7 2 1
students Again, in right TDCB ,
Ans : tan 60c = DC
BC
Modal class is 30-35, l = 30 , f1 = 25 f0 = 10 , f2 = 7
, h=5 3 = DC
40 3
f −f
Mode = l + c 1 0 m # h DC = 120 m
2f1 − f0 − f2
The height of chimney is 100 m,
& Mode = 30 + 25 − 10 5 Which is greater than the ideal height 100 m of a
50 − 10 − 7 # small emitting chimney.
= 30 + 2.27 or 32.27 approx.

or Section C
The following table gives the life time in days of 100 27. Find the HCF of 180, 252 and 324 by Euclid’s Division
bulbs : algorithm. [3]
Life Less Less Less Less Less Less Ans :
time in than than than than than than
We have 324 = 252×1 + 72
days 50 100 150 200 250 300
252 = 72×3 + 36
Number 8 23 55 81 93 100
of Bulbs 72 = 36×2 + 0
Change the above distribution as frequency Thus HCF (324, 252) = 36
distribution.
Now 180 = 36×5 + 0
Ans :
Thus HCF (36, 180) = 36
Frequency distribution table.
Thus HCF of 180, 252, and 324 is 36.
Class -Interval Frequency Hence required number = 999999 − 63 = 999936
0-50 8
or
50-100 15
100-150 32 144 cartons of Coke cans and 90 cartons of Pepsi cans
150-200 26 are to be stacked in a canteen. If each stack is of the
200-250 12 same height and if it equal contain cartons of the same
250-300 7 drink, what would be the greatest number of cartons
each stack would have?
Total 100
Ans :
26. The angle of elevation of the top of a chimney from
the foot of a tower is 60c and the angle of depression The required answer will be HCF of 144 and 90.
of the foot of the chimney from the top of the tower 144 = 2 4 ×32
is 30c. If the height of tower is 40 m, find the height 90 = 2×32 ×5
of smoke emitting chimney. According to pollution
control norms, the minimum height of a smoke HCF(144, 90) = 2×32 = 18
emitting chimney should be 100 m. What value is Thus each stack would have 18 cartons.
discussed in this problem? [2] 28. Solve for x : [3]
Ans : x + 1 + x − 2 = 4 − 2x + 3 ; x ! 1, − 2, 2
x−1 x+2 x−2
Ans :

We have x + 1 + x − 2 = 4 − 2x + 3
x−1 x+2 x−2
x2 + 3x + 2 + x2 − 3x + 2 = 4x − 8 − 2x − 3
x2 + x − 2 x−2
2x2 + 4 = 2x − 11
x +x−2
2 x−2
^2x + 4h^x − 2h = ^2x − 11h^x + x − 2h
2 2

5x2 + 19x − 30 = 0
Given AB = 40 m be the height of the tower and CD ^5x − 6h^x + 5h = 0
be the height of smoking chimney.
In right TABC , x = − 5, 6
5

To Get 20 Solved Paper Free PDF by whatsapp add +91 89056 29969 in your class Group Page 5
Mathematics Standard X Solved Sample Paper 7 www.cbse.online

29. The ninth term of an A.P. is equal to seven times the


second term and twelfth term exceeds five times the
third term by 2. Find the first term and the common
difference. [3]
Ans :
Let the first term be a , common difference be d and
n th term be an .
Now a9 = 7a2
a + 8d = 7 ^a + d h Here AB = CD and AB = EF
a + 8d = 7a + 7d Thus +CAB = 90º and +ABF = 90º
− 6a + d = 0 (1) Hence +CAB = +ABF
and a12 = 5a3 + 2 and +ABE = +BAD
Hence +CAB and +ABF also +ABE and +BAD
a + 11d = 5 ^a + 2d h + 2 are alternate interior angles.
a + 11d = 5a + 10d + 2 CD | | EF Hence Proved
− 4a + d = 2 ...(2) 31. Read the following, understand the mathematical idea
Subtracting (2) from (1), we get expressed in it answer the questions that follow:
− 2a = − 2 1,4,9,16, ......... are the square of the counting numbers.
The remainders got by dividing the square numbers
a =1 with natural numbers have a cyclic property. For
Substituting this value of a in (1) we get example, the remainders on dividing these numbers
−6 + d = 0 by 4 are tabulated here. [3]
d =6 Number 1 4 9 16 25 - - -
Hence first term is 1 and common difference is 6. Remainder 1 0 1 0 1 - - -
On dividing by 4 perfect squares leave only 0 and
or
1 as remainders. From this we can conclude that an
Find the 20 term of an A.P. whose 3rd term is 7 and
th
arithmetic sequence whose terms leaves remainder 2
the seventh term exceeds three times the 3rd term by on dividing by 4 do not have a perfect square.
2. Also find its nth term ^an h . (a) Which are the possible remainders on dividing
Ans : any number with 4?
Let the first term be a , common difference be d and (b) Which are the numbers we would not get on
n th term be an . dividing a perfect square by 4?
(c) What is the remainder that leaves on dividing the
We have a3 = a + 2d = 7 (1) terms of the arithmetic sequence 2,5,8,11, ........
a7 = 3a3 + 2 by 4?
a + 6d = 3 # 7 + 2 = 23 (2) Ans :
Solving (1) and (2) we have (a) Any number can be form of ^4d + r h
Where r = 0, 1, 2 and 3
4d = 16 & d = 4
When any number divided by 4 remainders are 0,
a + 8 = 7 & a =− 1 1, 2, and 3.
a20 = a + 19d = − 1 + 19 # 4 = 75 (b) A perfect square number divided by 4 leave the
remainder 0 and 1
a1 = a + ^n − 1h d = − 1 + 4n − 4
2 and 3 are not get as remainder when perfect
= 4n − 5. square number divided by 4.
Hence n term is 4n − 5
th (c) 2, 5, 8, 11 ......
remainders are 2, 1, 0, 3 .......
30. Prove that the tangents drawn at the ends of a
diameter of a circle are parallel. [3] 32. The angles of depression of the top and bottom of
Ans : a 50 m high building from the top of a tower are
45º and 60º respectively. Find the height of the tower
Let AB be a diameter of a given circle and let CD and and the horizontal distance between the tower and the
RF be the tangents drawn to the circle at A and B building. (Use 3 = 1.73) [3]
respectively as shown in figure below.
Ans :
As per given in question we have drawn figure below.

Download 20 Solved Sample Papers pdfs from www.cbse.online or www.rava.org.in Page 6


Mathematics Standard X Solved Sample Paper 7 www.rava.org.in

x = 10 2 = 10 # 1.414
= 14.14 m
Hence, the length of wire is 14.14 m
33. The sum of the radius of base and height of a solid
right circular cylinder is 37 cm. If the total surface
area of the solid cylinder is 1628 sq. cm, find the
volume of the cylinder. π = 227 [3]
Ans :

Here, r + h = 37 (1)
s and 2πr ^r + h h = 1628 (2)
Thus 2πr # 37 = 1628
We have tan 45º = h − 50
x 2πr = 1628
37
x = h − 50 ...(1)
r = 7 cm
and tan 60º = h
x Substituting r = 7 in (1) we have

3 =h h = 30 cm.
x
Here volume of cylinder
x = h ...(2) πr2 h = 22 # 7 # 7 # 30 = 4620 cm3
3 7
From (1) and (2) we have 34. From a solid wooden sphere with 13 centimetres radius,
a cone with 18 centimetres height and maximum base
h − 50 = h is made. [3]
3
(a) Taking the base radius of the cone as r . draw a
3 h − 50 3 = h rough figure.
3 h − h = 50 3 (b) Calculate the radius of the cone.
(c) What is the volume of the cone?
h ^ 3 − 1h = 50 3
Ans :
50 ^3 + 3h
h = 50 3 = (a)
3 −1 2
h = 25 (3 + 3 ) = 75 + 25 3
= 118.30 m

or
An electric pole is 10 m high. A steel wire tied to top
of the pole is affixed at a point on the ground to keep
the pole up right. If the wire makes an angle of 45º
with the horizontal through the foot of the pole, find
the length of the wire. [Use 2 = 1.414 ]
Ans :
(b) OC = OB = 12
Let OA be the electric pole and B be the point on the
ground to fix the pole. Let BA be x . OD = 18 − OC = 18 − 13 = 5
As per given in question we have drawn figure below. r = BD = 12 cm
(c) volume of cone = 1 πr2 h = 1 # 22 # 144 # 18
3 3 7
= 2715.4 cm3

Section D
35. If the polynomial x 4 − 6x3 + 16x2 − 25x + 10 is divided
by ^x2 − 2x + k h , the remainder comes out to be x + a,
find k and a . [4]
Ans :
In TABO, we have
sin 45º = AO
AB
1 = 10
2 x
To Get 20 Solved Paper Free PDF by whatsapp add +91 89056 29969 in your class Group Page 7
Mathematics Standard X Solved Sample Paper 7 www.cbse.online

x2 − 4x + (8 − k) 2X + 3Y = 2 ...(1)
g
x − 2x + k x − 6x + 16x2 − 25x + 10
2 4 3
4X − 9Y = − 1 ...(2)
x 4 − 2x3 + kx2 Multiplying equation (1) by 3, and adding in (2) we
get
− 4x3 + (16 − k) x2 − 25x + 10
− 4x3 + 8x2 − 4kx 10X = 5 & X = 5 = 1
10 2
(8 − k) x2 − (25 − 4k) x + 10 Thus 1 =1 & x=4
x 2
(8 − k) x − (16 − 2k) x + (8k − k2)
Putting the value of X in equation (1), we get
(2k − 9) x + (10 − 8k + k2)
2 # 1 + 3y = 2
Given, remainder = x + a 2
Comparing the multiples of x 3Y = 2 − 1
^2k − 9h x = 1 # x Y =1
3
2k − 9 = 1
k = 10 = 5 Now Y =1 & 1 =1&y=9
2 3 y 3
Substituting this value of k into other portion of Hence x = 4 , y = 9 .
remainder, we get 37. TPQR is right angled at Q , QX = PR, XY = RQ and
and a = 10 − 8k + k2 = 10 − 40 + 25 XZ = PQ are drawn. Prove that XZ2 = PZ # ZQ. [4]
=− 5

or
Obtain all other zeroes of the polynomial
9x 4 − 6x3 − 35x2 + 24x − 4, if two of its zeroes are 2
and − 2 .
Ans :
As 2 and − 2 are the zeroes of 9x 4 − 6x3 − 35x2 + 24x − 4,
So ^x − 2h and ^x + 2h are its two factors and
^x − 2h^x + 2h = x − 4
2
Ans :
Dividing 9x 4 − 6x3 − 35x2 + 24x − 4 by x2 − 4
We have redrawn the given figure as below.
9x − 6x + 1
g
2

x2 − 4 9x 4 − 6x3 − 35x2 + 24x − 4


9x 4 − 36x2
− 6x3 + x2 + 24x − 4
− 6x3 + 24x
x 2
−4
x 2
−4
0

Factorising this quotient It may be easily seen that RQ = PQ

= 69x2 − 3x − 3x + 1@ and XZ = PQ or XZ || YQ

= 63x ^3x − 1h − 1 ^3x − 1h@ Similarly XY || ZQ

= 6^3x − 1h^3x − 1h@ Thus XYQZ is a rectangle.

= ^3x − 1h^3x − 1h In TXZQ, +1 + +2 = 90º ...(1)


and in TPZX, +3 + +4 = 90º ...(2)
Hence, other two zeroes are 1 , 1 .
3 3 XQ = PR or, +2 + +3 = 90º ...(3)
36. Solve the following pair of equations : [4]
From eq. (1) and (3), +1 = +3
2 + 3 = 2 and 4 − 9 = − 1
x y x y From eq. (2) and (3), +2 = +4
Ans : Due to AA similarity

2 + 3 =2 4 − 9 =− 1 TPZX ~TXZQ
We have
x y x y PZ = XZ
XZ ZQ
Substitute 1 = X and 1 = Y
x y XZ2 = PZ # ZQ Hence proved

Download 20 Solved Sample Papers pdfs from www.cbse.online or www.rava.org.in Page 8


Mathematics Standard X Solved Sample Paper 7 www.rava.org.in

or Thus x = 3 or 1
If the area of two similar triangles are equal, prove 3
that they are congruent. or cot θ = 3 or cot θ = 1
Ans : 3
Therefore θ = 30º or θ = 60º
As per given condition we have drawn the figure
below. If θ = 30º, then
cot2 30º + tan2 30º = ^ 3 h + c 1 m
2 2

3
= 3 + 1 = 10
3 3
If θ = 60º, then
cot2 60º + tan2 60º = c 1 m + ^ 3 h
2 2

3
= + 3 = 10 .
1
We have TABC ~TPQR, 3 3
39. Find the coordinates of the point which divide the line
and arTABC = arTPQR
segment joining A ^2, − 3h and B ^− 4, − 6h into three
Since TABC ~TPQR , we have equal parts. [4]
ar ^TABC h 2 2 2 Ans :
= AB2 = BC2 = CA 2 ...(1)
ar ^TPQRh PQ QR RP Let P ^x1, y1h and Q ^x2, y2h trisect the line joining
Since ar ^TABC h = ar ^TPQRh we have A ^3, − 2h and B ^− 3, − 4h .
ar ^TABC h As per question, line diagram is shown below.
=1
ar ^TPQRh P divides AB in the ratio of 1: 2 and Q divides AB
From equation (1), we get in the ratio 2:1.
By section formula
AB2 = BC2 = CA2 = 1
my2 + ny1
PQ2 QR2 RP2 x1 = mx2 + nx1 and y =
1+2 m+n
AB = BC = CA = 1
1 ^− 4h + 2 ^2 h 2 ^− 6h + 1 ^− 3h
P ^x1, y1h = c m
PQ QR RP
,
2+1 2+1
AB = PQ,
− 6 − ^− 6h
BC = QR = c − 4 + 4, m
3 3
and CA = RA
= ^0, − 4h
By SSS similarity we have
2 ^− 4h + 1 ^2 h 2 ^− 6h + 1 ^− 3h
TABC , TPQR Q ^x2, y2h = c , m
2+1 2+1
38. Evaluate : [4]
12 + ^− 3h
tan2 30º sin 30º + cos 60º sin2 90º tan2 60º − 2 tan 45º cos2 0º sin 90º = c − 8 + 2, − m = ^− 2, − 5h
3 3
Ans : 40. In the figure, O is the centre of circle such that
tan2 30º sin 30º + cos 60º sin2 90º tan2 60º − 2 tan 45º cos2 0º sin 90º diameter AB = 13 cm and AC = 12 cm. BC is joined.
Find the area of the shaded region. ( π = 3.14 ) [4]
= c 1 m # 1 + 1 # ^1 h2 # ^ 3 h − 2 # 1 # 12 # 1
2 2

3 2 2

= 1 # 1 + 1 # 1 # 3 #− 2 # 1 # 1 # 1
3 2 2
= + − 2 = 1 + 9 − 12 = − 2 = − 1
1 3
6 2 6 6 3
or
If 3 cot θ − 4 cot θ +
2
3 = 0 , then find the value of
cot2 θ + tan2 θ. Ans :
Ans : We redraw the given figure as below.

We have 3 cot2 θ − 4 cot θ + 3 =0


Let cot θ = x , then we have
3 x2 − 4x + 3 =0
3 x − 3x − x +
2
3 =0
3 x ^x − 3 h − 1 ^x − 3h = 0
^x − 3 h^ 3x − 1h = 0
Radius of semi circle ACB = 13 cm
2
To Get 20 Solved Paper Free PDF by whatsapp add +91 89056 29969 in your class Group Page 9
Mathematics Standard X Solved Sample Paper 7 www.cbse.online

Area of semicircle = π r2 = 3.14 # 13 # 13


2 2 2 2

= 3.14 # 169 = 530.66 cm2


8 8
Semicircle subtend 90c at circle, thus +ACB = 90c
In ΔABC
AC2 + BC2 = AB2
122 + BC2 = 169
BC2 = ^160 − 144h = 25
BC = 5 cm
Also area Δ = 1 # Base # Hight
2

Area of ΔABC Δ = 1 # AC # BC
2

= 1 # 12 # 5
2
= 30 cm2
Area of shaded region = 530.66 − 30
8
= ^66.3325 − 30h cm2
= 36.3325 cm2
WWW.CBSE.ONLINE

Download unsolved version of this paper from


www.cbse.online

This sample paper has been released by website www.cbse.online for the benefits of the students. This paper has been prepared by
subject expert with the consultation of many other expert and paper is fully based on the exam pattern for 2019-2020. Please note
that website www.cbse.online is not affiliated to Central board of Secondary Education, Delhi in any manner. The aim of website is
to provide free study material to the students.

Download 20 Solved Sample Papers pdfs from www.cbse.online or www.rava.org.in Page 10


Mathematics Standard X Solved Sample Paper 8 www.rava.org.in

CLASS X (2019-20)
MATHEMATICS STANDARD(041)
SAMPLE PAPER-8

Time : 3 Hours Maximum Marks : 80


General Instructions :
(i) All questions are compulsory.
(ii) The questions paper consists of 40 questions divided into 4 sections A, B, C and D.
(iii) Section A comprises of 20 questions of 1 mark each. Section B comprises of 6 questions of 2 marks each. Section
C comprises of 8 questions of 3 marks each. Section D comprises of 6 questions of 4 marks each.
(iv) There is no overall choice. However, an internal choices have been provided in two questions of 1 mark each, two
questions of 2 marks each, three questions of 3 marks each, and three questions of 4 marks each. You have to
attempt only one of the alternatives in all such questions.
(v) Use of calculators is not permitted.

3. If 3x + 4y : x + 2y = 9 : 4 , then 3x + 5y : 3x − y is equal
Section A to [1]
(a) 4 : 1 (b) 1 : 4
Q.1-Q.10 are multiple choice questions. Select the (c) 7 : 1 (d) 1 : 7
most appropriate answer from the given options.
Ans : (c) 7 : 1
1. The values of x and y is the given figure are [1]
3x + 4y
=9
x + 2y 4
Hence, 12x + 16y = 9x + 18y
or 3x = 2y
x = 2y
3
Substitute x = 2 y in the required expression.
3
(a) 7, 13 (b) 13, 7 3 23 y + 5y 7y
= = 7 = 7: 1
(c) 9, 12 (d) 12, 9 3 23 y − y y 1
Ans : (a) 7, 13 4. The quadratic equation 2x2 − 5 x + 1 = 0 has [1]
Given number is 10001. Then, the factor tree of 1001 (a) two distinct real roots
is given as below (b) two equal real roots
(c) no real roots
(d) more than 2 real roots
Ans : (c) no real roots

Given equation is, 2x2 − 5x + 1 = 0


On comparing with ax2 + bx + c = 0 ,
we get a = 2 , b = − 5 and c = 1
Discriminant, D = b2 − 4ac
1001 = 7 # 11 # 13
= ^− 5 h − 4 # ^2 h # ^1 h
2
By comparing with given factor tree, we get
x = 7 , y = 13 = 5 − 8 =− 3 < 0
Since, discriminant is negative, therefore quadratic
2. If the sum of the zeroes of the polynomial
equation 2x2 − 5 x + 1 = 0 has no real roots i.e.,
f ^xh = 2x3 − 3kx2 + 4x − 5 is 6, then the value of k is [1]
imaginary roots.
(a) 2 (b) − 2
(c) 4 (d) − 4 5. There are 60 terms is an A.P. of which the first term
is 8 and the last term is 185. The 31 st term is [1]
Ans : (c) 4 (a) 56 (b) 94
Sum of the zeroes = 3k (c) 85 (d) 98
2
Ans : (d) 98
6 = 3k
2 Let d be the common difference;
Then 60th term, = 8 + 59d = 185
k = 12 = 4
3
To Get 20 Solved Paper Free PDF by whatsapp add +91 89056 29969 in your class Group Page 1
Mathematics Standard X Solved Sample Paper 8 www.cbse.online

59d = 177 n n
/ yi fi / yi fi
d =3 (c) i=1
(d) i=1
n
n
an = a + ^n − 1h d n / fi
/ yi i=1

Hence, 31th term = 8 + 30 # 3 = 98 Ans : (a) i=1


n
6. The point on the X -axis which if equidistant from the
10. A single letter is selected at random from the word
points A (− 2, 3) and B (5, 4) is [1]
“PROBABILITY”. The probability that the selected
(a) (0, 2) (b) (2, 0)
letter is a vowel is [1]
(c) (3, 0) (d) (− 2, 0) 2 3
(a) (b)
Ans : (b) (2, 0) 11 11

Let P (x, 0) be a point on X -axis such that, (c) 4 (d) 0


11
AP = BP Ans : (c) 4
11
AP 2 = BP 2
Required probability = 1 + 2 + 1 = 4
(x + 2) 2 + (0 − 3) 2 = (x − 5) 2 + (0 + 4) 2 11 11
x2 + 4x + 4 + 9 = x2 − 10x + 25 + 16 (Q.11-Q.15) Fill in the blanks.
14x = 28 11. Two polygons of the same number of sides are similar,
x =2 if all the corresponding angles are .......... [1]
Hence, required point = (2, 0) Ans : equal
7. The height of a tree, if it casts a shadow 15 m long on 12. Points (1, 5), (2, 3) and (− 2, − 11) are .......... [1]
the level of ground, when the angle of elevation of the Ans : Non-collinear
sun is 45c, is [1]
(a) 10 m (b) 14 m or
(c) 8 m (d) 15 m The value of the expression x2 + y2 is the distance of
the point P ^x, y h from the ..........
Ans : (d) 15 m
Ans : Origin
Let BC be the tree of height h meter.
Let AB be the shadow of tree. 13. The value of sin A or cos A never exceeds .......... [1]
Ans : 1

14. Tangent is perpendicular to the .......... through the


point of contact. [1]
Ans : radius

15. Two circles are drawn with same centre then the
.......... circle have bigger radius. [1]
Ans : Outer

(Q.16-Q.20) Answer the following


In TABC ,CB = 90c 16. In the given figure, AB is a 6 m high pole and DC is
a ladder inclined at an angle of 60c to the horizontal
BC = tan 45c
BA and reaches up to point D of pole. If AD = 2.54 m,
find the length of ladder. ( use 3 = 1.73 ) [1]
BC = AB = 15 m
8. Volume of a spherical shell is given by [1]
(a) 4π (R2 − r2) (b) π (R3 − r3)
(c) 4π (R3 − r3) (d) 4 π (R3 − r3)
3
Ans : (d) 4 π (R3 − r3)
3
Volume of spherical shell = 4 πR3 − 4 πr3
3 3

= 4 π (R3 − r3)
3
9. The mean of discrete observations y1, y2 ........ yn is
given by [1]
Ans :
n n
Ans :
/ yi / yi
(a) i=1
(b) i=1 We have AD = 2.54 m
n n
/i DB = 6 − 2.54 = 3.46 m
i=1

Download 20 Solved Sample Papers pdfs from www.cbse.online or www.rava.org.in Page 2


Mathematics Standard X Solved Sample Paper 8 www.rava.org.in

In TBCD , +B = 90c M a r k s Marks M o r e M o r e M o r e


sin 60c = BD obtained obtained than of than or than of
DC 5 equal to equal to equal to
3 = 3.46 10 15 20
2 DC Number 30 23 8 2
DC = 3.46 # 2 = 3.46 = 4 of student
3 1.73 (cummula-
Thus length of ladder is 4 m. tive
frequency)
17. If the circumferences of two concentric circles forming
a ring are 88 cm and 66 cm respectively. Find the Change the above data to a continuous grouped
width of the ring. [1] frequency distribution.
Ans : Ans :

Circumference of the outer circle 2πr1 = 88 cm C .I 5-10 10-15 15-20 20-25


r1 = 88 # 7 = 14 cm f 7 15 6 2
2 # 22
Circumference of the outer circle 2πr2 = 66 cm 20. A card is drawn at random from a well shuffled pack
of 52 cards. Find the probability of getting neither a
r2 = 66 # 7 = 21 cm = 10.5 cm red card nor a queen. [1]
2 # 22 2
Width of the ring Ans :
r1 − r2 = 14 − 10.5 cm = 3.5 cm
Given, Total number of cards = 52
18. Volume of two spheres are in the ratio 64 | 27 , find Number of red cards = 26
the ratio of their surface areas. [1]
Number of queens which are not red = 2
Ans :
` Cards which are neither red nor queen
Volume of I st sphere
= 64
27 = 52 − 626 + 2@
Volume of II nd
= 24
4
πr13
3
= 64
4
πr23 27 ` Required Probability = 24
3 52
r13 = 43
r23 33 = 6
13
r1 = 4

Ratio of their surface areas


r2 3
Section B
4A1 /A2 πr12 21. Find the HCF and LCM of 90 and 144 by the method
= a r1 k
2

4πr22 r2 of prime factorization. [2]


2 Ans :
= b4l
3
We have 90 = 9 # 10
= 16 = 2 # 32 # 5
9
or and 144 = 16 # 9

Find the volume (in cm3) of the largest right circular = 2 4 ×3 2


cone that can be cut off from a cube of edge 4.2 cm. HCF = 2×32 = 18
Ans : LCM = 2 4 ×32 ×5 = 720
Given, 22. Find the roots of the quadratic equation
Edge of the cube = 4.2 cm. 3 x2 − 2x − 3 . [2]
Height of the cone, h = 4.2 cm. Ans :

Radius of the cone, r = 4.2 = 2.1 cm. We have 3 x2 − 2x − 3 =0


2
3 x − 3x + x −
2
3 =0
Volume of the cone = 1 πr2 h 3 x ^x − 3 h + 1 ^x − 3h = 0
3
^x − 3 h^ 3 + 1h = 0
= 1 # 22 # ^2.1h2 # 4.2
3
x = 3, −1
7
Thus
= 19.4 cm3 3
19. Following distribution gives cumulative frequencies of 23. Given TABC ~TDEF , find TABC [2]
TDEF
‘more than type’ : [1]

To Get 20 Solved Paper Free PDF by whatsapp add +91 89056 29969 in your class Group Page 3
Mathematics Standard X Solved Sample Paper 8 www.cbse.online

(b) A white bead and a black bead are added to box


B and then a bead is taken from it. What is the
probability of getting a white bead from it ?
Ans :
Total number of beads in box A = 9W + 8B = 17
Total number of beads in box B = 7W + 8B = 15
(a) P (white bead from box A) = 9
17
P (white bead from box B ) = 7
15
` P (a white bead from each box)
= 9 # 7 = 21
17 15 85
(b) When a white bead and a black bead are added to
box B , then
Ans :
No. of white beads in box B = 7W + 1W = 8W
In TDEF, we have No. of black beads in box B = 8B + 1B = 9B
` Total number of beads in box B = 8W + 9B = 17
DE =^13h2 − ^12h2
= 169 − 144 = 25 = 5 Hence, P (white bead from box B ) = 8
17
arTABC = AB 2 = 3 2 = 9
Thus
arTDEF b DE l b 5 l 25 25. Find the value of λ , if the mode of the following data
or is 20 :
In the given figure, if ABCD is a trapezium in which 15, 20, 25, 18, 13, 15, 25, 15, 18, 17, 20, 25, 20, λ , 18. [2]
ED = FC
AB | | CD | | EF, then prove that AE BF
Ans :
Writing the data as discrete frequency distribution,
we get

xi fi
13 1
15 3
17 1
18 3
Ans :
20 3
We draw, AC intersecting EF at G as shown below.
λ 1
25 3

For 20 to be mode of the frequency distribution,


λ = 20 .
or
Find the unknown values in the following table :
Class Interval Frequency Cumulative
In TCAB,GF | | AB , thus by BPT we have Frequency
AG = BF ...(1) 0-10 5 5
CG FC
10-20 7 x1
In TADC, EG | | DC , thus by BPT we have
AE = AG 20-30 x2 18
...(2)
ED CG 30-40 5 x3
From equations (1) and (2), 40-50 x4 30
AE = BF . Hence Proved. Ans :
ED FC
x1 = 5 + 7 = 12
24. There are two small boxes A and B . In A, there are
9 white beads and 8 black beads. In B , there are 7 x2 = 18 − 12 = 6
white and 8 black beads. We want to take a bead from x3 = 18 + 5 = 23
a box. [2]
and x 4 = 30 − 23 = 7
(a) What is the probability of getting a white bead
from a box? 26. Two ships are approaching a light-house from opposite

Download 20 Solved Sample Papers pdfs from www.cbse.online or www.rava.org.in Page 4


Mathematics Standard X Solved Sample Paper 8 www.rava.org.in

directions. The angle of depression of two ships from Ans :


top of the light-house are 30º and 45°. If the distance The required answer is the LCM of 9, 12, and 15
between two ships is 100 m, find the height of light- minutes.
house. [2]
Finding prime factor of given number we have,
Ans :
9 = 3×3 = 32
Let AD be the height ^h meterh of the light-house and
BC is the distance between the ships. 12 = 2×2×3 = 22 ×3
15 = 3 # 5
LCM(9, 12, 15) = 22 # 32 # 5
= 150 minutes
The bells will toll next together after 180 minutes.
28. Solve for x : 1 + 2 = 1 , x ! 0, 2 , 2 . [3]
x 2x − 3 x − 2 3
Ans :
Given, BC = 100 m 1+ 2
We have = 1
In right ΔADC , x 2x − 3 x−2
tan 45c = AD 2x − 3 + 2x = 1
DC x (2x − 3) x−2
1 = h 4x − 3 = 1
DC x (2x − 3) x−2
DC = h ...(1) ^x − 2h^4x − 3h = 2x − 3x
2

In right ΔADB ,
4x2 − 11x + 6 = 2x2 − 3x
tan 30c = AD 2x2 − 8x + 6 = 0
BD
1 = h h x2 − 4x + 3 = 0
=
3 100 − DC 100 − h ^x − 1h^x − 3h = 0
100 − h = h 3 Thus x = 1, 3
100 = h + h 3 = h ^1 + 3 h 29. Determine an A.P. whose third term is 9 and when
fifth term is subtracted from 8th term, we get 6. [3]
h = 100 = 100 = 36.60
1+ 3 2.732 Ans :
` Height of tower = 36.60 m Let the first term be a , common difference be d and
n th term be an .
Section C We have a3 = 9
a + 2d = 9 ...(1)
27. Use Euclid division lemma to show that the square of
any positive integer cannot be of the form 5m + 2 or
and a8 − a5 = 6
5m + 3 for some integer m . [3] ^a + 7d h − ^a + 4d h = 6
Ans : 3d = 6
Let a be any positive integer, then by Euclid’s division d =2
algorithm a can be written as Substituting this value of d in equation (1), we get
a = bq + r , 0 # r < b and q ! ω a + 2 ^2 h = 9
Take b = 5 , then 0 # r < 5 because 0 # r < b a =5
Thus a = 5q, 5q + 1, 5q + 2, 5q + 3 and 5q + 4, So, A.P. is 5, 7, 9, 11, ...
Now a2 = (5q) 2 = 25q2 = 5 (5q2) = 5m or
a2 = (5q + 1) 2 = 25q2 + 10q + 1 = 5m + 1
If 7th term of an A.P. is 1 and 9th term is 1 , find 63rd
a2 = (5q + 2) 2 = 25q2 + 20q + 4 = 5m + 4 term.
9 7
Similarly a2 = (5q + 3) 2 = 5m + 4 Ans :
and a2 = (5q + 4) 2 = 5m + 1 Let the first term be a , common difference be d and
Thus square of any positive integer cannot be of the n th term be an .
form 5m + 2 or 5m + 3.
We have a7 = 1 & a + 6d = 1 (1)
or 9 9
Three bells toll at intervals of 9, 12, 15 minutes a9 = 1 & a + 8d = 1 (2)
respectively. If they start tolling together, after what 7 7
time will they next toll together? Subtracting equation (1) from (2) we get

To Get 20 Solved Paper Free PDF by whatsapp add +91 89056 29969 in your class Group Page 5
Mathematics Standard X Solved Sample Paper 8 www.cbse.online

2d = 1 − 1 = 2 = 1
7 9 63 63
Substituting the value of d in (2) we get
a+8# 1 = 1
63 7

a = 1 − 8 = 9−8 = 1
7 63 63 63
Thus a63 = a + (63 − 1) d
Proof : In triangles BAM and CAM ,
= 1 + 62 # 1 = 1 + 62
63 63 63 AB = AC (given)
= 63 = 1 +BAM = +CAM (given)
63
Hence, a63 = 1 and AM = AM (common)
30. In TABD, AB = AC. If the interior circle of TABC TBAM , TCAM (SAS)
touches the sides AB, BC and CA at D, E and F BM = CM
respectively. Prove that E bisects BC. [3]
and +BMA = +CMA
Ans :
As+BMA + +CMA = 180c (linear pair)
As per question we draw figure shown below.
+BMA = +CMA = 90c
AM is the perpendicular bisector of the chord BC .
AM passes through the centre O .
[Perpendicular bisector of chord of a circle passes
through the centre of the circle]
Hence, the centre of the park lies on the angle bisector
of +BAC .
32. An aeroplane, when flying at a height of 4000 m from
Since length of tangents from an external point to a the ground passes vertically above another aeroplane
circle are equal, at an instant when the angles of elevation of the two
At A, AF = AD (1) planes from the same point on the ground are 60º and
45º respectively. Find the vertical distance between
At B BE = BD (2)
the aeroplanes at that instant. (Use 3 = 1.73) [3]
At C CE = CF (3) Ans :
Now we have AB = AC Let the height first plane be AB = 4000 m and the
AD + DB = AF + FC height of second plane be BC = x m. As per given in
BD = FC ( AD = AF ) question we have drawn figure below.
BE = EC (BD = BE, CE = CF)
Thus E bisects BC.
31. Roja, Renu and Reena are three friends. They decided
to sweep a circular park near their homes. They
divided the park into three parts by two equal chords
AB and AC for convenience. [3]
(i) Prove that the centre of the park lies on the angle
bisector of +BAC .
(ii) Which mathematical concept is used in the above
problem?
Ans :
(i) Given : A circle C ^O, r h and chord AB = chord AC
. AD is bisector of +CAB . Here +BDC = 45º and +ADB = 60º
In TCBD , x = tan 45º = 1 & x = y
To prove : Centre O lies on the bisector of +BAC . y
Construction: Join BC , meeting bisector AD of
and in TABD , 4000 = tan 60º = 3
+BAC , at M . y

y = 4000 3
3
= 2306.67 m
Thus vertical distance between two,

Download 20 Solved Sample Papers pdfs from www.cbse.online or www.rava.org.in Page 6


Mathematics Standard X Solved Sample Paper 8 www.rava.org.in

4000 − y = 4000 − 2306.67 Total Cost = 33 # 500


= 1693.33 m = 16, 500
or 34. A circular sheet of radius 18 centimetre is divided into
Two men on either side of a 75 m high building and 9 equal sectors. [3]
in line with base of building observe the angles of (a) Find the measure of the central angle of a sector.
elevation of the top of the building as 30º and 60º. find (b) Find the slant height of a cone which can be made
the distance between the two men. (Use 3 = 1.73) by a sector.
(c) Find the lateral surface area of the cone thus
Ans : formed.
Let AB be the building and the two men are at P Ans :
and Q. As per given in question we have drawn figure
below. (a)

Radius = 18 cm
Central angle of the circle = 360c
In TABP , tan 30º = AB Central angle of the sector = 40c
BP
(b) Slant height = 18 cm
1 = 75
BP x =r
3 360 4
BP = 75 3 m
40 = r
In TABQ, tan 60º = AB 360 18
BQ
r = 2 cm
3 = 75
BQ (c) Curved surface area of cone = πrl
BQ = 75 = 25 3 π # 2 # 18 = 36π cm2
3
Distance between the two men,
PQ = BP + BQ = 75 3 + 25 3 Section D
= 100 3 = 100 # 1.73 = 173 35. Find the other zeroes of the polynomial
33. A tent is in the shape of cylinder surmounted by a x 4 − 5x3 + 2x2 + 10x − 8 if it is given that two zeroes
conical top of same diameter. If the height and diameter are − 2 and 2 . [4]
of cylindrical part are 2.1 m and 3 m respectively and Ans :
the slant height of conical part is 2.8 m, find the cost We have two zeroes 2 and − 2 .
of canvas needed to make the tent if the canvas is
available at the rate of Rs.500 per square meter. Use Two factors are ^x + 2 h and ^x − 2h
π = 227 [3]
g ^x h = ^x + 2 h^x − 2 h = x2 − 2 is a factor of the
Ans :
Given, given polynomial
x2 − 5x + 4
g
Height of cylinder = 2.1 m
x − 2 x − 5x3 + 2x2 + 10x − 8
2 4

Radius of cylinder = radius of cone = 3 m x4 − 2x 2


2
− 5x3 + 4x2 + 10x − 8
Slant height of cone = 2.8 m
− 5x3 − 10x
Surface area of tent
4x 2
−8
= C.S.A of cone +C.S.Aof cylinder.
4x 2
−8
= πrl + 2πrh = πr (l + 2h)
0
Area of canvas required will be surface area of tent.
Quotient = x2 − 5x + 4 = ^x − 4h^x − 1h
Thus πr (l + 2h) = 22 # 3 ^2.8 + 2 # 2.1h
7 2 Hence other zeroes are 4 and 1.
= 33 # 7 = 33 m2
7
To Get 20 Solved Paper Free PDF by whatsapp add +91 89056 29969 in your class Group Page 7
Mathematics Standard X Solved Sample Paper 8 www.cbse.online

or or, 3x + 4y + 4x − 4y = 10 + 4
Find all the zeros
polynomial of the or, 7x = 14
3x + 6x − 2x − 10x − 5 it two of its zeroes are
4 3 2 5 y =1
3
and − 5 Hence, x = 2 and y = 1.
3
Ans : (c)
x 2 + 2x + 1
g
We have, 3x − 5y = 4 ...(1)
3x − 5 3x 4 + 6x3 − 2x2 − 10x − 5
2
and 9x = 2y + 7 ...(2)
3x 4 − 5x 2
Multiplying equation (1) by 3 and rewriting equation
6x3 + 3x2 − 10x − 5
(2) we have
− 6x 3 − 10x
9x − 15y = 12 ...(3)
3x 2 −5
9x − 2y = 7 ...(4)
3x 2 −5
Subtracting equation (4) from equation (3),
0
− 13y = 5
Since 5 and − 5 are two zeroes of the given
3
polynomial.
3 y =− 5
13
Substituting value of y in equation (1),
So, bx −
3l b 3l
5 , x+ 5 will be its two factors
3x − 5 b − 5 l = 4
13
bx − 3 lb 3l 3^ h
5 x+ 5 = 1 3x2 − 5
3x = 4 − 25
13
is a factor of given polynomial
x = 27 = 9
Now, dividing it by 3x2 − 5 . 13 # 3 13
x2 + 2x + 1 = ^x + 1h2 = ^x + 1h^x + 1h Hence x = 9 and y = − 5
13 13
two other zeroes = − 1 and − 1
Hence all the zeroes of given polynomial 37. In TABC, the mid-points of sides BC, CA and AB
are D , E and F respectively. Find ratio of ar (TDEF)
to ar ^TABC.h
5 , 5 , − 1 and − 1
=
3 3 [4]
36. Solve the following pairs of linear equations by Ans :
elimination method. [4] As per given condition we have drawn the figure below.
(a) x + y = 5 and 2x − 3y = 4 Here F, E and D are the mid-points of AB, AC and
BC respectively.
(b) 3x + 4y = 10 and 2x − 2y = 2
(c) 3x − 5y − 4 = 0 and 9x = 2y + 7
Ans :

(a) We have, x+y = 5 ...(1)


and 2x − 3y = 4 ...(2)
Multiplying equation (1) by 3 and adding in (2) we
have
3 ^x + y h + ^2x − 3y h = 3 # 5 + 4
or, 3x + 3y + 2x − 3y = 15 + 4
Hence, FE | | BC, DE | | AB and DF | | AC
5x = 19 & x = 19
5 By mid-point theorem,

Substituting x = 19 in equation (1), If DE || BA then DE || BF


5
and if FE || BC then FE || BD
19 + y = 5 Therefore FEDB is a parallelogram in which DF is
5 diagonal and a diagonal of Parallelogram divides it
y = 5 − 19 = 25 − 19 = 6 into two equal Areas.
5 5 5
Hence ar ^TBDF h = ar ^TDEF h ...(1)
Hence, x = 19 and y = 6
5 5 Similarly ar ^TCDE h = ar ^TDEF h ...(2)
(b) We have, 3x + 4y = 10 ...(1) ^TAFE h = ar ^TDEF h ...(3)
and 2x − 2y = 2 ...(2) ^TDEF h = ar ^TDEF h ...(4)
Multiplying equation (2) by 2 and adding in (1), Adding equation (1), (2), (3) and (4), we have
^3x + 4y h + 2 ^2x − 2y h = 10 + 2 # 2 ar ^TBDF h + ar ^TCDE h + ar ^TAFE h + ar ^TDEF h

Download 20 Solved Sample Papers pdfs from www.cbse.online or www.rava.org.in Page 8


Mathematics Standard X Solved Sample Paper 8 www.rava.org.in

= 4ar ^TDEF h suitable values of A and B. [4]


ar ^TABC h = 4ar ^TDEF h Ans :
ar ^TDEF h
=1 We have tan ^A + B h = tan A + tan B
ar ^TABC h 4 1 − tan A tan B
or
(i) tan 75º = tan ^45º + 30ºh
In TABC, AD is the median to BC and in TPQR, PM
= tan 45º + tan 30º
is the median to QR. If AB = BC = AD . Prove that 1 $ tan 45º $ tan 30º
PQ QR PM
TABC~TPQR . 1+ 1
Prove that TABC~TPQR. = 3
= 3 +1
1− 1
3 −1
Ans : 3

^ 3 + 1h^ 3 + 1h
As per given condition we have drawn the figure =
below. ^ 3 − 1h^ 3 + 1h
= 3 + 22 3 + 1 = 4 + 2 3
^ 3 h − ^1 h2 2
Hence tan 75º = 2 + 3
(ii) tan 90º = tan ^60º + 30ºh
= tan 60º + tan 30º
1 − tan 60º tan 30º
3+1
3+ 1
3
= 3
=
1− 3# 1 0
3

Hence, tan 90º = 3


or
In an acute angled triangle ABC, if sin ^A + B − C h = 1
2
and cos ^B + C − Ah = 1 , find +A, +B and +C .
2
Ans :

We have sin ^A + B − C h = 1 = sin 30º


2
In TABC , AD is the median, therefore or, A + B − C = 30º ...(1)
BC = 2BD
and cos ^B + C − Ah = 1 = cos 45º
and in TPQR , PM is the median, 2
QR = 2QM or, B + C − A = 45º ...(2)
AB = AD = BC Adding equation (1) and (2), we get
Given,
PQ PM QR
2B = 75º
AB = AD = 2BD
or, or, B = 37.5º
PQ PM 2QM
Now subtracting equation (2) from equation (1) we
In triangles ABD and PQM, get,
AB = AD = BD 2 ^A − C h = − 15º
PQ PM QM
or, A − C = 7.5º ...(3)
By SSS similarity we have
Now A + B + C = 180º
TABD ~TPQM
A + B + C = 180º
By CPST we have
A + C = 180c − 37.5c = 142.5º ...(4)
+B = +Q,
Adding equation (3) and (4), we have
In TABC and TPQR,
2A = 135º
AB = BC
PQ QR or, A = 67.5º
By SAS similarity we have and, C = 75º
+B = +Q, Hence, +A = 67.5º, +B = 37.5º, +C = 75º
Thus TABC ~TPQR. Hence Proved. 39. Find the area of a quadrilateral ABCD , the co-
38. Given that tan ^A + B h = tan A + tan B , ordinates of whose vertices are A ^− 3, 2h, B ^5, 4h,
1 − tan A tan B C ^7, − 6h and D ^− 5, − 4h . [4]
find the values of tan 75º and tan 90º by taking Ans :

To Get 20 Solved Paper Free PDF by whatsapp add +91 89056 29969 in your class Group Page 9
Mathematics Standard X Solved Sample Paper 8 www.cbse.online

As per question the quadrilateral is shown below.

Area of triangle ABD


Δ ABD = 1 − 3 ^8h + 5 ^− 6h + − 5 ^2 − 4h
2
= 22 sq. units
Area of triangle BCD
Δ BCD = 1 5 ^− 2h + 7 ^− 8h − 5 ^10h
2
= 58 sq. units
Area ABCD = Δ ABD + Δ BCD
= 22 + 58 = 80 sq. units
40. Four equal circles are described at the four corners of
a square so that each touches two of the others. The
shaded area enclosed between the circle is 24 cm2.
7
Find the radius of each circle. [4]
Ans :
As per question statement the figure is shown below.

Let r cm be the radius of each circle.


Area of square – Area of 4 sectors = 24 cm2
7
^ h b 360º l
2 r 2
− 4 90 π r 2
= 24
# 7

4r2 − 22 r2 = 24
7 7
28r2 − 22r2 = 24
7 7
6r2 = 24
r2 = 4
r =! 2
Thus radius of each circle is 2 cm.
WWW.CBSE.ONLINE

Download unsolved version of this paper from


www.cbse.online

This sample paper has been released by website www.cbse.online for the benefits of the students. This paper has been prepared by
subject expert with the consultation of many other expert and paper is fully based on the exam pattern for 2019-2020. Please note
that website www.cbse.online is not affiliated to Central board of Secondary Education, Delhi in any manner. The aim of website is
to provide free study material to the students.
Download 20 Solved Sample Papers pdfs from www.cbse.online or www.rava.org.in Page 10
Mathematics Standard X Solved Sample Paper 9 www.rava.org.in

CLASS X (2019-20)
MATHEMATICS STANDARD(041)
SAMPLE PAPER-9
Time : 3 Hours Maximum Marks : 80
General Instructions :
(i) All questions are compulsory.
(ii) The questions paper consists of 40 questions divided into 4 sections A, B, C and D.
(iii) Section A comprises of 20 questions of 1 mark each. Section B comprises of 6 questions of 2 marks each. Section
C comprises of 8 questions of 3 marks each. Section D comprises of 6 questions of 4 marks each.
(iv) There is no overall choice. However, an internal choices have been provided in two questions of 1 mark each, two
questions of 2 marks each, three questions of 3 marks each, and three questions of 4 marks each. You have to
attempt only one of the alternatives in all such questions.
(v) Use of calculators is not permitted.

4. ^x2 + 1h2 − x2 = 0 has [1]


Section A (a) four real roots (b) two real roots
(c) no real roots (d) one real root
Q.1-Q.10 are multiple choice questions. Select the
most appropriate answer from the given options. Ans : (c) no real roots

1. Which of the following rational number have non- Given equation is,
terminating repeating decimal expansion? [1] ^x2 + 1h2 − x2 = 0
(a) 31
3125
(b) 71
512 x 4 + 1 + 2x2 − x2 = 0 8a ^a + b h2 = a2 + b2 + 2abB
23 x 4 + x2 + 1 = 0
(c) (d) None of these
200 Let, x2 = y
Ans : (d) None of these ^x h + x + 1 = 0
2 2 2

3125, 512 and 200 has factorization of the form y2 + y + 1 = 0


2m # 5n (where m and n are whole numbers). So On comparing with ay2 + by + c = 0 ,
given fractions has terminating decimal expansion.
we get a = 1, b = 1 and c = 1
2. If the sum of the zeroes of the polynomial
f ^xh = 2x3 − 3kx2 + 4x − 5 is 6, then the value of k is [1] Discriminant, D = b2 − 4ac
(a) 2 (b) − 2 = ^1 h2 − 4 ^1 h^1 h
(c) 4 (d) − 4 = 1 − 4 =− 3
Ans : (c) 4 Since, D < 0
y2 + y + 1 = 0
Sum of the zeroes = 3k
2 i.e., x 4 + x2 + 1 = 0

6 = 3k or ^x2 + 1h2 − x2 = 0 has no real roots.


2
5. An AP starts with a positive fraction and every
k = 12 = 4 alternate term is an integer. If the sum of the first 11
3 terms is 33, then the fourth term is [1]
3. A fraction becomes 4 when 1 is added to both the
(a) 2 (b) 3
numerator and denominator and it becomes 7 when 1 is
subtracted from both the numerator and denominator. (c) 5 (d) 6
The numerator of the given fraction is [1] Ans : (a) 2
(a) 2 (b) 3
Given, S11 = 33
(c) 5 (d) 15
26 @
11 2a + 10d = 33 & a + 5d = 3
Ans : (d) 15
i.e., a6 = 3 & a 4 = 2
Let the fraction be x ,
y [Since, Alternate terms are integers and the given sum
x+1 = 4 is possible]
...(1)
y+1 6. C is the mid-point of PQ , if P is (4, x), C is (y, − 1)
x−1 = 7 and Q is (− 2, 4), then x and y respectively are [1]
and ...(2) (a) − 6 and 1 (b) − 6 and 2
y−1
Solving (1) and (2), (c) 6 and − 1 (d) 6 and − 2
We have x = 15 , y = 3 , Ans : (a) − 6 and 1
i.e. x = 15 Since, C (y, − 1) is the mid-point of P (4, x) and
Q (− 2, 4).
To Get 20 Solved Paper Free PDF by whatsapp add +91 89056 29969 in your class Group Page 1
Mathematics Standard X Solved Sample Paper 9 www.cbse.online

We have, 4−2 = y 12. Point on the X -axis which is equidistant from (2, − 5)
2 and (− 2, 9) is .......... [1]
and 4 + x =− 1 Ans : (− 7, 0)
2
y =1 or
and x =− 6 Relation between x and y if the points (x, y), (1, 2)
7. In the adjoining figure, the length of BC is [1] and (7, 0) are collinear is .......... [1]
Ans : x + 3y = 7

13. Triangle in which we study trigonometric ratios is


called .......... [1]
Ans : Right Triangle

14. The common point of a tangent to a circle and the


circle is called .......... [1]
(a) 2 3 cm (b) 3 3 cm Ans : Point of contact
(c) 4 3 cm (d) 3 cm
15. Only two .......... can be drawn to a circle from an
Ans : (d) 3 cm external point. [1]
In TABC , Ans : Tangents

sin 30c = BC (Q.16-Q.20) Answer the following


AC
1 = BC 16. A ladder, leaning against a wall, makes an angle of 60c
2 6 with the horizontal. If the foot of the ladder is 2.5 m
away from the wall, find the length of the ladder. [1]
BC = 3 cm
Ans :
8. The volume of a largest sphere that can be cut from
Let the length of ladder be x . As per given in question
cylindrical log of wood of base radius 1 m and height
we have drawn figure below.
4 m, is [1]
(a) 16 πm 3 8
(b) π m 3
3 3
(c) 4 π m3 (d) 10 π m3
3 3
Ans : (c) 4 π m3
3
Volume of sphere = 4 πr3 = 4 π (1) 3
3 3

= 4 πm 3
3
9. If the coordinates of the point of intersection of less
than ogive and more than ogive is (13.5,20), then the In TACB , +C = 60c
value of median is [1]
cos 60c = 2.5
(a) 13.5 (b) 20 AC
(c) 33.5 (d) 7.5 1 = 2.5
Ans : (a) 13.5 2 AC

The abscissa of point of intersection gives the median AC = 2 # 2.5 = 5 m


of the data. So, median is 13.5. 17. The diameter of two circle with centre A and B are
10. A three digit number is to be formed using the digits 16 cm and 30 cm respectively. If area of another circle
3, 4, 7, 8 and 2 without repetition. The probability with centre C is equal to the sum of areas of these two
that it is an odd number is [1] circles, then find the circumference of the circle with
centre C. [1]
(a) 2 (b) 1
5 5 Ans :
(c) 4 (d) 3 As we know that,
5 5
Area of circle = πr2,
Ans : (a) 2 Let the radius of circle with centre C = R
5
According to question we have,
(Q.11-Q.15) Fill in the blanks.
π (8) 2 + π (15) 2 = πR2
11. .......... theorem states that if a line is drawn parallel
to one side of a triangle to intersect the other two 64π + 225π = πR2
sides in distinct points, the other two sides are divided 289π = πR2
in the same ratio. [1] R2 = 289 or R = 17 cm
Ans : Basic proportionality
Circumference of circle
Download 20 Solved Sample Papers pdfs from www.cbse.online or www.rava.org.in Page 2
Mathematics Standard X Solved Sample Paper 9 www.rava.org.in

2πR = 2π # 17 consonant. [1]


= 34π cm Ans :
18. 12 solid spheres of the same size are made by melting In the English language there are 26 alphabets.
a solid metallic cone of base radius 1 cm and height of Consonant are 21. The probability of chosen a
48 cm. Find the radius of each sphere. [1] consonant
Ans : P = 21
26
No. of spheres = 12
Radius of cone, r = 1 cm Section B
Height of the cone = 48 21. Using Euclid’s algorithm, find the HCF of 240 and
Volume of 12 spheres = Volume of cone 228. [2]
Let the radius of sphere be R . Ans :

12 # 4 πR3 = 1 πr2 h We have 240 = 228×1 + 12


3 3
and 228 = 12×19 + 0
12 # 4 πR3 = 1 π # ^1 h2 # 48
3 3 Hence, HCF of 240 and 228 = 12
R3 = 1 22. Solve for x : x2 − ^ 3 + 1h x + 3 =0 [2]
R = 1 cm Ans :

or w.f. x2 − ^ 3 + 1h x + 3 =0
Three cubes of iron whose edges are 3 cm, 4 cm and x −
2
3 x − 1x + 3 =0
5 cm respectively are melted and formed into a single x ^x − 3 h − 1 ^x − 3h = 0
cube, what will be the edge of the new cube formed ?
^x − 3 h^x − 1h = 0
Ans :
Thus x = 3,x = 1
Let the edge of single cube be x cm. 23. In the given figure, G is the mid-point of the side PQ
Volume of single cube= Volume of three cubes of TPQR and GH | | QR. Prove that H is the mid-
x3 = ^3 h3 + ^4 h3 + ^5 h3 point of the side PR or the triangle PQR. [2]
= 27 + 64 + 125 = 216
x = 6 cm
19. In the following frequency distribution, find the
median class. [1]

Height (in 104- 145- 150- 155- 160- 165-


cm) 145 150 155 160 165 170
Frequency 5 15 25 30 15 10
Ans : Ans :
Since G is the mid-point of PQ we have
Height Frequency c.f.
PG = GQ
140-145 5 5
PG = 1
145-150 15 20 GQ
150-155 25 45 According to the question, GH | | QR , thus
155-160 30 75 PG = PH (By BPT)
GQ HR
160-165 15 90
165-170 10 100 1 = PH
HR
/ f = 100 PH = HR. Hence proved.
Hence, H is the mid-point of PR .
N = 100
N th term = 100 = 50 th Mean or
&
2 2 In a rectangle ABCD, E is a point on AB such that
Hence, Median class in 155 − 160 . AE = 2 AB. If AB = 6 km and AD = 3 km, then find
3
20. A letter of English alphabet is chosen at random. DE.
Determine the probability that the chosen letter is a Ans :
As per given condition we have drawn the figure
To Get 20 Solved Paper Free PDF by whatsapp add +91 89056 29969 in your class Group Page 3
Mathematics Standard X Solved Sample Paper 9 www.cbse.online

below. Median = N th
2
= 28 = 14 th term
2
` Median class : 40-50 &Lower limit = 40
and Modal class : 40-50 & Upper limit = 50
Their sum = 40 + 50 = 90
26. Due to sudden floods, some welfare associations
jointly requested the government to get 100 tents
We have AE = 2 AB = 2 # 6 = 4 km fixed immediately and offered to contribute 50% of
3 3 the cost. If the lower part of each tent is of the form
In right triangle ADE, of a cylinder of diameter 4.2 m and height 4 m with
DE2 = ^3 h2 + ^4 h2 = 25 the conical upper part of same diameter but of height
Thus DE = 5 km 2.8 m and the canvas to be used cost ` 100 per sq.m,
find the amount, the associations will have to pay.
24. A box contains 8 black beads and 12 white beads.
[Use π = 22 ] [2]
Another box contains 9 black beads and 6 white beads. 7
One bead from each box is taken. [2] Ans :
(a) What is the probability that both beads are
black? Given, Height of upper conical part
(b) What is the probability of getting one black bead h = 2.8 m
and one white bead ?
and radius, r = 4.2 = 2.1 m
Ans : 2
Total number of cases, 20 # 15 = 300 Slant height, l = h2 + r2
Both are black 8 # 9 = 72
= ^2.8h2 + ^2.1h2
(a) Probability of getting both black = 72 = 6
300 25 = 7.84 + 4.41 = 3.5 m
One black and one white 8 # 6 + 12 # 9 = 156 Surface area of tent = 2πrh + πrl
(b) Probability = 156 = 13 Area of canvas for 1 tent
300 25
25. The mean and median of 100 observation are 50 and = 2 # 22 # 2.1 # 4 + 22 # 2.1 # 3.5
7 7
52 respectively. The value of the largest observation
is 100. It was later found that it is 110. Find the true = 6.6 ^8 + 3.5h
mean and median. [2] = 6.6 # 11.5 m2
Ans : Area for 100 tents = 6.6 # 11.5 # 100
/ fx = 66 # 115 m2 = 7590 m2
Mean =
/f Cost of 100 tents = ` 7590 # 100
& 50 =
/ fx 50 % cost = 50 # 7590 # 100
100 100
& / fx = 5000 = ` 379500
Value : Helping the flood victims.
/ fx' = 5000 − 100 + 110 = 5010
` Correct Mean = 5010 = 50.1
100 Section C
Median will remain same i.e. median = 52 . 27. Find the HCF, by Euclid’s division algorithm of the
numbers 92690, 7378 and 7161. [3]
or
Ans :
Find the sum of the lower limit of the median class
and the upper limit of the modal class : By using Euclid’s Division Lemma, we have

Classes 10- 20- 30- 40- 50- 60- 92690 = 7378 # 12 + 4154
20 30 40 50 60 70 7378 = 4154×1 + 3224
Frequency 1 3 5 9 7 3 4154 = 3224×1 + 930
Ans : 3224 = 930×3 + 434
930 = 434×2 + 62
Class 10- 20- 30- 40- 50- 60-
20 30 40 50 60 70 434 = 62×7 + 0
Frequency 1 3 5 9 7 3 HCF (92690, 7378) = 62

Cumulative 1 4 9 18 25 28 Now, using Euclid’s Division Lemma on 7161 and 62,


Frequency we have

Download 20 Solved Sample Papers pdfs from www.cbse.online or www.rava.org.in Page 4


Mathematics Standard X Solved Sample Paper 9 www.rava.org.in

7161 = 62×115 + 31 = 6n + 2
62 = 31×2 + 0 Thus A.P. is 8, 14, 20, .......
Thus HCF (7161, 62) = 31 Now a15 = a + 14d = 8 + 14 ^6 h = 92
Hence, HCF of 92690, 7378 and 7161 is 31. or
or Divide 56 in four parts in A.P. such that the ratio
Find HCF and LCM of 16 and 36 by prime factorization of the product of their extremes (1st and 4rd ) to the
and check your answer. product of means (2nd and 3rd ) is 5: 6.
Ans : Ans :
Let the four numbers be a − 3d, a − d, a + d, a + 3d
Finding prime factor of given number, we have, Now a − 3d + a − d + a + d + a + 3d = 56
16 = 2×2×2×2 = 2 4 4a = 56 & a = 14
36 = 2×2×3×3 = 22 ×32 Hence numbers are 14 − 3d, 14 − d, 14 + d, 14 + 3d
HCF(16, 36) = 2×2 = 4 Now, according to question,
LCM (16, 36) = 2 4 ×32 ^14 − 3d h^14 + 3d h
=5
= 16×9 = 144 ^14 − d h^14 + d h 6
To check HCF and LCM by using formula 196 − 9d2 = 5
HCF(a, b) ×LCM(a, b) = a # b 196 − d2 6

or, 4×144 = 16×36 6 ^196 − 9d2h = 5 ^196 − d2h


576 = 576 6 # 196 − 54d2 = 5 # 196 − 5d2
^6 − 5h # 196 = 49d
2
Thus LHS = RHS
28. Solve for x : [3] d2 = 196 = 4
49
1 + 1 = 2 ; x ! 1, 2, 3
^x − 1h^x − 2h ^x − 2h^x − 3h 3 d =! 2
Ans : Thus numbers are a − 3d = 14 − 3 # 2 = 8

1 1 a − d = 14 − 2 = 12
We have + =2
^x − 1h^x − 2h ^x − 2h^x − 3h 3 a + d = 14 + 2 = 16
x−3+x−1 =2 a + 3d = 14 + 3 # 2 = 20
^x − 1h^x − 2h^x − 3h 3
Thus required A.P is 8, 12, 16, 20.
2x − 4 =2
^x − 1h^x − 2h^x − 3h 3 30. Two tangents TP and TQ are drawn to a circle with
centre O from an external point T . Prove that [3]
2 ^x − 2h
=2 +PTO = +OPQ
^x − 1h^x − 2h^x − 3h 3
Ans :
2 =2
^x − 1h^x − 3h 3 As per question we draw figure shown below.
3 = ^x − 1h^x − 3h
x − 4x + 3 = 3
2

x2 − 4x = 0
x ^x − 4h = 0
Thus x = 0 or x = 4
29. The sum of n terms of an A.P. is 3n2 + 5n. Find the
A.P. Hence find its 15th term. [3]
Ans : Let +TPQ be θ . the tangent is perpendicular to the
Let the first term be a , common difference be d , n th end point of radius,
term be an and sum of n term be Sn +TPO = 90º
Now Sn = 3n + 5m
2
Now +TPQ = +TPO − θ
Sn − 1 = 3 ^n − 1h + 5 ^n − 1h
2
= ^90º − θh
= 3 ^n + 1 − 2n h + 5n − 5
2 Since, TP = TQ and opposite angels of equal sides
are always equal, we have
= 3n2 + 3 − 6n + 5n − 5
+TQP = ^90º − θh
= 3n2 − n − 2
Now in ΔTPQ we have
a n = Sn − Sn − 1
+TPQ + +TQP + +PTQ = 180º
= 3n2 + 5n − ^3n2 − n − 2h

To Get 20 Solved Paper Free PDF by whatsapp add +91 89056 29969 in your class Group Page 5
Mathematics Standard X Solved Sample Paper 9 www.cbse.online

90º − θ + 90º − θ + +PTQ = 180º (Use 3 = 1.73) [3]


+PTQ = 180º − 180º + 2θ = 2θ Ans :
Hence +PTQ = 2+OPQ . As per given in question we have drawn figure below.
31. Three Students Priyanka, Sania and David are
Protesting against killing innocent animals for
commercial purposes in a circular park of radius 20 m.
They are standing at equal distance on its boundary
by holding banners in their hands. [3]
(i) Find the distance between each of them?
(ii) Which mathematical concept is used in it?
Ans :
(i) Let us assume that A, B and C are the position
of Priyanka, Sania and David respectively on the
boundary of circular park with centre O .
Draw AD = BC
Since the centre of the circle coincides with the
x = tan 45º = 1 & x = y
centroid of the equilateral TABC . y
Radius of circumscribed circle = 3 AD x + 7 = tan 60º =
2 3
x
20 = 3 AD 7 = ^ 3 − 1h x
2
x = 7
AD = 20 # 3 3 −1
2
AD = 30 m
7 ^ 3 + 1h
x =
^ h ^ 3 + 1h
Now, AD = BC , and letAB = BC = CA = x #
3 − 1
BD = CD = 1 BC = x 7 ^ 3 + 1h 7 (2.73)
2 2 x = = = 9.6 m
2 2
or
An aeroplane, when flying at a height of 4000 m from
the ground passes vertically above another aeroplane
at an instant when the angles of elevation of the two
planes from the same point on the ground are 60º and
45º respectively. Find the vertical distance between
the aeroplanes at that instant. (Use 3 = 1.73)
Ans :
Let the height first plane be AB = 4000 m and the
height of second plane be BC = x m. As per given in
question we have drawn figure below.
In rt. TBDA , +D = 90c
By Pythagoras Theorem, we have
AB2 = BD2 + AD2

x2 = d x n + ^30h2
2

2
2
x −
2 x = 900
4
3 x2 = 900
4
x2 = 900 # 4 = 1200
3
x = 1200 = 20 3
Hence, distance between each of them is 20 3 .
(ii) Properties of circle, equilateral triangle and
Here +BDC = 45º and +ADB = 60º
Pythagorean theorem.
In TCBD , x = tan 45º = 1 & x = y
32. A 7 m long flagstaff is fixed on the top of a tower y
standing on the horizontal plane. From point on the
ground, the angles of elevation of the top and bottom and in TABD , 4000 = tan 60º = 3
y
of the flagstaff are 60º and 45º respectively. Find the
height of the tower correct to one place of decimal. y = 4000 3 = 2309.40 m
3

Download 20 Solved Sample Papers pdfs from www.cbse.online or www.rava.org.in Page 6


Mathematics Standard X Solved Sample Paper 9 www.rava.org.in

Thus vertical distance between two, Hence, distance between the tower and the building
4000 − y = 4000 − 2309.40 = 1690.59 m = 30 m
(c) In right TACB
33. A solid sphere of diameter 6 cm is dropped in a right
circular cylindrical vessel partly filled with water. AC = BC tan 50c = 30 # 1.19
The diameter of the cylindrical vessel is 12 cm. If the = 35.7 m
sphere is completely submerged into water, by how Hence, height of the tower
much will the level of water rise in the cylindrical = AC + CE + EG
vessel ?
= 35.7 + 1.6 + 9.2 = 46.5 m
Ans :
Let the rise in level of water be h cm.
Radius of sphere = 3 cm. radius of cylinder
Section D
= 12 = 6 cm 35. Show that 3 is a zero of the polynomial 2x2 − x2 − 13x − 6.
2 Hence find all the zeroes of this polynomial. [4]
Volume of water displaced in cylinder will be equal to Ans :
the volume of sphere.
2x2 + 5x + 2
Thus πr 2 h = 4 πr 3
3 g
x − 3 2x3 − x2 − 13x − 6
2x3 − 6x2
π#6#6#h = 4 #π#3#3#3
3 5x2 − 13x − 6
h = 4 # 3 # 3 # 3 = 1 cm 5x2 − 15x
3#6#6
2x − 6
34. Hari, standing on the top of a building, sees the top of 2x − 6
a tower at an angle of elevation of 50c and the foot of
the tower at an angle of depression of 20c. Hari is 1.6 0
metre tall and the height of the building on which he p ^x h = 2x2 − x2 − 13x − 6
is standing is 9.2 mitres. [3] = 2 ^3 h3 − ^3 h2 − 13 ^3 h − 6
(a) Draw a rough sketch according to the given
information. = 2 ^27h − 9 − 39 − 6
(b) How far is the tower from the building? =54 − 54 = 0
(c) Calculate the height of the tower. So, x − 3 is a factor of p ^x h .
[sin 20c = 0.34 , cos 20c = 0.94 , tan 20c = 0.36
by long division
sin 50c = 0.77 , cos 50c = 0.64 , tan 50c = 1.19 ] Factorising the quotient, we get
Ans : = 3x2 + 4x + x + 2
(a) Rough sketch = ^2x + 1h^x + 2h
x = − 1, − 2
2
Hence, All the zeroes of p ^x h are − 1 , − 2, 3
2
or
Given that x − 5 is a factor of the polynomial
x3 − 3 5 x2 − 5x + 15 5 , find all the zeroes of the
polynomial.
Ans :

g
x2 − 2 5 x − 15
x− 5 x3 − 3 5 x2 − 5x + 15 5
x3 − 5 x2
Hari is standing at D . His height BD is 1.6 m. − 2 5 x2 − 5x + 15 5
Height of the building, DF = 9.2 m − 2 5 x2 + 10x
The angle of elevation of the top of the tower AG is 50c.
The angle of depression of the foot of the tower is 20c. − 15x + 15 5
(b) Distance between the tower and the building, − 15x + 15 5
BC = DE = FG 0
In right TBCG , Factorising the quotient we get
tan 20c = CG & 0.36 = 1.6 + 9.2 x2 − 2 5 x − 15 = x2 − 3 5 x + 5 x − 15
BC BC
= x (x − 3 5 ) + 5 ^x − 3 5 h
BC = 10.8 = 30 m
0.36 = ^x + 5 h^x − 3 5 h

To Get 20 Solved Paper Free PDF by whatsapp add +91 89056 29969 in your class Group Page 7
Mathematics Standard X Solved Sample Paper 9 www.cbse.online

^x + 5 h^x − 3 5 h = 0 & x = 5 , 3 5
All the zeroes are 5 , − 5 and 3 5 .
36. A train covered a certain distance at a uniform speed.
If the train would have been 10 km/hr scheduled time.
And, if the train were slower by 10 km/hr, it would
have taken 3 hr more than the scheduled time. Find
the distance covered by the train. [4]
Ans :
Let the actual speed of the train be x km/hr and
actual time taken y hr.
Distance = Speed × Time
= xy km
According to the given condition, we have We have +BED = +BDE

xy = ^x + 10h^y − 2h Since E is mid-point of BC ,

xy = xy − 2x + 10y − 20 or, BE = BD = EC ...(1)

2x − 10y + 20 = 0 In TBCG , DE || FG
BD = BE = 1 (from (1))
x − 5y = − 10 (1) DG EC
and xy = ^x − 10h^y + 3h
BD = DG = EC = BE [using (1)]
xy = xy + 3x − 10y − 30
In TADF, CG || FD
3x − 10y = 30 ...(2) AG = AC
Multiplying equation (1) by 3 and subtracting (By BPT)
GD CF
equation (2) from equation (1),
AG + GD = AF + CF
3 # ^x − 5y h − ^3x − 10y h = − 3 # 10 − 30 GD CF
− 5y = − 60 AD = AF
,
y = 12 GD CF
Substituting value of y equation (1), Thus AF = AD (using (1))
CF BE
x − 5 # 12 = − 10
or
or, x = − 10 + 60
In the given figure, D and E trisect BC. Prove that
or, x = 50 8AE2 = 3AC2 + 5AD2 .
Hence, the distance covered by the train
= 50 # 12
= 600 km.
37. In the figure, +BED = +BDE and E is the mid-
point of BC . Prove that AF = AD . [4]
CF BE

Ans :
As per given condition we have drawn the figure
below.

Ans :
We have redrawn the given figure as below. Here
CG | | FD .
Since D and E trisect BC , letBD = DE = EC be x .
Then BE = 2x and BC = 3x
In TABE , AE2 = AB2 + BE2 = AB2 + 4x2 ...(1)
Download 20 Solved Sample Papers pdfs from www.cbse.online or www.rava.org.in Page 8
Mathematics Standard X Solved Sample Paper 9 www.rava.org.in

In TABC , AC2 = AB2 + BC2 = AB2 + 9x2 ...(2) = 1 # 144 = 72 sq. units
2
In TADB , AD = AB + BD = AB + x
2 2 2 2 2
...(3) 40. An elastic belt is placed around the rim of a pulley
Multiplying (2) by 3 and (3) by 5 and adding we have of radius 5 cm. From one point C on the belt elastic
3AC2 + 5AD2 = 3 ^AB2 + 9x2h + ^AB2 + x2h belt is pulled directly away from the centre O of the
pulley until it is at P, 10 cm from the point O. Find
= 3AB2 + 27x2 + 5AB2 + 5x2 the length of the belt that is still in contact with the
= 8AB2 + 32x2 pulley. Also find the shaded area.
= 8 (AB2 + 4x2) = 8AE2 (Use π = 3.14 and 3 = 1.73 ) [4]
Thus 3AC2 + 5AD2 = 8AE2 Hence Proved
38. Evaluate : [4]
sin 30º cos 45º + 4 tan 30º + sin 90º − 2 cos 90º + 1
2 2 21 2
2 24
Ans :
sin2 30º cos2 45º + 4 tan2 30º + 1 sin 90º − 2 cos2 90º + 1
2 24

= b 1 l # c 1 m + 4 c 1 m + 1 ^1 h2 − 2 ^0 h + 1
2 2 2
Ans :
2 2 3 2 24
Here AP is tangent at point A on circle.
= 1b1l+ 4 + 1 + 1 = 1 + 4 + 1 + 1
4 2 3 2 24 8 3 2 24 Thus +OAP = 90c
= 3 + 32 + 12 + 1 = 48 = 2 Now cos θ = OA = 5 = 1
24 24 OP 10 2
or or, θ = 60º
Prove that : tan θ + cot θ = 1 + tan θ + cot θ. Reflex +AOB = 360c − 60c − 60c = 240c
1 − cot θ 1 − tan θ
Ans :
Now arc ADB = 2 # 3.14 # 5 # 120
360
= 20.93 cm
1
tan θ + cot θ = tan θ + tan θ
1 − cot θ 1 − tan θ 1 − tan1 θ 1 − tan θ Hence length of elastic in contact = 20.93 cm
= tan θ +
2
1 Now, AP = 5 3 cm
tan θ − 1 ^1 − tan θh tan θ
Area, ^TOAP + TOBP h = 25 3 = 43.25 cm2
= tan θ −
2
1
tan θ − 1 ^tan θ − 1h tan θ Area of sector, OACB = 25 # 3.14 # 120
360
= tan3 θ − 1
^ tan θ − 1h tan θ = 26.16 cm2.

[`a3 − b3 = ^a − b h^a2 + b2 + ab h] Shaded Area = 43.25 − 26.16


= 17.09 cm2
^tan θ − 1h^tan θ + tan θ + 1h
2

=
^tan θ − 1h^tan θh WWW.CBSE.ONLINE

= tan θ + tan θ + 1
2

tan θ
= tan θ + 1 + cot θ
Hence Proved.
39. If A ^− 4, 8h, B ^− 3, − 4h, C ^0, − 5h and D ^5, 6h are the
vertices of a quadrilateral ABCD, find its area. [4]
Ans :
We have A ^− 4, 8h , B ^− 3, − 4h , C ^0, 5h and D ^5, 6h
Area of quadrilateral
= 1 6^x1 y2 − x2 y1h + ^x2 y3 − x3 y2h + ^x3 y 4 − x 4 y3h Download unsolved version of this paper from
2
+ ^x 4 y1 − x1 y 4h@ www.cbse.online

Area = 1 6"− 4 # ^− 4h − ^− 3h^8h, + "^− 3h^− 5h − 0


2
# ^− 4h, + "0 # 6 − 5 ^− 5h, + 6"5 # 8 − ^− 4h^6 h,@
= 1 616 + 24 + 15 − 0 + 0 + 25 + 40 + 24@
2

= 1 640 + 15 + 25 + 40 + 24@
2

To Get 20 Solved Paper Free PDF by whatsapp add +91 89056 29969 in your class Group Page 9
Mathematics Standard X Solved Sample Paper 10 www.rava.org.in

CLASS X (2019-20)
MATHEMATICS STANDARD(041)
SAMPLE PAPER-10
Time : 3 Hours Maximum Marks : 80
General Instructions :
(i) All questions are compulsory.
(ii) The questions paper consists of 40 questions divided into 4 sections A, B, C and D.
(iii) Section A comprises of 20 questions of 1 mark each. Section B comprises of 6 questions of 2 marks each. Section
C comprises of 8 questions of 3 marks each. Section D comprises of 6 questions of 4 marks each.
(iv) There is no overall choice. However, an internal choices have been provided in two questions of 1 mark each, two
questions of 2 marks each, three questions of 3 marks each, and three questions of 4 marks each. You have to
attempt only one of the alternatives in all such questions.
(v) Use of calculators is not permitted.

Section A then y2 + y − 2 = 0
^y − 1h ^y + 2h = 0
Q.1-Q.10 are multiple choice questions. Select the y =1
most appropriate answer from the given options. or y =− 2
1. The least number which is a perfect square and is x1/3
=1
divisible by each of 16, 20 and 24 is [1]
(a) 240 (b) 1600 or x1/3
=− 2
(c) 2400 (d) 3600 x = ^1 h3
Ans : (d) 3600 or x = ^− 2h3 = − 8
Hence, the real roots of the given equations are 1, -8.
The L.C.M. of 16, 20 and 24 is 240. The least multiple
of 240 that is a perfect square is 3600 and also we can 5. In an AP , if a = 3.5 , d = 0 and n = 101, then an will
easily eliminate choices (a) and (c) since they are not be [1]
perfect number. (a) 0 (b) 3.5
2. If the sum of the zeroes of the polynomial (c) 103.5 (d) 104.5
f ^xh = 2x3 − 3kx2 + 4x − 5 is 6, then the value of k is [1] Ans : (b) 3.5
(a) 2 (b) − 2
For an AP , an = a + ^n − 1h d
(c) 4 (d) − 4
= 3.5 + ^101 − 1h # 0
Ans : (c) 4
[by given conditions]
Sum of the zeroes = 3k = 3.5
2
6. If the area of the triangle formed by the points (x, 2x)
6 = 3k , (− 2, 6) and (3, 1) is 5 sq units, then x equals [1]
2
(a) 2/3 (b) 3/5
k = 12 = 4 (c) 3 (d) 5
3
3. x and y are 2 different digits. If the sum of the two Ans : (a) 2/3
digit numbers formed by using both the digits is a
perfect square, then value of x + y is [1] We have, area = 5 sq units
(a) 10 (b) 11 1 [x (6 − 1) − 2 (1 − 2x) + 3 (2x − 6)] = ! 5
(c) 12 (d) 13 2
5x − 2 + 4x + 6x − 18 = ! 10
Ans : (b) 11
15x = ! 10 + 20
The numbers that can be formed are xy and yx .
Hence, (10x + y) + (10y + x) = 11 (x + y). If this is a 15x = 30 or 10
perfect square that x + y = 11. x = 30 or 10
15 15
4. The real roots of the equation x2/3 + x1/3 − 2 = 0 are [1]
(a) 1, 8 (b) − 1, − 8 x = 2 or 2
3
(c) − 1, 8 (d) 1, − 8 7. The ratio of the length of a rod and its shadow is
Ans : (d) 1, − 8 1 : 3 then the angle of elevation of the sun is [1]
The given equation is (a) 90c (b) 45c
(c) 30c (d) 75c
x2/3 + x1/3 − 2 = 0
Ans : (c) 30c
Put x1/3 = y ,
Let AB be the rod of length h meter.
To Get 20 Solved Paper Free PDF by whatsapp add +91 89056 29969 in your class Group Page 1
Mathematics Standard X Solved Sample Paper 10 www.cbse.online

Let BC be its shadow of length 3 h meter. Required probability = 5 = 1


25 5
(Q.11-Q.15) Fill in the blanks.
11. Two figures having the same shape and size are said
to be ........... . [1]
Ans : congruent

12. Points (3, 2), (− 2, − 3) and (2, 3) form a ..........


triangle. [1]
Ans : right angle
Let angle of elevation of the sun be ‘θ ’.
In TABC , or
h = tan θ The distance of the point (x1, y1) from the origin is
3h ..........
tan θ = 1 Ans : x 12 + y 12
3
13. sin2 θ + sin2 (90c − θ) = .......... [1]
θ = 30c
Ans : 1 [Hint : sin2 (90c − θ)] = cos2 θ ]
8. A sphere is melted and half of the melted liquid is used
to form 11 identical cubes, whereas the remaining half 14. The tangent to a circle is .......... to the radius through
is used to form 7 identical smaller spheres. The ratio the point of contact. [1]
of the side of the cube to the radius of the new small Ans : perpendicular
sphere is [1]
15. A curve made by moving one point at a fixed distance
(a) b 3 l (b) b 3 l
4 1/3 8 1/3
from another is called .......... [1]
(c) (3) 1/3 (d) 2 Ans : Circle

Ans : (b) b 3 l
8 1/3
(Q.16-Q.20) Answer the following
16. If the angles of elevation of the top of a tower from
As per the given conditions, two points distant a and b ^a 2 b h from its foot and in
11a3 = 7 # 4 # π # r3 the same straight line from it are respectively 30c and
3 60c, then find the height of the tower. [1]
a = b 8 l1/3 Ans :
r 3
9. If the mean of the observation x, x + 3, x + 5, x + 7 and Let the height of tower be h . As per given in question
x + 10 is 9, the mean of the last three observation we have drawn figure below.
is [1]
(a) 10 1 (b) 10 2
3 3
(c) 11 1 (d) 11 2
3 3
Ans : (c) 11 1
3
We know,
Mean = Sum of all the observations
Total no. of observation

Mean = x − x + 3 − x + 5 − x + 7 + x − 10
5
h = tan 30c
9 = 5x + 25 From TABD,
a
5
x =4 h = a# 1 = a ...(1)
3 3
So, mean of last three observation is
From TACD, h = tan 60c
3x + 22 = 12 + 22 = 34 = 11 1 b
3 3 3 3
h = b# 3 =b 3 ...(2)
10. If in a lottery, there are 5 prizes and 20 blanks, then
the probability of getting a prize is [1] From (1) a = 3h
(a) 2 (b) 4 h
5 5 From (2) b =
3
(c) 1 (d) 1 Thus a#b = 3h# h
5
3
Ans : (c) 1
5 ab = h2

Download 20 Solved Sample Papers pdfs from www.cbse.online or www.rava.org.in Page 2


Mathematics Standard X Solved Sample Paper 10 www.rava.org.in

h = ab So, frequency of the class 30 - 40 is 3.


Hence, the height of the tower is ab . (ii) Class mark of the class : 10 − 25 = 10 + 25
2
17. The diameter of a wheel is 1.26 m. What the distance
covered in 500 revolutions. [1] = 35 = 17.5
2
Ans : 20. A bag contains cards numbered from 1 to 25. A card
Distance covered in 1 revolution is equal to is drawn at random from the bag. Find the probability
circumference of wheel and that is πd . that number is divisible by both 2 and 3. [1]
Distance covered in 500 revolutions Ans :

= 500 # π # 1.26 The numbers divisible by 2 and 3 both


= 6, 12, 18, 24
= 500 # 22 # 1.26
7 =4
= 1980 m. = 1.98 km ` P (number divisible by 2 and 3) = 4
25
18. The slant height of a bucket is 26 cm. The diameter of
upper and lower circular ends are 36 cm and 16 cm.
Find the height of the bucket. [1] Section B
Ans : 21. Given that HCF (306, 1314) = 18. Find LCM
Given, (306, 1314) [2]
Here, l = 26 cm, upper radius = 18 cm, Ans :
lower radius = 8 cm
We have HCF (306, 314) = 18
d =difference in radius = 18 − 8 = 10 cm.
LCM (306, 1314) = ?
Let h be the height of bucket
Let a = 306 and b = 1314 , then we have
h = ^26h2 − ^10h2
l2 − d2 =
LCM (a, b) # HCF (a, b) = a # b
= 676 − 100 = 576 = 24 cm.
Substituting values we have
or or, LCM (a, b) ×18 = 306 # 1314
A cylinder and a cone have base radii 5 cm and 3 cm or, LCM (a, b) = 306 # 1314
respectively and their respective heights are 4 cm and 18
8 cm. Find the ratio of their volumes. LCM (306, 1, 314) = 22, 338
Ans : 22. If one root of the quadratic equation 6x2 − x − k = 0 is
Volume of cylinder = π ^5 h2 # 4 cm3 = 100π cm3
2 , then find the value of k . [2]
3
Volume of cone = 1 π # 32 # 8 = 24π Ans :
3
Required ratio = 100π | 24π = 25 | 6 . We have 6x2 − x − k = 0
19. Consider the following distribution : [1] Substituting x = 2 , we get
3
2
M a r k s 0 or 10 or 20 or 30 or 40 or 50 or 6b 2 l − 2 − k = 0
Obtained more more more more more more 3 3

Number 63 58 55 51 48 42 6# 4 −2−k = 0
3 3
of
k = 6# 4 −2
students 9 3

(i) Calculate the frequency of the class 30 - 40. = 24 − 6 = 2


9
(ii) Calculate the class mark of the class 10 - 25. Thus k = 2 .
Ans :
23. In the given figure, in a triangle PQR, ST | | QR and
(i)
SQ = 5 and PR = 28 cm, find PT. [2]
PS 3

Class Interval c.f. f


0-10 63 5
10-20 58 3
20-30 5 4
30-40 51 3
40-50 48 6
50-60 42 42

To Get 20 Solved Paper Free PDF by whatsapp add +91 89056 29969 in your class Group Page 3
Mathematics Standard X Solved Sample Paper 10 www.cbse.online

Ans : = 30 # 25 + 30 # 25 + 30 # 25
3000
We have PS = 3
SQ 5 = 2250 = 3
3000 4
PS = 3
PS + SQ 3+5 25. Find the mean of the following distribution : [2]
PS = 3 Class 0-6 6-12 12-18 18-24 24-30
PQ 8 interval
According to the question, ST | | QR , thus Frequency 5 4 1 6 4
PS = PT (By BPT) Ans :
PQ PR

PT = PS # PR xi fi xi fi
PQ
3 5 15
= 3 # 28 = 10.5 cm 9 4 36
8
or 15 1 15
ABCD is a trapezium in which AB | | CD and its 21 6 126
diagonals intersect each other at the point O. Show 27 4 108
that AO = CO .
BO DO Total / fi = 20 / xi fi = 300
Ans :
Mean =
/ fi xi
As per given condition we have drawn the figure / fi
below.
= 300 = 15
20
or
Find the mode of the following distribution :
Classes 25- 30- 35- 40- 45- 50-
30 35 40 45 50 55
Frequency 25 34 50 42 38 14
Ans :
In TAOB and TCOD, AB || CD ,
Thus +OAB = +DCO Here, Modal class = 35 − 40
and +OBA = +ODC (Alternate angles) l = 35, f1 = 50 , f2 = 42 , f0 = 34 , h = 5
By AA similarity we have (f1 − f0)
Mode = l + h
2f1 − f0 − r2 #
TAOB ~TCOD
= 35 + 50 − 34 5
For corresponding sides of similar triangles we have 100 − 34 − 42 #
AO = BO
CO DO = 35 + 16 # 5 = 38.33
24
AO = CO . 26. A gulab jamun, contains sugar syrup upto about 30%
Hence Proved of its volume. Find approximately how much syrup
BO DO
24. There are 60 students in a class among which 30 are would be found in 45 gulab jamuns, each shaped like
boys. In another class there are 50 students among a cylinder with two hemispherical ends with length
which 25 of them are boys. If one from each class is 5 cm and diameter 2.8 cm . [2]
selected, [2]
(a) What is the probability of both being girls ?
(b) What is the probability of having atleast one girl?
Ans :
Total number of students in the first class = 60
No. of boys = 30
No. of girls = 30
Total number of students in the second class = 50
No. of boys = 25
No. of girls = 25 Ans :
(a) Probability of both being girls Radius of cylindrical portion and hemispherical
= 30 # 25 = 750 = 1 portion of a gulab jamun
60 # 50 3000 4
(b) Probability of at least one girl = 2.8 = 1.4 cm
2

Download 20 Solved Sample Papers pdfs from www.cbse.online or www.rava.org.in Page 4


Mathematics Standard X Solved Sample Paper 10 www.rava.org.in

Length of cylindrical portion Here 6q, , 6q + 2 and 6q + 4 are divisible by 2 and so


= 5 − 1.4 − 1.4 = 2.2 cm 6q, 6q + 2 and 6q + 4 are even positive integers.
But 6q + 1, 6q + 3, 6q + 5 are odd, as they are not
divisible by 2.
Thus any positive odd integer is of the form
6q + 1, 6q + 3 or 6q + 5 .

28. Solve for x : 3 x2 − 2 2 x − 2 3 = 0 [3]


Ans :

We have 3 x2 − 2 2 x − 2 3 = 0

Now, Volume of one gulab jamun = Volume of 3 x2 − 63 2 − 2 @x − 2 3 = 0


cylindrical part + 2 # Volume of hemispherical part 3x −3 2x+
2
2x−2 3 = 0
= π ^1.4h2 # 2.2 + 2 # 2 π ^1.4h3 3x − 2x+ 2x− 3 =0
2
3 3 2 2
3
3 x 6x − 3 . 2@ + 2 6x − 2 3@ = 0
= 22 # ^1.4h2 ;2.2 + 4 # 1.4E 3 x 6x − 6@ + 2 6x − 6@ = 0
7 3
^x − 6 h^ 3 x + 2 h = 0
= 22 # 1.96 # 12.2
7 3 2
Thus x = 6 =−
3
= 75.152 cm2
3 29. The sum of n terms of an A.P. is 3n2 + 5n. Find the
Volume of 45 gulab jamun A.P. Hence find its 15th term. [3]
= 45 # 75.152 = 1127.28 cm2 Ans :
3
Volume of syrup in 45 gulab jamuns Let the first term be a , common difference be d , n th
term be an and sum of n term be Sn
= 30% of 1127.28
Now Sn = 3n2 + 5m
= 30 # 1127.28 = 338.18 cm3 Sn − 1 = 3 ^n − 1h2 + 5 ^n − 1h
100
= 338 cm3 (approx). = 3 ^n2 + 1 − 2n h + 5n − 5
= 3n2 + 3 − 6n + 5n − 5

Section C = 3n2 − n − 2
a n = Sn − Sn − 1
27. Find the HCF and LCM of 510 and 92 and verify that
= 3n2 + 5n − ^3n2 − n − 2h
HCF × LCM = Product of two given numbers. [3]
Ans : = 6n + 2
Thus A.P. is 8, 14, 20, .......
Finding prime factor of given number we have,
Now a15 = a + 14d = 8 + 14 ^6 h = 92
92 = 22 ×23
510 = 30 # 17 = 2×3×5×17 or
HCF (510, 92) = 2 Find the 20 term of an A.P. whose 3rd term is 7 and
th

the seventh term exceeds three times the 3rd term by


LCM (510, 92 = 22 ×23×3×5×14
2. Also find its nth term ^an h .
= 23460 Ans :
HCF (510, 92) × LCM (510, 92) Let the first term be a , common difference be d and
= 2×23460 = 46920 n th term be an .
Product of two numbers = 510×92 = 46920 We have a3 = a + 2d = 7 (1)
Hence, HCF×LCM = Product of two numbers a7 = 3a3 + 2

or a + 6d = 3 # 7 + 2 = 23 (2)
Solving (1) and (2), we have
Show that any positive odd integer is of the form
6q + 1, 6q + 3 or 6q + 5, where q is some integer. 4d = 16 & d = 4
Ans : a + 8 = 7 & a =− 1
Let a be any positive integer, then by Euclid’s division a20 = a + 19d = − 1 + 19 # 4 = 75
algorithm a can be written as a1 = a + ^n − 1h d
a = bq + r = − 1 + 4n − 4
Take b = 6 , then 0 # r < 6 because 0 # r < b, = 4n − 5.
Thus a = 6q, 6q + 1, 6q + 2, 6q + 3, 6q + 4, 6q + 5 Hence n term is 4n − 5
th

To Get 20 Solved Paper Free PDF by whatsapp add +91 89056 29969 in your class Group Page 5
Mathematics Standard X Solved Sample Paper 10 www.cbse.online

30. A circle is inscribed in a TABC, with sides AC, AB = 9 + 9 = 18 = 3 2 units.


and BC as 8 cm, 10 cm and 12 cm respectively. Find
the length of AD, BE and CF. [3] BC = ^8 − 6h2 + ^6 − 4h2
Ans : = ^2h2 + ^2h2
As per question we draw figure shown below. = 4+4 = 8 = 2 2 units.
^8 − 3h + ^6 − 1h
2
AC = 2

= ^5h2 + ^5h2 = 25 + 25
= 50 = 5 2 units
Since, AB + BC = 3 2 + 2 2 = 5 2 = AC
` A, B and C are collinear.
Thus, Ashima, Bharti and Camella are seated in a
line.
(ii) Co-ordinate Geometry.
32. The angle of elevation of the top of a building from
We have AC = 8 cm the foot of the tower is 30c and the angle of elevation
AB = 10 cm of the top of the tower from the foot of the building is
45c. If the tower is 30 m high, find the height of the
and BC = 12 cm building. [3]
Let AF be x . Since length of tangents from an
Ans :
external point to a circle are equal,
At A, AF = AD = x (1) Let the height of the building be AB = h m. and
distant between tower and building be, BD = x m.
At B BE = BD = AB − AD = 10 − x (2) As per given in question we have drawn figure below.
At C CE = CF = AC − AF = 8 − x (3)
Now BC = BE + EC
12 = 10 − x + 8 − x
2x = 18 − 12 = 6
or x =3
Now AD = 3 cm,
BE = 10 − 3 = 7 cm
and CF = 8 − 3 = 5
31. Given figure shows the arrangement of desks in a
classroom. Ashima, Bharti and Camella are seated at
A ^3, 1h , B ^6, 4h and C ^8, 6h respectively. [3]
(i) Do you think they are seated in a line? Give In TABD tan 45c = AB
BD
reasons for your answer.
(ii) Which mathematical concept is used in the above 1 = 30
problem? x
x = 30 ...(1)
Now in TBDC ,
tan 30c = CD
BD
1 =h
3 x

3h = x & h = x ...(2)
3
From (1) and (2), we get
h = 30 = 10 3 m.
3
Therefore height of the building is 10 3 m

or
Ans : A man standing on the deck of a ship, which is 10 m
above water level, observes the angle of elevation of
(i) Using distance formula, we have
the top of a hill as 60º and the angle of depression of
AB = ^6 − 3h2 + ^4 − 1h2 the base of hill as 30º. Find the distance of the hill
= ^3 h2 + ^3 h2 from the ship and the height of the hill.
Ans :

Download 20 Solved Sample Papers pdfs from www.cbse.online or www.rava.org.in Page 6


Mathematics Standard X Solved Sample Paper 10 www.rava.org.in

As per given in question we have drawn figure below.


Here AC is height of hill and man is at E . ED = 10 Hence, the height of bottle = 5.4 cm
is height of ship from water level. As per given in 34. A boy, 1.4 metre tall standing at the edge of a river
question we have drawn figure below. bank sees the top of a tree on the edge of the other
bank at an elevation of 55º. Standing back by 3 metre,
he sees it at elevation of 45º. [3]
(a) Draw a rough figure showing these facts.
(b) How wide is the river and how tall is the
tree ? [ sin 55c = 0.8192 , cos 55c = 0.5736 ,
tan 55c = 1.4281]
Ans :
(a) The rough sketch is as follows :

In TBCE, BC = 10 m and
+BEC = 30º
Now tan 30º = BC
BE
1 = 10
3 BE
BE = 10 3 (b) Here, BF represents the tree, and CD represents
Since BE = CD , distance of hill from ship the river.
DG is the initial position of the boy and AE is
CD = 10 3 m = 10 # 1.732 m the new position.
= 17.32 m Here, AE = DG = BC = 1.4 m
Now in TABE , +AEB = 60º If DC = x m
where AB = h, BE = 10 3 m then EC = ^x + 3h m
and +AEB = 60º In right ΔECF ,
Thus tan 60º = AB tan 45c = CF
BE EC

3 = AB 1 = CF
10 3 x+3
AB = 10 3 # 3 = 30 m CF = ^x + 3h
Thus height of hill AB + 10 = 40 m In right ΔDCF ,

33. A hemispherical bowl of internal diameter 36 cm tan 55c = CF


DC
contains liquid is filled into 72 cylindrical bottles of
diameter 6 cm. Find. the height of the each bottle, if 1.4281 = x + 3
x
10% liquid is wasted in this transfer. [3]
Ans : 1.4281x = x + 3
0.4281x = 3
Volume of bowl = 2 πr3 3 =7
3 x =
0.4281
Volume of liquid in bowl = 2 π # ^18h3 cm3
3 Width of the river = CD = 7 m

Volume of one after wastage = 2 π ^18h3 # 90 cm3 Height of the tree = BF + BC + CF


= ^1.4 + 7 + 3h = 11.4 m
3 100
Volume of one bottle = πr2 h
Volume of liquid in 72 bottles
Section D
= π # ^3 h2 # h # 72 cm2
35. Obtain all other zeroes of the polynomial
Volume of bottles = volume in liquid after wastage
x 4 + 6x3 + x2 − 24x − 20 , if two of its zeroes are + 2
π # ^3 h2 # h # 72 = 2 π # ^18h2 # 90 and − 5 . [4]
3 100
Ans :
^ h
2π 18 2 90
# #
h = 3
100

π # ^3 h2 # 72
To Get 20 Solved Paper Free PDF by whatsapp add +91 89056 29969 in your class Group Page 7
Mathematics Standard X Solved Sample Paper 10 www.cbse.online

x2 + 3x + 2 = 150 km
g
x + 3x − 10 x + 6x3 + x2 − 24x − 20
2 4
x + y = 150 ...(2)
x 4 + 3x3 − 10x2 from equation (1) and (2), we have
3x3 + 11x2 − 24x − 20 x = 80 km
3x3 + 9x2 − 30x y = 70
2x2 + 6x − 20 i.e. Speed of the car I from A = 80 km/hr. and speed
2x2 + 6x − 20 of the car II from B = 70 km/hr.
0 37. In TABC, AD is a median and O is any point on AD.
BO and CO on producing meet AC and AB at E
As x=2 and −5 are the zeroes of and F respectively. Now AD is produced to X such
x 4 + 6x3 + x2 − 24x − 20.
that OD = DX as shown in figure. [4]
So ^x − 2h and ^x + 5h are two factors of Prove that :
x 4 + 6x3 + x2 − 24x − 20 and the product of factors is (1) EF | | BC
(2) AO : AX = AF : AB
^x − 2h^x + 5h = x + 3x − 10 = 0
2

Dividing x 4 + 6x3 + x2 − 24x − 20 by x2 + 3x − 10


= x 4 + 6x3 + x2 − 24x − 20
= ^x2 + 3x − 10h^x2 + 3x + 2h
= ^x − 2h^x + 5h^x + 2h^x + 1h
Hence other two zeroes are − 2 and 1.

or
Obtain all other zeroes of the polynomial
4x 4 + x3 − 72x2 − 18x , if two of its zeroes are 3 2 and
−3 2 .
Ans :
As 3 2 and − 3 2 are the zeroes of
4x 4 + x3 − 72x2 − 18x , So ^x − 3 2 h and ^x + 3 2 h
Ans :
are its two factors
Now, ^x − 3 2 h^x + 3 2 h = 0 Since BC and OX bisect each other, BXCO is a
parallelogram. Therefore BE | | XC and BX | | CF .
or, x2 − 18 = 0
In TABX, by BPT we get,
On Factorising quotient 4x2 + 2 AF = AO ..(1)
We get, x = 0 and 1 FB OX
4
In TAXC, AE = AO ...(2)
= ^x2 − 18h x ^4x + 1h EC OX
= ^x − 3 2 h^x + 3 2 h^x h^4x + 1h From equation (1) and (2), we get
Hence, other two zeroes are 0 and − 1 . AF = AE
4 FB EC
36. A and B are two points 150 km apart on a highway.
By converse of BPT we have
Two cars start A and B at the same time. If they
move in the same direction they meet in 15 hours. But EF || BC
if they move in the opposite direction, they meet in 1
From (1) we get OX = FB
hours. Find their speeds. [4] OA AF
Ans : OX + OA = FB + AF
Let the speed of the car I from A be x km/hr. OA AF
Speed of the car II from B be y km/hr. AX = AB
OA AF
Same Direction :
AO = AF
Distance covered by car I = 150 +(distance covered AX AB
by car II) Thus AO : AX = AF : AB Hence Proved
15x = 150 + 15y
or
15x − 15y = 150
Let ABC be a triangle D and E be two points on side
x − y = 10 ...(1)
AB such that AD = BE. If DP | | BC and EQ | | AC,
Opposite Direction : then prove that PQ | | AB.
Distance covered by car I + distance covered by car II Ans :

Download 20 Solved Sample Papers pdfs from www.cbse.online or www.rava.org.in Page 8


Mathematics Standard X Solved Sample Paper 10 www.rava.org.in

As per given condition we have drawn the figure or


below.
Given that tan ^A + B h = tan A + tan B , find the
1 − tan A tan B
values of tan 75º and tan 90º by taking suitable values
of A and B.
Ans :

We have tan ^A + B h = tan A + tan B


1 − tan A tan B
(i) tan 75º = tan ^45º + 30ºh

= tan 45º + tan 30º


1 $ tan 45º $ tan 30º
In TABC, DP || BC 1+ 1
= 3
= 3 +1
By BPT we have AD = AP , ...(1) 1− 1
3 3 −1
DB PC
Similarly, in TABC , EQ || AC ^ 3 + 1h^ 3 + 1h
=
BQ ^ 3 − 1h^ 3 + 1h
= BE ...(2)
QC EA
From figure, EA = AD + DE = 3 + 22 3 + 1 = 4 + 2 3
^ 3 h − ^1 h2 2
= BE + ED ( BE = AD )
Hence tan 75º = 2 + 3
= BD
(ii) tan 90º = tan ^60º + 30ºh
Therefore equation (2) becomes,
= tan 60º + tan 30º
BQ 1 − tan 60º tan 30º
= AD ...(3)
QC BD 3+1
From (1) and (3), we get 3+ 1
3
= 3
=
1− 3# 1 0
AP = BQ 3
PC QC Hence, tan 90º = 3
By converse of BPT, 39. Find the values of k for which the points A ^k + 1, 2k h,
PQ || AB Hence Proved B ^3k, 2k + 3h and C ^5k − 1, 5k h are collinear. [4]
Ans :
38. When is an equation called ‘an identity’. Prove the
trigonometric identity 1 + tan2 A = sec2 A . [4] If three points are collinear, then area covered by
Ans : given points must be zero.

Consider the triangle shown below. = 1 6x1 ^y2 − y3h + x2 ^y3 − y1h + x3 ^y1 − y2h@ = 0
2
6x1 ^y2 − y3h + x2 ^y3 − y1h + x3 ^y1 − y2h@ = 0
8^k + 1h^2k + 3 − 5k h + 3k ^5k − 2k h +
+ ^5k − 1h^2k − 2k − 3h = 0
− 3k + 3k − 3k + 3 + 9k2 − 15k + 3 = 0 = 0
2

6k2 − 15k + 6 = 0
2k2 − 5k + 2 = 0
^k − 2h^2k − 1h = 0
Thus k = 2 or k = 1
2
40. Figure depicts a racing track whose left and right ends
Let tan A = P and sec A = H
B B are semi-circular. The distance between the two inner
H2 = P2 + B2 parallel line segments is 60 m and they are each 106
2
m long. If the track is 10 m wide everywhere, find the
2
Now 1 + tan2 A = 1 + b P l = 1 + P 2 area of the track. [4]
B B

= B +2 P = H2 = b H l
2 2 2 2

B B B
= sec2 A Hence Proved.
Equations that are true no matter what value is
plugged in for the variable. On simplifying an identity
equation, one always get a true statement.

To Get 20 Solved Paper Free PDF by whatsapp add +91 89056 29969 in your class Group Page 9
Mathematics Standard X Solved Sample Paper 10 www.cbse.online

Ans :
Width of the inner parallel lines = 60 m
And the width of the outer lines = 40 # 2 = 80 m

Radius of the inner semicircles = 60 = 30 m


2

Radius of the other semicircles = 80 = 40 m


2
Area of inner rectangle = 106 # 60 = 3180 m2
Area of outer rectangle = 106 # 80 = 4240 m2.
Area of the inner semicircle
= 2 1 # 22 # 30 # 30 = 19800 m2
2 7 7
Area of outer semicircles
= 2 # 1 # 22 # 40 # 40 = 35200 m2
2 7 7
Area of racing track
= (area of outer rectangle + area of outer semicircles)
– (area of inner rectangle + area of inner semicircles)
= 4240 + 35200 − b 3180 + 19800 l
7 7

= 1060 + 15400 = 7420 + 15400


7 7

= 22820 = 3260 m2
7
Hence, area of track is 3260 m2
WWW.CBSE.ONLINE

Download unsolved version of this paper from


www.cbse.online

This sample paper has been released by website www.cbse.online for the benefits of the students. This paper has been prepared by
subject expert with the consultation of many other expert and paper is fully based on the exam pattern for 2019-2020. Please note
that website www.cbse.online is not affiliated to Central board of Secondary Education, Delhi in any manner. The aim of website is
to provide free study material to the students.
Download 20 Solved Sample Papers pdfs from www.cbse.online or www.rava.org.in Page 10

Вам также может понравиться